You are on page 1of 130

INSIGHTSIAS

IA SIMPLIFYING IAS EXAM PREPARATION

RTM COMPILATIONS
PRELIMS 2022

July 2022

www.insightsactivelearn.com | www.insightsonindia.com
Revision Through MCQs (RTM) Compilation (July - 2022)

Telegram: https://t.me/insightsIAStips
2
Youtube: https://www.youtube.com/channel/UCpoccbCX9GEIwaiIe4HLjwA
Revision Through MCQs (RTM) Compilation (July - 2022)

Table of Contents

RTM- REVISION THROUGH MCQS –1st-Jul-2022.................................................................... 4


RTM- REVISION THROUGH MCQS –2nd-Jul-2022 ................................................................... 8
RTM- REVISION THROUGH MCQS –4th-Jul-2022 ................................................................. 13
RTM- REVISION THROUGH MCQS –5th-Jul-2022 ................................................................. 18
RTM- REVISION THROUGH MCQS –6th-Jul-2022 ................................................................. 23
RTM- REVISION THROUGH MCQS –7th-Jul-2022 ................................................................. 28
RTM- REVISION THROUGH MCQS –8th-Jul-2022 ................................................................. 32
RTM- REVISION THROUGH MCQS –9th-Jul-2022 ................................................................. 37
RTM- REVISION THROUGH MCQS –11th-Jul-2022 ............................................................... 44
RTM- REVISION THROUGH MCQS –12th-Jul-2022 ............................................................... 48
RTM- REVISION THROUGH MCQS –13th-Jul-2022 ............................................................... 52
RTM- REVISION THROUGH MCQS –14th-Jul-2022 ............................................................... 58
RTM- REVISION THROUGH MCQS –15th-Jul-2022 ............................................................... 63
RTM- REVISION THROUGH MCQS –16th-Jul-2022 ............................................................... 68
RTM- REVISION THROUGH MCQS –18th-Jul-2022 ............................................................... 73
RTM- REVISION THROUGH MCQS –19th-Jul-2022 ............................................................... 77
RTM- REVISION THROUGH MCQS –20th-Jul-2022 ............................................................... 82
RTM- REVISION THROUGH MCQS –21th-Jul-2022 ............................................................... 87
RTM- REVISION THROUGH MCQS –22nd-Jul-2022 ............................................................... 91
RTM- REVISION THROUGH MCQS –23rd-Jul-2022 ............................................................... 96
RTM- REVISION THROUGH MCQS –25th-Jul-2022 ............................................................. 101
RTM- REVISION THROUGH MCQS –26th-Jul-2022 ............................................................. 105
RTM- REVISION THROUGH MCQS –27th-Jul-2022 ............................................................. 109
RTM- REVISION THROUGH MCQS –28th-Jul-2022 ............................................................. 115
RTM- REVISION THROUGH MCQS –29th-Jul-2022 ............................................................. 120
RTM- REVISION THROUGH MCQS –30th-Jul-2022 ............................................................. 125

Telegram: https://t.me/insightsIAStips
3
Youtube: https://www.youtube.com/channel/UCpoccbCX9GEIwaiIe4HLjwA
Revision Through MCQs (RTM) Compilation (July - 2022)

RTM- REVISION THROUGH MCQS –1st-Jul-2022

1. Consider the following statements:


1. Constitution of India vests the superintendence, direction and control of the conduct
of election to the office of the Vice-President of India in the Secretary General of the
Rajya Sabha.
2. As per Article 66 of the Constitution of India, the Vice-President is elected by the
members of the Electoral College.
Which of the statements given above is/are correct?
(a) 1 only
(b) 2 only
(c) Both 1 and 2
(d) Neither 1 nor 2
Ans: (b)
Explanation:
• Article 324 of the Constitution read with the Presidential and Vice-
Presidential Elections Act, 1952 and the Presidential and Vice-Presidential
Elections Rules, 1974, vests the superintendence, direction and control of
the conduct of election to the office of the Vice-President of India in the
Election Commission of India.
• As per Article 66 of the Constitution of India, the Vice-President is elected
by the members of the Electoral College.
• Electoral College consists of:
o Elected members of Rajya Sabha.
o Nominated members of Rajya Sabha.
o Elected members of Lok Sabha.
Refer: https://www.insightsonindia.com/2022/07/01/election-to-the-office-of-the-vice-
president-of-india-2022-16th-vice-presidential-election/
2. Consider the following statements
1. Vice-President is a member and chairman of Council of States.
2. Vice-President can be removed only through a formal impeachment process similar
to that of President.
3. Dr S. Radhakrishnan was elected as Vice President continuously for two terms.
Which of the above statements is/are correct?
(a) 3 only
(b) 1 and 3 only
(c) 2 and 3 only
(d) 1 only
Ans: (a)
Explanation:
• Vice-President is not a member, however he/she is the chairman of Council of
States. There is no exact procedure mentioned in Indian Constitution for the
removal of Vice President.
Refer: https://www.insightsonindia.com/2022/07/01/election-to-the-office-of-the-vice-
president-of-india-2022-16th-vice-presidential-election/
3. Which of the following statements about Leprosy is/are correct?
1. The exact mechanism of transmission is not known
2. In most parts of the world males are affected more frequently than females

Telegram: https://t.me/insightsIAStips
4
Youtube: https://www.youtube.com/channel/UCpoccbCX9GEIwaiIe4HLjwA
Revision Through MCQs (RTM) Compilation (July - 2022)

Select the correct answer using the code given bellow:


(a) 1 Only
(b) 2 Only
(c) Both 1 and 2
(d) Neither 1 nor 2
Ans: (c)
Explanation:
• S1: The disease can be transmitted from person to person by prolonged close
contact, but even today scientists are uncertain of the exact mechanism.
Readmore>>
• S2: Although leprosy affects both the sexes, in most parts of the world males are
affected more than females at a ratio of 2:1. Readmore>>
Refer: https://www.insightsonindia.com/2022/07/01/concern-over-shortage-of-leprosy-
drugs-in-the-private-sector/
4. Consider the following statements about UN World Food Programme (WFP)
1. It was created in 1961, as an experiment to provide food aid through the UN system.
2. It was awarded the Nobel Peace Prize in 2020.
3. It is headquartered in Rome, Italy.
Which of the statements given above is/are correct?
(a) 1 and 2 only
(b) 2 and 3 only
(c) 1 and 3 only
(d) 1, 2 and 3
Ans: (d)
Explanation:
• S1 and S3: The World Food Programme[a] (WFP) is the food-assistance branch of
the United Nations. It is the world's largest humanitarian organization focused
on hunger and food security, and the largest provider of school meals. Founded
in 1961, it is headquartered in Rome and has offices in 80 countries.
• S2:The World Food Programme was awarded the Nobel Peace Prize in 2020 for
its efforts to provide food assistance in areas of conflict, and to prevent the use of
food as a weapon of war and conflict.
Refer: https://www.insightsonindia.com/2022/07/01/niti-aayog-and-world-food-program-
releases-report-take-home-ration/
5. Financial Stability Report (FSR) is released by
(a) Reserve Bank of India (RBI)
(b) Bank for International Settlements (BIS)
(c) NITI Aayog
(d) World Bank Group
Ans: (a)
Explanation:
• Reserve Bank of India (RBI) released its bi-annual Financial Stability Report
(FSR)
Refer: https://www.insightsonindia.com/2022/07/01/rbis-financial-stability-report/

Telegram: https://t.me/insightsIAStips
5
Youtube: https://www.youtube.com/channel/UCpoccbCX9GEIwaiIe4HLjwA
Revision Through MCQs (RTM) Compilation (July - 2022)

6. Consider the following statements regarding Polar Satellite Launch Vehicle (PSLV) and
Geosynchronous Satellite Launch Vehicle (GSLV).
1. GSLV was developed to launch low-Earth Orbit satellites into polar and sun
synchronous orbits.
2. PSLV was developed to launch the heavier INSAT class of geosynchronous satellites
into orbit.
Which of the above statements is/are incorrect?
(a) 1 only
(b) 2 only
(c) Both 1 and 2
(d) Neither 1 nor 2
Ans: (c)
Explanation: here the directive word is incorrect!!
• Difference between PSLV and GSLV:
• India has two operational launchers- Polar Satellite Launch Vehicle (PSLV) and
Geosynchronous Satellite Launch Vehicle (GSLV).
• PSLV was developed to launch low-Earth Orbit satellites into polar and sun
synchronous orbits. It has since proved its versatility by launching
geosynchronous, lunar and interplanetary spacecraft successfully.
• On the other hand, GSLV was developed to launch the heavier INSAT class of
geosynchronous satellites into orbit. In its third and final stage, GSLV uses the
indigenously developed cryogenic upper stage.
Refer: https://www.insightsonindia.com/2022/07/01/isros-poem-platform/
7. With reference to India’s satellite launch vehicle, consider the following statements:
1. PSLVs launch the satellites useful for Earth resources monitoring whereas GSLVs
are designed mainly to launch communication satellites.
2. Satellites launched by PSLV appear to remain permanently fixed in the same
position in the sky, as viewed from a particular location on Earth.
3. GSLV MK III is a four-staged launch vehicle with the first and third stages using
solid rocket motors; and the second and fourth stages using liquid rocket engines.
Which of the statements given above is/are correct?
(a) 1 only
(b) 2 and 3
(c) 1 and 2
(d) 3 only
Ans: (a)
Explanation:
• PSLV is designed mainly to deliver the “earth-observation” or “remote-sensing”
satellites with lift-off mass of up to about 1750 Kg to Sun-Synchronous circular
polar orbits of 600-900 Km altitude. The GSLV is designed mainly to deliver the
communication-satellites to the highly elliptical (typically 250 x 3.6000 Km)
Geosynchronous Transfer Orbit (GTO). The satellite in GTO is further raised to
its final destination, viz., Geo-synchronous Earth orbit (GEO) of about 3.6000
Km altitude (and zero degree inclination on equatorial plane) by firing its in-built
on-board engines. Due to their geo-synchronous nature, the satellites in these
orbits appear to remain permanently fixed in the same position in the sky, as
viewed from a particular location on Earth, thus avoiding the need of a tracking
ground antenna and hence are useful for the communication applications.

Telegram: https://t.me/insightsIAStips
6
Youtube: https://www.youtube.com/channel/UCpoccbCX9GEIwaiIe4HLjwA
Revision Through MCQs (RTM) Compilation (July - 2022)

• Source: http://www.thehindu.com/sci-tech/science/what-is-the-difference-
between-gslv-and-pslv/article6742299.ece
• GSLV Mk 3 is a three-stage heavy lift launch vehicle developed by ISRO. The
vehicle has two solid strap-one, a core liquid booster and a cryogenic upper
stage.
Refer: https://www.insightsonindia.com/2022/07/01/isros-poem-platform/

8. What is ‘green hydrogen’, sometimes mentioned in the news?


(a) Hydrogen generated through electrolysis powered by nuclear energy.
(b) Hydrogen generated through electrolysis with solar power.
(c) Hydrogen created from fossil sources, where the carbon emissions are captured and
stored.
(d) Hydrogen produced by electrolysis using renewable energy.
Ans: (d)
Explanation:
• Hydrogen when produced by electrolysis using renewable energy is known as
Green Hydrogen which has no carbon footprint.
• Significance of Green Hydrogen:
• Green hydrogen energy is vital for India to meet its Nationally Determined
Contribution (INDC) Targets and ensure regional and national energy security,
access and availability.
• Green Hydrogen can act as an energy storage option, which would be essential
to meet intermittencies (of renewable energy) in the future.
• In terms of mobility, for long distance mobilisations for either urban freight
movement within cities and states or for passengers, Green Hydrogen can be
used in railways, large ships, buses or trucks, etc.
Refer: https://www.insightsonindia.com/2022/07/01/niti-aayog-report-harnessing-green-
hydrogen-opportunities-for-deep-decarbonisation-in-india/
9. Consider the following statements:
1. Aurangabad was founded in 1610 by Aurangzeb.
2. Recently, Maharashtra government renamed Aurangabad as Shivaji Nagar.
Which of the above statements is/are incorrect?
(a) 1 only
(b) 2 only
(c) Both 1 and 2
(d) Neither 1 nor 2
Ans: (d)
Explanation:
• Maharashtra government renamed Aurangabad as Sambhajinagar.
• Aurangabad was founded in 1610 by Malik Ambar, the Siddi general of the
Nizamshahi dynasty of Ahmadnagar.
• Chhatrapati Sambhaji Maharaj, the son and successor of Chhatrapati Shivaji
Maharaj, was tortured and killed in brutal fashion on Aurangzeb’s orders in
1689.
Refer: Facts For Prelims: https://www.insightsonindia.com/2022/07/01/mission-2023-
insights-daily-current-affairs-pib-summary-01-july-2022/
10. World Drug Report 2022 is released by
(a) WHO

Telegram: https://t.me/insightsIAStips
7
Youtube: https://www.youtube.com/channel/UCpoccbCX9GEIwaiIe4HLjwA
Revision Through MCQs (RTM) Compilation (July - 2022)

(b) UNODC
(c) WEF
(d) WTO
Ans: (b)
Explanation:
• UN Office on Drugs and Crime (UNODC) has released World Drug Report 2022
Refer: Facts For Prelims: https://www.insightsonindia.com/2022/07/01/mission-2023-
insights-daily-current-affairs-pib-summary-01-july-2022/

RTM- REVISION THROUGH MCQS –2nd-Jul-2022

11. Consider the following statements about National Human Rights Commission (NHRC):
1. It is a Constitutional Body.
2. It was established in compliance with the Paris Principles of Human Rights, 1991.
3. It is composed of a Chairperson and eight other members.
Which of the statements given above is/are not correct?
(a) 1 and 2 only
(b) 2 and 3 only
(c) 1 and 3 only
(d) 1, 2 and 3
Ans: (c)
Explanation: here the directive word is not correct!!
• S1: NHRC is a statutory body established on 12th October, 1993 under the
Protection of Human Rights Act (PHRA), 1993.
• S2: NHRC was established in compliance with the Paris Principles of Human
Rights, 1991 which were adopted for the promotion and protection of Human
Rights and were endorsed by the United Nations at its General Assembly of
1993.
• S3: The Commission consists of a Chairperson, five full-time Members and seven
deemed Members. The statute lays down qualifications for the appointment of
the Chairperson and Members of the Commission.
Refer: https://www.insightsonindia.com/2022/07/02/indias-nuclear-policy-reflects-its-
past-ideology-nhrc-chief/
12. Consider the following statements about ‘Banks Board Bureau (BBB)’:
1. It was part of the Indradhanush Plan.
2. The Governor of RBI is the Chairman of BBB.
Which of the statements given above is/are correct?
(a) 1 only
(b) 2 only
(c) Both 1 and 2
(d) Neither 1 nor 2
Ans: (a)
Explanation: About BBB:

• It was set up in February 2016 as an autonomous body– based on the


recommendations of the RBI-appointed Nayak Committee.

Telegram: https://t.me/insightsIAStips
8
Youtube: https://www.youtube.com/channel/UCpoccbCX9GEIwaiIe4HLjwA
Revision Through MCQs (RTM) Compilation (July - 2022)

• It was part of the Indradhanush Plan.


• It will make recommendations for the appointment of whole-time directors as
well as non-executive chairpersons of Public Sector Banks (PSBs) and state-
owned financial institutions.
• The Ministry of Finance takes the final decision on the appointments in
consultation with the Prime Minister’s Office.
• S2: The Chairman is selected by the central government and the RBI governor
does not head it.
Refer: https://www.insightsonindia.com/2022/07/02/financial-services-institutions-
bureau-fsib/
13. Consider the following statements regarding Anti-dumping duty.
1. Anti-dumping measures are taken to ensure fair trade for domestic industries.
2. Anti-dumping duty is meant for level playing field between domestic producers of a
product and foreign producers of the same product who can afford to sell it at a
lower price because of the subsidy they receive from their government.
Which of the above statements is/are incorrect?
(a) 1 only
(b) 2 only
(c) Both 1 and 2
(d) Neither 1 nor 2
Ans: (b)
Explanation:
• Anti-dumping duty is different from countervailing duty. The countervailing duty
is imposed in order to counter the negative impact of import subsidies to protect
domestic producers.
• Countervailing Duties (CVDs) are tariffs levied on imported goods to offset
subsidies made to producers of these goods in the exporting country.
• CVDs are meant to level the playing field between domestic producers of a
product and foreign producers of the same product who can afford to sell it at a
lower price because of the subsidy they receive from their government.
• Anti-dumping duty is a customs duty on imports providing a protection against
the dumping of goods at prices substantially lower than the normal value
whereas Countervailing duty is a customs duty on goods that have received
government subsidies in the originating or exporting country.
• Anti-dumping measures are taken to ensure fair trade and provide a level
playing field to the domestic industry.
Refer: https://www.insightsonindia.com/2022/07/02/dumping/
14. Recently, Ministry of Environment, Forest and Climate Change (MoEFCC) has notified
Forest (conservation) Rules 2022, under the
(a) Wild Life (Protection) Act, 1972
(b) Environment (Protection) Act,1986
(c) Forest (conservation) Act, 1980
(d) Biological Diversity Act, 2002
Ans: (c)
Explanation:
• ‘MoEFCC has notified Forest (conservation) Rules 2022, under the Forest
(conservation) Act, 1980.
o New Rules says:

Telegram: https://t.me/insightsIAStips
9
Youtube: https://www.youtube.com/channel/UCpoccbCX9GEIwaiIe4HLjwA
Revision Through MCQs (RTM) Compilation (July - 2022)

o On monitoring: Constitute an advisory committee, a regional empowered


committee and a screening committee at the State/UT
o Integrated Regional Office: It will examine all the linear projects (e.g.
roads, highways, etc) involving land up to 40 hectares and the use of
forest land up to 0.7 canopy density
o Time frame: A fixed time for quicker review of each project
o Responsibility to states: States are given the responsibility of settling
forest rights of forest dwellers (Forest Rights Act, 2006) and allowing
diversion of forest land.
o Allows compensatory afforestation (CA) in other states: If the state
already has over two-thirds area under green cover or over one-third
area under forest cover, then CA could be taken in other states/UTs
where the cover is less than 20%
Refer: https://www.insightsonindia.com/2022/07/02/forest-conservation-rules-2022-
notified/
15. With reference to Ashadhi Bij, consider the following statements:
1. Kutchi community of Gujarat celebrates their new year on this day.
2. The festival is associated with the beginning of rains in the Kutch region of Gujarat.
Which of the above statements is/are correct?
(a) 1 only
(b) 2 only
(c) Both 1 and 2
(d) Neither 1 nor 2
Ans: (c)
Explanation:
• Kutchi community of Gujarat celebrates their new year on this day.
Ashadhi Bij falls on the second day of Shukla paksha of Ashada month of the
Hindu calendar.
• The festival is associated with the beginning of rains in the Kutch region of
Gujarat.
• During Ashadhi Bij, the moisture in the atmosphere is checked to predict which
crop would do best in the coming monsoon
Refer: facts for prelims: https://www.insightsonindia.com/2022/07/02/mission-2023-
insights-daily-current-affairs-pib-summary-02-july-2022/
16. Consider the following statements:
1. In India, a person cannot be prosecuted more than once for the same offence.
2. Article 21 of the Constitution guarantees the right against double jeopardy.
Which of the above statements is/are correct?
(a) 1 only
(b) 2 only
(c) Both 1 and 2
(d) Neither 1 nor 2
Ans: (a)
Explanation:
• A person cannot be prosecuted more than once for the same offence.
• Article 20(2) of the Constitution guarantees the right against double jeopardy.

Telegram: https://t.me/insightsIAStips
10
Youtube: https://www.youtube.com/channel/UCpoccbCX9GEIwaiIe4HLjwA
Revision Through MCQs (RTM) Compilation (July - 2022)

• Multiple FIRs on the same incident would virtually mean multiple trials.
Approaching the Supreme Court in such situations is a procedural safeguard
against excessive litigation.
• Other judgements:
• In T T Anthony v State of Kerala, a 2001 verdict, the Supreme Court held that
there cannot be a “second FIR” on the same issue.
• In 2020, the Supreme Court in the case of Arnab Goswami v Union of
India expanded this ruling and said that similar FIRs in different jurisdictions
also violate fundamental rights.
Refer: facts for prelims: https://www.insightsonindia.com/2022/07/02/mission-2023-
insights-daily-current-affairs-pib-summary-02-july-2022/
17. Consider the following statements about the RAMP scheme:
1. It is a World Bank assisted Central Sector Scheme.
2. It aims to improve the condition of MSMEs in India.
Which of the statements given above is/are correct?
(a) 1 only
(b) 2 only
(c) Both 1 and 2
(d) Neither 1 nor 2
Ans: (c)
Explanation:
• “Raising and Accelerating MSME Performance” (RAMP) is a World Bank assisted
Central Sector Scheme.
• It has been launched to support various Resilience and Recovery
Interventions of the Ministry of Micro, Small and Medium Enterprises
(MoMSME).
• In addition to building the MoMSME’s capacity at the national level, the
RAMP program will seek to scale up implementation capacity and MSME
coverage in States.
Refer: facts for prelims: https://www.insightsonindia.com/2022/07/02/mission-2023-
insights-daily-current-affairs-pib-summary-02-july-2022/
18. Consider the following phenomena :
1. Light is affected by gravity.
2. The Universe is constantly expanding.
3. Matter warps its surrounding space-time.
Which of the above is/are the prediction/predictions of Albert Einstein’s General Theory
of Relativity, often discussed in media?
(a) 1 and 2 only
(b) 3 only
(c) 1 and 3 only
(d) 1, 2 and 3
Ans: (d)
Explanation:
• S1: Einstein’s theory of relativity has important astrophysical implications. For
example, it implies the existence of black holes—regions of space in which space
and time are distorted in such a way that nothing, not even light, can escape—as
an end-state for massive stars.

Telegram: https://t.me/insightsIAStips
11
Youtube: https://www.youtube.com/channel/UCpoccbCX9GEIwaiIe4HLjwA
Revision Through MCQs (RTM) Compilation (July - 2022)

• There is ample evidence that the intense radiation emitted by certain kinds of
astronomical objects is due to black holes; for example, micro quasars and active
galactic nuclei result from the presence of stellar black holes and super massive
black holes, respectively.
• The bending of light by gravity can lead to the phenomenon of gravitational
lensing, in which multiple images of the same distant astronomical object are
visible in the sky. Hence, statement 1 is correct.
• S2: General relativity also predicts the existence of gravitational waves, which
have since been observed directly by the physics collaboration LIGO. In addition,
general relativity is the basis of current cosmological models of a
consistently expanding universe. Hence, statement 2 is correct.
• S3: General relativity is Einstein’s law of gravity, his Justification of that
fundamental force which holds us to the surface of the Earth. Gravity, Einstein
asserted, is caused by warping of space and time or space-time. Hence,
statement 3 is correct.
• Source: https://en.wikipedia.org/wiki/General_relativity
• http://www.einstein-online.info/spotlights/light_deflection.html
• https://www.its.caltech.edu/~kip/index.html/PubScans/VI-47.pdf
Refer: UPSC CSE 2018
19. The staple commodities of export by the English East India Company from Bengal in
the middle of the 18th century were
(a) Raw cotton, oil-seeds and opium
(b) Sugar, salt, zinc and lead
(c) Copper, silver, gold, spices and tea
(d) Cotton, silk, saltpetre and opium
Ans: (d)
Explanation:
• The East India Company was originally formed in Britain for pursuing trade with
the East Indies in Southeast Asia. In fact, it ended up trading mainly with the
Indian subcontinent and China, where the main items of trade were cotton, silk,
tea, opium, and saltpetre (potassium nitrate).
• Source: https://en.wikipedia.org/wiki/East_India_Company
Refer: UPSC CSE 2018
20. With reference to the cultural history of India, consider the following statements:
1. White marble was used in Mughal Buland Darwaza and Khankah at Fatehpur Sikri.
2. Red sandstone and marble were used in making Bara lmambara and Rumi Darwaza
at Lucknow.
Which of the statements given above is/are correct?
(a) 1 only
(b) 2 only
(c) Both 1 and 2
(d) Neither 1 nor 2
Ans: (a)
Explanation:
• S1: The Buland Darwaza is made of red and buff sandstone, decorated by white
and black marble and is higher than the courtyard of the mosque. The Buland
Darwaza is symmetrical and is topped by large free standing kiosks, which are
the chhatris.

Telegram: https://t.me/insightsIAStips
12
Youtube: https://www.youtube.com/channel/UCpoccbCX9GEIwaiIe4HLjwA
Revision Through MCQs (RTM) Compilation (July - 2022)

• S2: The architectural style of the Rumi Darwaza is completely in sync with the
Nawabi architecture of Lucknow, and its significantly different from the
Mughals. The material used for the drawaza is bricks and its then coated with
lime, while the Mughals often used red sand stone. This is why the detailing on
the Darwaza is more intricate, which would be impossible to achieve in stone.
• Source: https://en.wikipedia.org/wiki/Bara_Imambara
Refer: UPSC CSE 2018

RTM- REVISION THROUGH MCQS –4th-Jul-2022


21. With reference to Rampa Rebellion, consider the following statements:
1. It marked the first major rebellion of people in Punjab towards the political order of
British in 1849.
2. It was led by Alluri Sitarama Raju.
Which of the statements given above is/are correct?
(a) 1 only
(b) 2 only
(c) Both 1 and 2
(d) Neither 1 nor 2
Ans: (b)
Explanation:
• The Rampa Rebellion of 1922, also known as the Manyam Rebellion,was
a tribal uprising, led by Alluri Sitarama Raju in Godavari Agency of Madras
Presidency, British India. It began in August 1922 and lasted until the capture
and killing of Raju in May 1924.
Refer: https://www.insightsonindia.com/2022/07/04/alluri-sitarama-raju/
22. With reference to the provisions made under the National Food Security Act, 2013,
consider the following statements:
1. The families coming under the category of ‘below poverty line (BPL)’ only are eligible
to receive subsidies food grains.
2. The eldest woman in a household, of age 18 years or above, shall be the head of the
household for the purpose of issuance of a ration card.
3. Pregnant women and lactating mothers are entitled to a ‘take-home ration’ of 1600
calories per day during pregnancy and for six months thereafter
Which of the statement given above is/are correct?
(a) 1 and 2
(b) 2 only
(c) 1 and 3
(d) 3 only
Ans: (b)
Explanation:
• Statement 1: Coverage and entitlement under Targeted Public Distribution
System (TPDS): Upto 75% of the rural population and 50% of the urban
population will be covered under TPDS, with uniform entitlement of 5 kg per
person per month. Not restricted to only BPL families.
• Statement 3: Pregnant women and lactating mothers and children in the age
group of 6 months to 14 years will be entitled to meals as per prescribed

Telegram: https://t.me/insightsIAStips
13
Youtube: https://www.youtube.com/channel/UCpoccbCX9GEIwaiIe4HLjwA
Revision Through MCQs (RTM) Compilation (July - 2022)

nutritional norms under Integrated Child Development Services (ICDS) and Mid-
Day Meal (MDM) schemes.
• The nutritional and feeding norms for supplementary nutrition as prescribed by
GOI are 500 calories and 12-15 grams of protein of children between the ages of
6 months to 6 years, 600 (not 1600) calories and 18-20 grams of protein to
pregnant and nursing mothers and 800 calories and 20-25 gm. of protein to
malnourished children.
• Statement 2: Eldest woman of the household of age 18 years or above to be the
head of the household for the purpose of issuing of ration cards.
Refer: https://www.insightsonindia.com/2022/07/04/panel-calls-for-boosting-protein-
and-nutrients-in-government-meal-programmes/
23. Consider the following statements:
1. Economic Advisory Council to the Prime Minister (EAC-PM) is an independent body
constituted to give advice on economic and related issues to the Government of
India, specifically to the Prime Minister.
2. The Cabinet Secretary is the ex-officio Chairman of the Economic Advisory Council
to the Prime Minister (EAC-PM).
Which of the statements given above is/are correct?
(a) 1 only
(b) 2 only
(c) Both 1 and 2
(d) Neither 1 nor 2
Ans: (a)
Explanation:
• Economic Advisory Council to the Prime Minister (EAC-PM) is an independent
body constituted to give advice on economic and related issues to the
Government of India, specifically to the Prime Minister.
• At present, the composition of EAC-PM is: Dr. Bibek Debroy (Chairman),
Shri Rakesh Mohan (Part-Time Member), Dr. Sajjid Chinoy (Part-Time Member),
Dr. Neelkanth Mishra (Part-Time Member), Shri Nilesh Shah (Part-Time
Member), Prof. T.T. Ram Mohan (Part-Time Member) and Dr. Poonam Gupta
(Part-Time Member).
• The Terms of Reference of EAC-PM include analyzing any issue, economic or
otherwise, referred to it by the Prime Minister and advising him thereon,
addressing issues of macroeconomic importance and presenting views thereon to
the Prime Minister. These could be either suo-motu or on reference from the
Prime Minister or anyone else. They also include attending to any other task as
may be desired by the Prime Minister from time to time.
Refer: https://www.insightsonindia.com/2022/07/04/indias-patent-law-safeguards-
under-fire/
24. Consider the following statements about National Investigation Agency (NIA):
1. It is the primary counter-terrorist task force of India.
2. It is empowered to deal with the investigation of terror related crimes across states
without special permission from the states.
3. The founding Director-General of NIA was T. N. Seshan.
Which of the statements given above is/are correct?
(a) 1 and 2 only
(b) 2 and 3 only

Telegram: https://t.me/insightsIAStips
14
Youtube: https://www.youtube.com/channel/UCpoccbCX9GEIwaiIe4HLjwA
Revision Through MCQs (RTM) Compilation (July - 2022)

(c) 1 and 3 only


(d) 1, 2 and 3
Ans: (a)
Explanation:
• S3: The founding Director-General of NIA was Radha Vinod Raju, and he served
until 31 January 2010.
• S1 and S2: The National Investigation Agency (NIA) is the primary counter-
terrorist task force of India. The agency is empowered to deal with the
investigation of terror related crimes across states without special permission
from the states under written proclamation from the Ministry of Home Affairs.
Refer: https://www.insightsonindia.com/2022/07/04/the-functioning-of-the-national-
investigation-agency/
25. Regarding Foreign Contribution (Regulation) Act, which of the following statements
is/are true?
1. FCRA is implemented by the Ministry of Home Affairs.
2. Once granted, FCRA registration is valid for three years.
3. The entities covered by the Act does not include an individual.
Select the correct answer using the code below:
(a) 1 only
(b) 2 and 3 only
(c) 1 and 2 only
(d) 1, 2 and 3
ANs: (a)
Explanation:
• FCRAregulates foreign donations and ensures that such contributions do not
adversely affect the internal security of the country.
o The Act, first enacted in 1976 was amended in the year 2010 and then
2020.
o Section 5 of the Foreign Contribution (Regulation) Act, 2010 gives the
Union government “unchecked and unbridled powers” to declare an
organisation as being one of political nature and deny it access to funds
from sources abroad.
• FCRA is implemented by the Ministry of Home Affairs.
• Applicability:
o The provisions of the Act apply to the territory of India, to citizens of
India who may be outside India and to companies or their branches
outside India that are registered or incorporated in India.
o The entities covered by the Act include an individual, a Hindu undivided
family, an association, or a registered company.
• For how long is approval granted?
o Once granted, FCRA registration is valid for five years. NGOs are
expected to apply for renewal within six months of the date of expiry of
registration. In case of failure to apply for renewal, the registration is
deemed to have expired, and the NGO is no longer entitled to receive
foreign funds or utilise its existing funds without permission from the
ministry.
Refer: https://www.insightsonindia.com/2022/07/04/indians-can-get-%e2%82%b910-
lakh-from-relatives-abroad/

Telegram: https://t.me/insightsIAStips
15
Youtube: https://www.youtube.com/channel/UCpoccbCX9GEIwaiIe4HLjwA
Revision Through MCQs (RTM) Compilation (July - 2022)

26. With reference to UN (United Nations) Ocean Conference 2022, consider the following
statements:
1. The conference was co-hosted by the Governments of Norway and Sweden.
2. Minister of Earth Sciences led the Indian delegation at UN Ocean Conference.
3. The conference ended with Lisbon Declaration titled ‘Our Ocean, Our Future: call for
action’.
Which of the statements given above is/are corrcet?
(a) 1 only
(b) 2 and 3 only
(c) 1 and 2 only
(d) 1, 2 and 3
ANs: (b)
Explanation:
• The conference was co-hosted by the Governments of Kenya and Portugal.
• Minister of Earth Sciences led the Indian delegation at UN Ocean Conference.
• 2nd UNOC ended with Lisbon Declaration titled ‘Our Ocean, Our Future: call for
action’.
Refer: https://www.insightsonindia.com/2022/07/04/un-ocean-conference-unoc-lisbon-
declaration/
27. Consider the following statements about the ‘GloLitter Partnerships Project’:
1. It is launched by the Intergovernmental Oceanographic Commission (IOC) and the
Food and Agriculture Organization (FAO) and initial funding from the World Bank
Group.
2. It is aimed to prevent and reduce marine plastic litter from shipping and fisheries.
Which of the statements given above is/are correct?
(a) 1 only
(b) 2 only
(c) Both 1 and 2
(d) Neither 1 nor 2
Ans: (b)
Explanation:
• It is launched by theInternational Maritime Organization (IMO) and the FAO
and initial funding from the Government of Norway. It is aimed to prevent and
reduce marine plastic litter from shipping and fisheries.
Refer: https://www.insightsonindia.com/2022/07/04/un-ocean-conference-unoc-lisbon-
declaration/
28. Consider the following statements:
1. Agricultural soils release nitrogen oxides into environment.
2. Cattle release ammonia into environment.
3. Poultry industry releases reactive nitrogen compounds into environment.
Which of the statements given above is/are correct?
(a) 1 and 3 only
(b) 2 and 3 only
(c) 2 only
(d) 1, 2 and 3
Ans: (d)
Explanation:

Telegram: https://t.me/insightsIAStips
16
Youtube: https://www.youtube.com/channel/UCpoccbCX9GEIwaiIe4HLjwA
Revision Through MCQs (RTM) Compilation (July - 2022)

• Agricultural soils contributed to over 70% of N2O emissions from India in 2010,
followed by waste water (12%) and residential and commercial activities (6%).
Since 2002, N2O has replaced methane as the second largest Greenhouse Gas
(GHG) from Indian agriculture.
• Cattle account for 80% of the ammonia production, though their annual growth
rate is 1%, due to a stable population.
• The poultry industry, on the other hand, with an annual growth rate of 6%,
recorded an excretion of reactive nitrogen compounds of 0.415 tonnes in 2016.
Refer: https://www.insightsonindia.com/2022/07/04/new-pathway-to-regulate-nitrate-
absorption-in-plants/
29. Consider the following statements about the PM Formalisation of Micro Food processing
enterprise (PMFME) Scheme:
1. It is a centrally sponsored scheme.
2. It will be implemented over a period of five years from 2020-21 to 2024-25.
Which of the statements given above is/are correct?
(a) 1 only
(b) 2 only
(c) Both1 and 2
(d) Neither 1 nor 2
Ans: (c)
Explanation:
• Centrally sponsored scheme under Ministry of Food processing industries
(MoFPI) under AatamaNirbhar Bharat Abhiyan
• Aim: To empower unorganized Micro Enterprises through the ‘One District One
product’ approach. States to identify food products e.g., perishable Agri-produce,
and cereal-based products.
• Time: 2020-21 to 2024-25
• Convergence: With NULM- will provide Seed capital to the Area level
Federation involved in food processing for giving grants to micro-enterprises in
the area.
• Achievements: Implemented in 35 states, Digital ODOP map developed, over 1
lakh SHG members.
Refer: facts for prelims: https://www.insightsonindia.com/2022/07/04/mission-2023-
insights-daily-current-affairs-pib-summary-04-july-2022/
30. Which of the following statements are correct about the deposits of ‘methane hydrate?
1. Global warming might trigger the release of methane gas from these deposits.
2. Large deposits of ‘methane hydrate’ are found in Arctic Tundra and under the
seafloor.
3. Methane in atmosphere oxidizes to carbon dioxide after a decade or two.
Select the correct answer using the code given below.
(a) 1 and 2 only
(b) 2 and 3 only
(c) 1 and 3 only
(d) 1, 2 and 3
Ans: (d)
Explanation:

Telegram: https://t.me/insightsIAStips
17
Youtube: https://www.youtube.com/channel/UCpoccbCX9GEIwaiIe4HLjwA
Revision Through MCQs (RTM) Compilation (July - 2022)

• S1 and S2: Large but poorly known amounts of methane are trapped in the
sediments beneath the sea floor (in Tundra region and elsewhere), frozen into a
form of water ice called methane hydrate.
• At low temperatures the methane hydrates on the sea floor are stable, but if the
water and the sea floor become warmer, then the hydrates can break down.
Because microorganisms then oxidize the resulting methane gas to form the
greenhouse gas carbon dioxide (CO2), methane hydrates have recently become a
topic of intense discussion within the context of climate change.
See https://worldoceanreview.com/en/wor-1/energy/methane-hydrates/
• S3: Methane is relatively short-lived in the atmosphere; a molecule of methane is
oxidized to water and carbon dioxide within a decade or so, mainly by reaction
with another trace gas, the hydroxyl radical OH-. Thus, unlike the case of
carbon dioxide (which stays in the atmosphere longer than methane), a
concerted effort to reduce methane emissions would have almost immediate
results in terms of reduction of greenhouse effect.
See http://earthguide.ucsd.edu/virtualmuseum/climatechange1/03_3.shtml
Refer: UPSC CSE 2019

RTM- REVISION THROUGH MCQS –5th-Jul-2022


31. Consider the following statements regarding POSHAN Abhiyaan.
1. It was launched by the government on the occasion of the International Women’s
Day on 8th March, 2018.
2. NITI Aayog is the monitoring authority for POSHAN Abhiyaan.
3. The target of the missionis to eliminate stunting among children in the age group 0-
6 years by 2022.
Which of the above statements is/are correct?
(a) 1 and 3 only
(b) 1 and 2 only
(c) 2 and 3 only
(d) 1, 2 and 3
Ans: (b)
Explanation: About Poshan Abhiyaan:
• The programme seeks to improve nutritional outcomes for children,
pregnant women and lactating mothers.
• Launched in 2018 with specific targets to be achieved by 2022.
• It aims to reduce:
o Stunting and wasting by 2% a year (total 6% until 2022) among children.
o Anaemia by 3% a year (total 9%) among children, adolescent girls and
pregnant women and lactating mothers.
o The target of the mission is to bring down stunting among children in the
age group 0-6 years from 38.4% to 25% by 2022.
• NITI Aayog has played a critical role in shaping the POSHAN Abhiyaan. NITI
Aayog has been entrusted with the task of closely monitoring the POSHAN
Abhiyaan and undertaking periodic evaluations.
Refer: https://www.insightsonindia.com/2022/07/05/india-needs-to-scale-up-direct-
nutrition-interventions/

Telegram: https://t.me/insightsIAStips
18
Youtube: https://www.youtube.com/channel/UCpoccbCX9GEIwaiIe4HLjwA
Revision Through MCQs (RTM) Compilation (July - 2022)

32. Consider the following statements about the Mid-Day meal scheme:
1. The scheme guarantees one meal to all children in government and aided schools
and madarsas supported under Samagra Shiksha.
2. The Scheme comes under the Ministry of Consumer Affairs, Food and Public
Distribution.
Which of the statements given above is/are correct?
(a) 1 only
(b) 2 only
(c) Both 1 and 2
(d) Neither 1 nor 2
Ans: (a)
Explanation: About the Mid-Day meal scheme:
• The scheme guarantees one meal to all children in government and aided
schools and madarsas supported under Samagra Shiksha.
• Students up to Class VIII are guaranteed one nutritional cooked meal at least
200 days in a year.
• The Scheme comes under the Ministry of HRD.
• It was launched in the year 1995 as the National Programme of Nutritional
Support toPrimary Education (NP – NSPE), a centrally sponsored scheme. In
2004, the scheme was relaunched as the Mid Day Meal Scheme.
• The Scheme is also covered by the National Food Security Act, 2013
Refer: https://www.insightsonindia.com/2022/07/05/india-needs-to-scale-up-direct-
nutrition-interventions/
33. As per the Plastic Waste Management Amendment Rules, 2021 in India, which one of
the following statements is correct?
(a) The thickness of plastic carry bags has been increased from twenty microns to fifty
microns.
(b) Waste generator has to segregate waste into five categories.
(c) It is mandatory on the part of waste generator that the waste generated in one
district cannot be moved to another district.
(d) The waste management infrastructure in the States/UTs is being strengthened
through the Swachh Bharat Mission.
Ans: (d)
Explanation:
• Thickness of plastic carry bags to be increased from 50 microns –
o to 75 microns from 30th September, 2021
o to 120 microns from the 31st December, 2022
• The waste management infrastructure in the States/UTs is being strengthened
through the Swachh Bharat Mission.
• Refer: Government notifies the Plastic Waste Management Amendment
Rules, 2021, prohibiting identified single use plastic items by 2022.
• https://pib.gov.in/PressReleaseIframePage.aspx?PRID=1745433
Refer: https://www.insightsonindia.com/2022/07/05/niti-aayog-report-on-alternatives-to-
plastics/
34. Which of the following are particle detectors at Large Hadron Collider (LHC)?
1. ALICE (A Large Ion Collider Experiment)
2. The Large Hadron Collider beauty (LHCb)
3. The Compact Muon Solenoid (CMS)

Telegram: https://t.me/insightsIAStips
19
Youtube: https://www.youtube.com/channel/UCpoccbCX9GEIwaiIe4HLjwA
Revision Through MCQs (RTM) Compilation (July - 2022)

4. ATLAS
Select the correct answer using the code given below:
(a) 1, 2 and 3 only
(b) 3 and 4 only
(c) 1, 2 and 4 only
(d) 1, 2, 3 and 4
Ans: (d)
Explanation: What is the LHC?
• The Large Hadron Collider (LHC) is the world’s largest and most powerful
particle accelerator. It first started up on 10 September 2008, and remains the
latest addition to CERN’s accelerator complex. The LHC consists of a 27-
kilometre ring of superconducting magnets with a number of accelerating
structures to boost the energy of the particles along the way.
• The beams inside the LHC are made to collide at four locations around the
accelerator ring, corresponding to the positions of four particle detectors –
ATLAS, CMS, ALICE and LHCb.

• ALICE (A Large Ion Collider Experiment) is a detector dedicated to heavy-ion
physics at the Large Hadron Collider (LHC). It is designed to study the physics of
strongly interacting matter at extreme energy densities, where a phase of matter
called quark-gluon plasma forms.
• The Large Hadron Collider beauty (LHCb) experiment specializes in investigating
the slight differences between matter and antimatter by studying a type of
particle called the “beauty quark”, or “b quark”.
• The Compact Muon Solenoid (CMS) is a general-purpose detector at the Large
Hadron Collider (LHC). It has a broad physics programme ranging from studying
the Standard Model (including the Higgs boson) to searching for extra
dimensions and particles that could make up dark matter. Although it has the
same scientific goals as the ATLAS experiment, it uses different technical
solutions and a different magnet-system design.
• ATLAS is one of two general-purpose detectors at the Large Hadron Collider
(LHC). It investigates a wide range of physics, from the search for the Higgs
boson to extra dimensions and particles that could make up dark matter.
Although it has the same scientific goals as the CMS experiment, it uses
different technical solutions and a different magnet-system design.
Refer: https://www.insightsonindia.com/2022/07/05/large-hadron-collider/
35. With reference to the ancient history of India, the terms “Kurinji, Mullai, Marutham,
Neithal and Paalai” refers to
(a) places of exquisite stone art and architecture
(b) landscapes of Sangam Age
(c) ports handling foreign trade
(d) important Buddhist pilgrimage centres
Ans: (b)
Explanation:
• T.N. sends soil samples for a project relating to the new Parliament building
o They pertain to the 5 ecological regions mentioned in ancient Tamil
Sangam literature

Telegram: https://t.me/insightsIAStips
20
Youtube: https://www.youtube.com/channel/UCpoccbCX9GEIwaiIe4HLjwA
Revision Through MCQs (RTM) Compilation (July - 2022)

o Soil samples from five ecological regions mentioned in ancient Tamil


Sangam literature — Kurinji, Mullai, Marutham, Neithal and Paalai —
have been collected and sent to the capital recently.
Refer: facts for prelims: https://www.insightsonindia.com/2022/07/05/mission-2023-
insights-daily-current-affairs-pib-summary-05-july-2022/
36. ‘Donetsk and Luhansk’ are the two areas that together make up the ‘Donbas region’ on
the border of which of the following countries?
(a) China – Magnolia
(b) Ukraine – Russia
(c) Israel – Palestine
(d) Bangladesh – Myanmar
Ans: (b)
Explanation:
• Donetsk and Luhansk are the two areas that together make up the Donbas
region on the Ukraine-Russia border.
• Donbas is a key industrial hub and important from a resource perspective
having the largest coal reserves in Ukraine.
• These two areas broke away from the Ukrainian government’s control back in
2014 and proclaimed themselves independent “people’s republics”. Donbas
region also offers strategic advantages for Russia. By controlling the region,
Russia intends to create a ‘land bridge’ to Crimea, a territory it annexed in
2014.
• Access to the Black Sea and warm water ports in Crimea such as Sevastopol
allows it to access important trade routes throughout the year.
Refer: facts for prelims: https://www.insightsonindia.com/2022/07/05/mission-2023-
insights-daily-current-affairs-pib-summary-05-july-2022/
37. In which one of the following States is Shendurney Wildlife Sanctuary located?
(a) Goa
(b) Karnataka
(c) Andhra Pradesh
(d) Kerala
Ans: (d)
Explanation: Saving Chenkurinji from climate change
• The Shendurney Wildlife Sanctuary derives its name from Gluta travancorica, a
species endemic to the Agasthyamala Biosphere Reserve known as ‘Chenkurinji’
in local parlance.
• Shendurney Wildlife Sanctuary is a protected area in the Western Ghats,
India, located in Kollam district of Kerala and comes under the control of
Agasthyamalai Biosphere Reserve.
Refer: facts for prelims: https://www.insightsonindia.com/2022/07/05/mission-2023-
insights-daily-current-affairs-pib-summary-05-july-2022/
38. Consider the following statements:
1. The definition of “Critical Wildlife Habitat is incorporated in the Forest Rights Act,
2006.
2. For the first time in India, Baigas have been given Habitat Rights.
3. Union Ministry of Environment, Forest and Climate Change officially decides and
declares Habitat Rights for Primitive and Vulnerable Tribal Groups in any part of
India.

Telegram: https://t.me/insightsIAStips
21
Youtube: https://www.youtube.com/channel/UCpoccbCX9GEIwaiIe4HLjwA
Revision Through MCQs (RTM) Compilation (July - 2022)

Which of the statements given above is/are correct?


(a) 1 and 2 only
(b) 1, 2 and 2 only
(c) 3 only
(d) 1, 2 and 3
Ans: (a)
Explanation:
• The phrase ‘critical wildlife habitat’ is defined only in the Scheduled Tribes and
Other Traditional Forest Dwellers (Recognition of Forest Rights) Act, 2006, and
NOT in the Wildlife (Protection) Act, 1972.
• Union Ministry of Tribal Affairs officially decides and declares Habitat Rights for
Primitive and Vulnerable Tribal Groups in any part of India.
• Source: http://pib.nic.in/newsite/PrintRelease.aspx?relid=69806
• Source: http://www.downtoearth.org.in/news/baiga-tribals-become-india-s-
first-community-to-get-habitat-rights52452
Refer: UPSC CSE 2018
39. Regarding Money Bill, which of the following statements is not correct?
(a) A bill shall be deemed to be money bill if it contains only provisions related to
imposition, abolition, remission, alteration or regulation of any tax.
(b) A Money Bill has provisions for the custody of the Consolidated Fund of India or the
Contingency Fund of India.
(c) A Money Bill is concerned with appropriation of moneys out of the Contingency
Fund of India.
(d) A Money Bill deals with the regulation of borrowing of money or giving of any
guarantee by the government of India.
Ans: (c)
Explanation: A110. Definition of Money Bill
• (1) For the purposes of this Chapter, a Bill shall be deemed to be a Money Bill if
it contains only provisions dealing with all or any of the following matters,
namely
• (a) the imposition, abolition, remission, alteration or regulation of any tax;
• (b) the regulation of the borrowing of money or the giving of any guarantee by the
Government of India, or the amendment of the law with respect to any financial
obligations undertaken or to be undertaken by the Government of India;
• (c) the custody of the consolidated Fund or the Contingency Fund of India, the
payment of moneys into or the withdrawal of moneys from any such Fund;
• (d) the appropriation of moneys out of the consolidated Fund of India (not
Contingency Fund of India). So, C is incorrect.
Refer: UPSC CSE 2018
40. Consider the following statements:
1. Speaker of the legislative assembly shall vacate his/her office of he/she ceases to be
a member of the assembly
2. Whenever the legislative assembly is dissolved the speaker shall vacate his/her
office immediately
Which of the statements given above is/are correct?
(a) 1 only
(b) 2 only
(c) Both 1 and 2
(d) Neither 1 not 2

Telegram: https://t.me/insightsIAStips
22
Youtube: https://www.youtube.com/channel/UCpoccbCX9GEIwaiIe4HLjwA
Revision Through MCQs (RTM) Compilation (July - 2022)

Ans: (a)
Explanation:
• Usually, the speaker remains in office during the life of the assembly. However,
he vacates his office earlier in any of the following cases:
o If he ceases to be a member of the assembly
o If he resigns by writing to the deputy speaker and;
o If he is removed by a resolution passed by a majority of all the then
members of the assembly. Such a resolution can be moved only after
giving 14 days advance notice
• Statement 2 is incorrect because the Speaker holds office from the date of her
election till immediately before the first meeting of the Legislative assembly after
the dissolution of the one to which she was elected. She is eligible for re-election.
On the dissolution of the Legislative assembly, although the Speaker ceases to
be a member of the House, she does not vacate her office.
Refer: UPSC CSE 2018

RTM- REVISION THROUGH MCQS –6th-Jul-2022


41. Consider the following statements:
1. Tuberculosis (TB) is caused by a bacteria.
2. TB can spread from person to person through the air.
3. TB is a treatable and curable disease.
Which of the statements given above is/are correct?
(a) 1 only
(b) 1 and 2 only
(c) 2 and 3 only
(d) 1, 2 and 3
Ans: (d)
Explanation:
• Tuberculosis (TB) is caused by bacteria (Mycobacterium tuberculosis) that most
often affect the lungs. Tuberculosis is curable and preventable.
• TB is spread from person to person through the air. When people with lung TB
cough, sneeze or spit, they propel the TB germs into the air. A person needs to
inhale only a few of these germs to become infected.
Refer: https://www.insightsonindia.com/2022/07/06/the-way-to-control-tuberculosis/
42. It is also known as Hansen’s disease; the disease can affect the nerves, skin, eyes, and
lining of the nose (nasal mucosa). If left untreated, the nerve damage can result in
paralysis of hands and feet.
The above description refers to which one of the following diseases?
(a) Diabetes
(b) Ebola
(c) Tuberculosis (TB)
(d) Leprosy
Ans: (d)
Explanation:

Telegram: https://t.me/insightsIAStips
23
Youtube: https://www.youtube.com/channel/UCpoccbCX9GEIwaiIe4HLjwA
Revision Through MCQs (RTM) Compilation (July - 2022)

• Hansen’s disease (also known as leprosy) is an infection caused by bacteria


called Mycobacterium leprae. These bacteria grow very slowly and it may take
up to 20 years to develop signs of the infection.
• The disease can affect the nerves, skin, eyes, and lining of the nose (nasal
mucosa). The bacteria attack the nerves, which can become swollen under the
skin. This can cause the affected areas to lose the ability to sense touch and
pain, which can lead to injuries, like cuts and burns. Usually, the affected skin
changes color and either becomes:
o lighter or darker, often dry or flaky, with loss of feeling, or
o reddish due to inflammation of the skin.
Refer: https://www.insightsonindia.com/2022/07/06/the-way-to-control-tuberculosis/
43. Regarding Parliamentary Committees, which of the following statements is/are correct?
1. Committees which are appointed or elected by the House or nominated by the
Speaker
2. Committees which works under the direction of the Speaker and presents its report
to the House or to the Speaker and the Secretariat
3. Committees which are established by the Prime Minister as per the exigencies of the
time and needs of the situation
Select the correct answer using the code below:
(a) 1 only
(b) 1 and 2 only
(c) 2 and 3 only
(d) 1, 2 and 3
Ans: (b)
Explanation:
• Parliamentary Committees:
o The Lok Sabha website describes a parliamentary committee as a
“committee which is appointed or elected by the House or nominated by
the Speaker and which works under the direction of the Speaker and
presents its report to the House or to the Speaker and the Secretariat”.
• Cabinet Committees:
o The Prime Minister sets up different cabinet committees with selected
members of the Cabinet and assigns specific functions to these
committees.
Refer: https://www.insightsonindia.com/2022/07/06/panel-suggests-increasing-tenure-
of-house-committees/
44. Consider the following statements:
1. There is no mention of the Collegium system either in the original Constitution of
India or in successive amendments.
2. High Court judges are recommended by a Collegium system comprising the CJI and
four senior-most judges.
Which of the given above statements is/are correct?
(a) 1 only
(b) 2 only
(c) Both 1 and 2
(d) Neither 1 nor 2
Ans: (a)
Explanation:

Telegram: https://t.me/insightsIAStips
24
Youtube: https://www.youtube.com/channel/UCpoccbCX9GEIwaiIe4HLjwA
Revision Through MCQs (RTM) Compilation (July - 2022)

• S1: The Collegium of judges is the Supreme Court’s invention. It does not figure
in the Constitution, which says judges of the Supreme Court and High Courts
are appointed by the President and speaks of a process of consultation.
• S2: High Court judges are recommended by a Collegium comprising the CJI and
two senior-most judges.
o The proposal, however, is initiated by the Chief Justice of the High Court
concerned in consultation with two senior-most colleagues. The
recommendation is sent to the Chief Minister, who advises the Governor
to send the proposal to the Union Law Minister.
Refer: https://www.insightsonindia.com/2022/07/06/fewer-judges-increasing-pendency-
hope-for-movement-by-govt-to-clear-names-cji/
45. The terms ‘Param Siddhi, Param Ganga, Pratyush’ sometimes mentioned news recently
are related to
(a) Student Satellites
(b) Glacial lakes
(c) Loan category under MUDRA scheme
(d) Super-Computers
Ans: (d)
Explanation:
• As per the global ranking service of super-computers (Top 500), India has only
3 top-ranked supercomputers among 500 and none in the top 100.
• (Param Siddhi (5.27 PFlops), Param Ganga (1.66 PFlops)
and Pratyush supercomputer (Indian Institute of Tropical Meteorology’s))
Refer: https://www.insightsonindia.com/2022/07/06/indias-supercomputing-capabilities/
46. With reference to an elementary particle or fundamental particle, consider the following
statements:
1. Fundamental particles called quarks come in six different flavors.
2. Protons are made of two down quarks and one up quark, while neutrons contain
two up quarks and one down quark.
Which of the statements given above is/are correct?
(a) 1 only
(b) 2 only
(c) Both 1 and 2
(d) Neither 1 nor 2
Ans: (a)
Explanation:
• Elementary particles are the smallest known building blocks of the universe.
They are thought to have no internal structure, meaning that researchers think
about them as zero-dimensional points that take up no space. Electrons are
probably the most familiar elementary particles.
• Fundamental particles called quarks come in six different flavors. Protons
are made of two up quarks and one down quark, while neutrons contain two
down quarks and one up quark.

Telegram: https://t.me/insightsIAStips
25
Youtube: https://www.youtube.com/channel/UCpoccbCX9GEIwaiIe4HLjwA
Revision Through MCQs (RTM) Compilation (July - 2022)


Refer: https://www.insightsonindia.com/2022/07/06/pentaquarks/
47. Consider the following statements about Dr Rajendra Prasad:
1. He was the first president of independent India.
2. He served as the Food and Agriculture Minister in the central government after the
1946 elections.
3. He served as the President of the Constituent Assembly of India.
Which of the statements given above is/are correct?
(a) 1 and 2 only
(b) 2 and 3 only
(c) 1 and 3 only
(d) 1, 2 and 3
Ans: (d)
Explanation:
• Awards in public administration in the field of academic excellence, in memory
of the first President of India.
• Dr Rajendra Prasad was the first president of independent India. He was an
Indian Freedom Activist, Lawyer, and Scholar too. He served as the Food and
Agriculture Minister in the central government after the 1946 elections. He also
served as the President of the Constituent Assembly of India.
• LITERARY WORKS
o Satyagraha at Champaran (1922)
o India Divided (1946)
o Atmakatha (1946) his autobiography written during his 3-year prison
term in Bankipur Jail
o Mahatma Gandhi and Bihar, Some Reminiscences (1949)
o Bapu Ke Kadmon Mein (1954)
o Since Independence (1960)
Refer: Facts For Prelims: https://www.insightsonindia.com/2022/07/06/mission-2023-
insights-daily-current-affairs-pib-summary-06-july-2022/
48. Consider the following statements:
1. The Fields Medal is awarded by the International Mathematical Union (IMU), an
international non-governmental and non-profit scientific organisation.
2. The Fields medals are awarded every four years to the most promising
mathematicians under the age of 40.
3. A person cannot receive more than one IMU Award.

Telegram: https://t.me/insightsIAStips
26
Youtube: https://www.youtube.com/channel/UCpoccbCX9GEIwaiIe4HLjwA
Revision Through MCQs (RTM) Compilation (July - 2022)

Which of the statements given above is/are correct?


(a) 1 and 2 only
(b) 2 and 3 only
(c) 1 and 3 only
(d) 1, 2 and 3
Ans: (d)
Explanation:
• The International Mathematical Union (IMU) recognised Ukrainian
mathematician Maryna Viazovska’s work on the sphere-packing problem in 8
and 24 dimensions.

• The Fields Medal is awarded by the IMU, an international non-governmental and
non-profit scientific organisation.
• The Fields Medal is awarded every four years to one or more mathematicians
under the age of 40 in recognition of “outstanding mathematical achievement for
existing work and for the promise of future achievement.
• A person can not receive more than one IMU Award. A person can receive the
Leelavati Prize in addition to one other IMU Award.
Refer: Facts For Prelims: https://www.insightsonindia.com/2022/07/06/mission-2023-
insights-daily-current-affairs-pib-summary-06-july-2022/
49. Consider the following statements regarding Purchasing Manager’s Index (PMI):
1. It is compiled and released by Confederation of Indian Industry (CII).
2. A PMI reading above 50 represents an expansion in business activity when
compared to the previous year.
Which of the given above statements is/are correct?
(a) 1 only
(b) 2 only
(c) Both 1 and 2
(d) Neither 1 nor 2
Ans: (d)
Explanation:
• PMI or a Purchasing Managers’ Index (PMI) is an indicator of business activity
— both in the manufacturing and services sectors.
• S1: Purchasing Manager’s Index (PMI) is released and compiled by the Institute
for Supply Management (ISM), USA
o The PMI is usually released at the start of the month, much before most
of the official data on industrial output, manufacturing and GDP growth
becomes available
o It is calculated separately for the manufacturing and services sectors and
then a composite index is constructed.
• S2: The headline PMI is a number from 0 to 100. A PMI above 50 represents an
expansion when compared to the previous month. A PMI reading under 50
represents a contraction, and a reading at 50 indicates no change.
Refer: Facts For Prelims: https://www.insightsonindia.com/2022/07/06/mission-2023-
insights-daily-current-affairs-pib-summary-06-july-2022/
50. Anthrax is a serious infectious disease, which is caused by
(a) Virus
(b) Bacteria

Telegram: https://t.me/insightsIAStips
27
Youtube: https://www.youtube.com/channel/UCpoccbCX9GEIwaiIe4HLjwA
Revision Through MCQs (RTM) Compilation (July - 2022)

(c) Fungi
(d) Protozoa
Ans: (b)
Explanation:
• An outbreak of Anthrax has been declared in Athirappilly of Thrissur district,
after finding several carcasses of wild boar.
• Anthrax is a serious infectious disease, which is caused by spore-forming
bacteria.
• Anthrax is also called woolsorter’s disease or malignant pustule. It is a rare
but serious disease, caused due to rod-shaped bacteria called Bacillus
anthracis. These bacteria occur naturally in soil.
As per WHO, Anthrax is a disease of herbivores, affecting wild as well as
domestic animals. It is a zoonotic disease; thus, it is transmissible from animals
to humans.
Refer: Facts For Prelims: https://www.insightsonindia.com/2022/07/06/mission-2023-
insights-daily-current-affairs-pib-summary-06-july-2022/

RTM- REVISION THROUGH MCQS –7th-Jul-2022

51. The famous Shreya Singhal case was in news recently, is related to which of the
following?
(a) Capital punishment
(b) Disqualification of legislators
(c) Model code of conduct
(d) None of the above
Ans: (d)
Explanation:
• What is Section 66A?
o Section 66A defines the punishment for sending “offensive” messages
through a computer or any other communication device like a mobile
phone or a tablet.
o A conviction can fetch a maximum of three years in jail and a fine.
o It empowered police to make arrests over what policemen, in terms of
their subjective discretion, could construe as “offensive” or “menacing” or
for the purposes of causing annoyance, inconvenience, etc.
• Shreya Singhal case:
o The Supreme Court had in its judgment in the Shreya Singhal case
struck down Section 66A.
Refer: https://www.insightsonindia.com/2022/07/07/it-act-sec-69a-when-social-media-
content-is-blocked/
52. Which of the following is/are the aims of “Digital India” Plan of the Government of
India?
1. Formation of India’s own Internet companies like China did.
2. Establish a policy framework to encourage overseas multinational corporations that
collect Big Data to build their large data centres within our national geographical
boundaries.

Telegram: https://t.me/insightsIAStips
28
Youtube: https://www.youtube.com/channel/UCpoccbCX9GEIwaiIe4HLjwA
Revision Through MCQs (RTM) Compilation (July - 2022)

3. Connect many of our villages to the Internet and bring Wi-Fi to many of our schools,
public places and major tourist centres.
Select the correct answer using the code given below:
(a) 1 and 2 only
(b) 3 only
(c) 2 and 3 only
(d) 1, 2 and 3
Ans: (b)
Explanation:’
• Only statement 3 is correct
• The Digital India programme is centred on three key vision areas:
• Digital Infrastructure as a Core Utility to Every Citizen
• Governance and Services on Demand
• Digital Empowerment of Citizens
• Source: https://www.insightsonindia.com/2015/07/02/lok-sabha-tv-insights-
transformation-by-digital-india-a-detailed-article-on-digital-india-program/
Refer: https://www.insightsonindia.com/2022/07/07/governance-digitization-for-an-
inclusive-society/
53. In India, the population Census is a
(a) Union subject
(b) State subject
(c) Concurrent subject
(d) Reserved subject
Ans: (a)
Explanation:
• The authority to conduct Census in India is derived from Article 246 of the
Constitution of India. This article empowers the Parliament to make laws with
respect to any of the matters enumerated in the List-I in the Seventh Schedule
referred to as 'Union List" and the subject 'Census'.
Refer: https://www.insightsonindia.com/2022/07/07/caste-census-constitution-and-
power-of-states-to-hold-such-enumeration-of-the-population/
54. Consider the following statements regarding National Commission for Backward
Classes (NCBC).
1. National Commission for Backward Classes is a non-constitutional body under the
Ministry of Social Justice and Empowerment.
2. The commission considers inclusions in and exclusions from the lists of
communities notified as backward for the purpose of job reservations.
3. The commission have the same powers as a Civil Court.
Which of the above statements is/are correct?
(a) 1 and 2 only
(b) 1 and 3 only
(c) 2 and 3 only
(d) 1, 2 and 3
Ans: (c)
Explanation:
• National Commission for Backward Classes is a constitutional body (123rd
constitutional amendment bill 2017 and 102nd amendment 2018 in constitution
to make it constitutional body) (Article 338B of the Indian Constitution) under

Telegram: https://t.me/insightsIAStips
29
Youtube: https://www.youtube.com/channel/UCpoccbCX9GEIwaiIe4HLjwA
Revision Through MCQs (RTM) Compilation (July - 2022)

Ministry of Social Justice and Empowerment established on 14 August 1993. It


was constituted pursuant to the provisions of the National Commission for
Backward Classes Act, 1993.
• The commission was the outcome of Indra Sawhney & Ors. Vs. Union of India.
• The commission considers inclusions in and exclusions from the lists of
communities notified as backward for the purpose of job reservations and
tenders the needful advice to the Central Government as per Section 9(1) of the
NCBC Act, 1993. Similarly, the states have also constituted commissions for
BC's. The National Commission for Backward Classes, National Commission for
Scheduled Castes as well as National Commission for Scheduled Tribes have the
same powers as a Civil Court.
Refer: https://www.insightsonindia.com/2022/07/07/caste-census-constitution-and-
power-of-states-to-hold-such-enumeration-of-the-population/
55. Which of the following adopted a new law, known as Markets in Crypto-Assets (MiCA),
to regulate cryptocurrencies?
(a) China
(b) Sweden
(c) The European Union
(d) The United States of America
Ans: (c)
Explanation:
• Recently, EU Parliament agreed upon a new law Markets in Crypto-Assets
(MiCA) to regulate cryptocurrencies.
• Recent issues: Stablecoins came in question after the crash of Terraform Labs’
Luna token.
• About MiCA:
o It seeks to address concerns like money-laundering, protection of
consumers and investors, accountability of crypto firms, stablecoins and
the environmental footprint of crypto mining.
o It excludes NFT (non-fungible tokens)
Refer: https://www.insightsonindia.com/2022/07/07/the-markets-in-crypto-assets-mica/
56. ‘Global Findex database’ released by which of the following?
(a) International Monetary Fund
(b) World Economic Forum
(c) Organisation for Economic Co-operation and Development
(d) World Bank
Ans: (d)
Explanation:
• World Bank’s Global Findex database surveyed how people in 123 economies
use formal and informal financial services e.g., cards, ATMs, mobile phones, and
the internet.
Refer: https://www.insightsonindia.com/2022/07/07/world-bank-releases-global-findex-
database-2021/
57. Consider the following pairs:
Toys/Handicrafts State
1. Etikoppaka bommalu Andhra Pradesh
2. Choppu saman Odisha
3. Ghuggu and Handwai Punjab

Telegram: https://t.me/insightsIAStips
30
Youtube: https://www.youtube.com/channel/UCpoccbCX9GEIwaiIe4HLjwA
Revision Through MCQs (RTM) Compilation (July - 2022)

Which of the given pairs is/are correctly matched?


(a) 1 only
(b) 1 and 3 only
(c) 2 and 3 only
(d) 1 and 2 only
Ans: (b)
Explanation:
• Etikoppaka bommalu (Andhra):
Traditional shapes of the toys include a farmer with nagali, a bridal
set, veena, sannai melam, a spread of traditional wedding sweets and more.
• Choppu saman (TN): These wooden toy sets of miniature kitchen utensils come
in different sizes and shapes, polished at the edges and painted bright.
• Chankana, Ghuggu and Handwai (Punjab)
Refer: Facts For Prelims: https://www.insightsonindia.com/2022/07/07/mission-2023-
insights-daily-current-affairs-pib-summary-07-july-2022/

58. With reference to India, the terms ‘Gorias, Moriyas and Jolhas’ pertain to
(a) dance forms of Northwest India
(b) musical instruments
(c) pre-historic cave paintings
(d) indigenous communities
Ans: (d)
Explanation:
• The Assam government approved the indigenous status of five Assamese Muslim
sub-groups- Gorias, Moriyas, Jolhas, Deshis and Syeds sub-groups as
indigenous Assamese Muslim communities.
Refer: Facts For Prelims: https://www.insightsonindia.com/2022/07/07/mission-2023-
insights-daily-current-affairs-pib-summary-07-july-2022/
59. Consider the following statements:
1. Wolbachia are natural bacteria present in up to 60% of insect species, including
some mosquitoes.
2. The Wolbachia inhibits the multiplication of the chikungunya virus when present in
the mosquitoes.
Which of the statements given above is/are correct?
(a) 1 only
(b) 2 only
(c) Both 1 and 2
(d) Neither 1 nor 2
Ans: (c)
Explanation:
• Wolbachia bacteria blocks the proliferation of harmful viruses but doesn’t harm
the mosquitoes themselves
• Wolbachia is a bug which is found in 60% of insects naturally. But in the Aedes
mosquito, this bug is in very low frequencies.
• Wolbachia inhibits dengue and chikungunya virus replication in mosquitoes
Refer: Facts for Prelims: https://www.insightsonindia.com/2022/07/07/mission-2023-
insights-daily-current-affairs-pib-summary-07-july-2022/

Telegram: https://t.me/insightsIAStips
31
Youtube: https://www.youtube.com/channel/UCpoccbCX9GEIwaiIe4HLjwA
Revision Through MCQs (RTM) Compilation (July - 2022)

60. What is Cas9 protein that is often mentioned in news?


(a) A molecular scissors used in targeted gene editing
(b) A biosensor used in the accurate detection of pathogens in patients
(c) A gene that makes plants pest-resistant
(d) A herbicidal substance synthesized in genetically modified crops
Ans: (a)
Explanation:
• Cas9 (CRISPR associated protein 9) is a protein which plays a vital role in the
immunological defense of certain bacteria against DNA viruses, and which is
heavily utilized in genetic engineering applications. Its main function is to cut
DNA and therefore it can alter a cell’s genome.
Refer: UPSC CSE 2019

RTM- REVISION THROUGH MCQS –8th-Jul-2022

61. Consider the following statements:


1. Ozone is made up of three oxygen atoms.
2. Ozone is both a natural and a man-made product.
3. The Vienna Convention is the first international agreement dedicated to the
protection of the ozone layer.
Which of the statements given above is/are correct?
(a) 1 and 2 only
(b) 1 and 3 only
(c) 2 and 3 only
(d) 1, 2 and 3
Ans: (d)
Explanation:
• Ozone (O3) is a highly reactive gas composed of three oxygen atoms. It is
both a natural and a man-made product that occurs in the Earth’s upper
atmosphere (the stratosphere) and lower atmosphere (the troposphere).
• Vienna convention: The Vienna Convention is the first international agreement
dedicated to the protection of the ozone layer. The Convention commits all
countries to take measures to protect human health and the environment
resulting from modifications to the ozone layer.
• Montreal Protocol: The Montreal Protocol on Substances that Deplete the
Ozone Layer was designed to reduce the production and consumption of ozone-
depleting substances in order to reduce their abundance in the atmosphere and
thereby protect the earth’s fragile ozone Layer.
Refer: https://www.insightsonindia.com/2022/07/08/large-year-round-ozone-hole-over-
tropics/
62. Consider the following statements about “Montreal Protocol”:
1. It is the landmark multilateral environmental agreement that commits state parties
to reduce greenhouse gas emissions, based on the scientific consensus.
2. It led to the replacement of Hydrofluorocarbons (HFCs) with Chlorofluorocarbons
(CFCs) which do not destroy the Ozone layer.
Which of the given above statements is/are correct?
(a) 1 only

Telegram: https://t.me/insightsIAStips
32
Youtube: https://www.youtube.com/channel/UCpoccbCX9GEIwaiIe4HLjwA
Revision Through MCQs (RTM) Compilation (July - 2022)

(b) 2 only
(c) Both 1 and 2
(d) Neither 1 nor 2
Ans: (d)
Explanation:
• S1: The Montreal Protocol on Substances that Deplete the Ozone Layer is
the landmark multilateral environmental agreement that regulates the
production and consumption of nearly 100 man-made chemicals referred to
as ozone depleting substances (ODS). When released to the atmosphere, those
chemicals damage the stratospheric ozone layer, Earth’s protective shield that
protects humans and the environment from harmful levels of ultraviolet
radiation from the sun. Adopted on 15 September 1987, the Protocol is to date
the only UN treaty ever that has been ratified every country on Earth – all 198
UN Member States.
• S2: The Montreal Protocol led to the replacement of CFCs with
Hydrofluorocarbons (HFCs) which do not destroy the Ozone layer.
o Hydrochlorofluorocarbons (HCFCs) are gases used worldwide in
refrigeration, air-conditioning and foam applications, but they are being
phased out under the Montreal Protocol since deplete the ozone layer.
HCFCs are both ODS and powerful greenhouse gases: the most
commonly used HCFC is nearly 2,000 times more potent than carbon
dioxide in terms of its global warming potential (GWP).
o Hydrofluorocarbons (HFCs), were introduced as non-ozone depleting
alternatives to support the timely phase out of CFCs and HCFCs. HFCs
are now widespread in air conditioners, refrigerators, aerosols, foams and
other products. While these chemicals do not deplete the
stratospheric ozone layer, some of them have high GWPs ranging from
12 to 14,000.
Refer: https://www.insightsonindia.com/2022/07/08/large-year-round-ozone-hole-over-
tropics/
63. In which one of the following groups are all the four countries members of G20?
(a) Argentina, Mexico, South Africa and Turkey
(b) Australia, Canada, Malaysia and New Zealand
(c) Brazil, Iran, Saudi Arabia and Vietnam
(d) Indonesia, Japan, Singapore and South Korea
Ans: (a)
Explanation:
• The members of the G20 are: Argentina, Australia, Brazil, Canada, China,
France, Germany, India, Indonesia, Italy, Japan, Republic of Korea, Mexico,
Russia, Saudi Arabia, South Africa, Turkey, the United Kingdom, the United
States, and the European Union.
Refer: https://www.insightsonindia.com/2022/07/08/in-bali-meeting-with-wang-
jaishankar-raises-lac-issues/
64. With reference to Development Monitoring and Evaluation Office (DMEO), consider the
following statements:
1. It is an attached office of National Statistical Office (NSO).
2. It supports the Government achieve the national development agenda through
monitoring and evaluation (M&E) of government policies and programs.
Which of the statements given above is/are correct?

Telegram: https://t.me/insightsIAStips
33
Youtube: https://www.youtube.com/channel/UCpoccbCX9GEIwaiIe4HLjwA
Revision Through MCQs (RTM) Compilation (July - 2022)

(a) 1 only
(b) 2 only
(c) Both 1 and 2
(d) Neither 1 nor 2
Ans: (b)
Explanation:
• The Development Monitoring and Evaluation Office (DMEO) is an attached
office of NITI Aayog. As the apex monitoring and evaluation (M&E) office in the
country, DMEO supports the Government achieve the national development
agenda through M&E of government policies and programs.
Refer: https://www.insightsonindia.com/2022/07/08/gender-budgeting-act/
65. The State of Food Security and Nutrition in the World is an annual flagship report
jointly prepared which of the following?
1. Food and Agriculture Organization
2. International Fund for Agriculture Development
3. World Resources Institute
4. UN world food Programme
5. World Health Organization
Select the correct answer using the code below:
(a) 1, 2, 3 and 4 only
(b) 2, 3, 4 and 5 only
(c) 1, 3, 4 and 5 only
(d) 1, 2, 4 and 5 only
Ans: (d)
Explanation:
• The State of Food Security and Nutrition in the World is an annual flagship
report jointly prepared by Food and Agriculture Organization of the United
Nations (FAO), the International Fund for Agricultural Development (IFAD),
the United Nations Children's Fund (UNICEF), the World Food Programme
(WFP) and the World Health Organization (WHO).


Refer: https://www.insightsonindia.com/2022/07/08/the-state-of-food-security-and-
nutrition-in-the-world-sofi-2022/
66. Kharchi Puja is a Hindu festival celebrated in the Indian state of
(a) Maharashtra
(b) Odisha

Telegram: https://t.me/insightsIAStips
34
Youtube: https://www.youtube.com/channel/UCpoccbCX9GEIwaiIe4HLjwA
Revision Through MCQs (RTM) Compilation (July - 2022)

(c) Manipur
(d) West Bengal
Ans: (c)
Explanation:
• Kharchi Puja is a Hindu festival from Tripura, the festival involves
the worship of the fourteen gods forming the dynasty deity of the Tripuri
people.
Refer: facts for prelims: https://www.insightsonindia.com/2022/07/08/mission-2023-
insights-daily-current-affairs-pib-summary-08-july-2022/
67. Consider the following statements:
1. Sandfish is naturally found in India only.
2. Asiatic lion is naturally found in India only.
3. One-horned rhinoceros is naturally found in India only.
Which of the statements given above is / are correct?
(a) 1 and 2 only
(b) 2 only
(c) 1 and 3 only
(d) 1, 2 and 3
Ans: (b)
Explanation:
• S1: Sandfish are migratory freshwater fish found only in South Africa that
can grow to over half a metre in length. They help to keep the rivers clean and
the food web balanced.
• S2: The Asiatic lion’s range is restricted to the Gir National Park and environs in
the Indian state of Gujarat.
• S3: The one-horned rhinoceros is native to the Indian subcontinent (not only
India). The Indian rhinoceros once ranged throughout the entire stretch of the
Indo-Gangetic Plain, but excessive hunting and agricultural development
reduced their range drastically to 11 sites in northern India and southern Nepal.
Refer: facts for prelims: https://www.insightsonindia.com/2022/07/08/mission-2023-
insights-daily-current-affairs-pib-summary-08-july-2022/
68. Consider the following statements regarding PM SVANidhi scheme.
1. The PM SVANidhi scheme, is a credit facility that provides street vendors a
collateral-free loan of Rs 1 lakh with low rates of interest for a period of one year.
2. The scheme is part of the AtmaNirbhar Bharat package.
3. Small Industries Development Bank of India (SIDBI) is the implementing agency for
the scheme.
Which of the above statements is/are correct?
(a) 1 and 2
(b) 1 and 3
(c) 2 and 3
(d) 2 only
Ans: (c)
Explanation:
• Ministry of Housing & Urban Affairs launched a scheme PM Street Vendor's
AtmaNirbhar Nidhi (PM SVANidhi) to empower Street Vendors by not only
extending loans to them, but also for their holistic development and economic
upliftment.

Telegram: https://t.me/insightsIAStips
35
Youtube: https://www.youtube.com/channel/UCpoccbCX9GEIwaiIe4HLjwA
Revision Through MCQs (RTM) Compilation (July - 2022)

• The scheme intends to facilitate collateral free working capital loans of up to


INR10, 000/- of one-year tenure, to approximately 50 lakh street vendors, to
help resume their businesses in the urban areas, including surrounding peri-
urban/rural areas. Small Industries Development Bank of India (SIDBI) is
the implementing agency.
Refer: facts for prelims: https://www.insightsonindia.com/2022/07/08/mission-2023-
insights-daily-current-affairs-pib-summary-08-july-2022/
69. 'Chicago Convention' related to
(a) Disaster mitigation
(b) International Civil Aviation
(c) Crypto currency
(d) Population control
Ans: (b)
Explanation:
• In response to SpiceJet’s increasing number of air safety problems, the Directorate General of
Civil Aviation (DGCA) issued a show-cause notice
• Convention on International Civil Aviation (also known as Chicago
Convention), was signed on 7 December 1944 by 52 States, India is also part of
this.


Refer: facts for prelims: https://www.insightsonindia.com/2022/07/08/mission-2023-
insights-daily-current-affairs-pib-summary-08-july-2022/

70. In the context of which of the following do some scientists suggest the use of cirrus
cloud thinning technique and the injection of sulphate aerosol into stratosphere?
(a) Creating the artificial rains in some regions
(b) Reducing the frequency and intensity of tropical cyclones
(c) Reducing the adverse effects of solar wind on the Earth
(d) Reducing the global warming
Ans: (d)

Telegram: https://t.me/insightsIAStips
36
Youtube: https://www.youtube.com/channel/UCpoccbCX9GEIwaiIe4HLjwA
Revision Through MCQs (RTM) Compilation (July - 2022)

Explanation:
• The ability of stratospheric sulfate aerosols to create a global dimming effect has
made them a possible candidate for use in solar radiation management climate
engineering projects to limit the effect and impact of climate change due to rising
levels of greenhouse gases. Delivery of precursor sulfide gases such as sulfuric
acid, hydrogen sulfide (H2S) or sulfur dioxide (SO2) by artillery, aircraft and
balloons has been proposed.
• Cirrus cloud thinning is a proposed form of climate engineering. Cirrus clouds
are high cold ice that, like other clouds, both reflect sunlight and absorb
warming infrared radiation. However, they differ from other types of clouds in
that, on average, infrared absorption outweighs sunlight reflection, resulting in a
net warming effect on the climate. Therefore, thinning or removing these clouds
would reduce their heat trapping capacity, resulting in a cooling effect on Earth’s
climate.
Refer: UPSC CSE 2019

RTM- REVISION THROUGH MCQS –9th-Jul-2022

71. With reference to the temples at Khajuraho, consider the following statements:
1. The site of Khajuraho belonged to the Chandela Kings who ruled over Central India
from the 9th to 13th centuries.
2. The monuments at Khajuraho are a group of Hindu and Jaina temples.
3. The Brahma temple is the only temple at Khajuraho completely built of soapstone
and has a four-faced lingam in its sanctum sanctorum.
Which of the statements given above is/are correct?
(a) 1 only
(b) 2 and 3 only
(c) 1 and 2 only
(d) 1, 2 and 3
Ans: (c)
Explanation:
• The site of Khajuraho belonged to the Chandela Kings who ruled over Central
India from the 9th to 13th centuries. Khajuraho was the cultural capital of the
Chandela rulers as a result of which it continued to flourish despite the frequent
shifting of its political capital. Initially, this group of monuments were a total of
eighty-four temples out of which only twenty-five remain intact today. The
monuments at Khajuraho are a group of Hindu and Jaina temples. Widely
known for its magnificent structures and the much-debated erotic sculptures,
Khajuraho was declared a World Heritage Site by UNESCO in the year 1986.
• Khajuraho temples are considered as some of the best examples of Nagara style
of temple architecture with a sanctum cella (garbhagriha), small vestibule
(antarala), a large hall (mahamandapa), an assembly hall (mandapa) and an
entrance portico (ardhamandapa).
• The temples are grouped into three categories; the Western Group, the Eastern
Group and the Southern Group.
• Western Group:
o It is the western group that is known worldwide with the Kandariya
Mahadev Temple, Lakshmana Temple, Visvanatha Temple, Chausath

Telegram: https://t.me/insightsIAStips
37
Youtube: https://www.youtube.com/channel/UCpoccbCX9GEIwaiIe4HLjwA
Revision Through MCQs (RTM) Compilation (July - 2022)

Yogini and Chitragupta Temple being a part of it. They are considered to
be some of the best representations of a typical Khajuraho temple. Out of
these, the Kandariya Mahadev (102 feet in length and 66 feet in breadth)
temple has a lingam enshrined in its sanctum sanctorum. Known for its
intricately carved interiors depicting Gods and Goddesses, the exteriors
of this temple have erotic sculptures carved on them.
• Eastern Group:
o This group is dominated by the Jaina Temples out of which the
Parsvanath Temple is the largest. Initially, the temple was dedicated to
Adinath which was later replaced with the idol of Parsvanath. Close to
the Parsvanath temple is the Ghantai Temple and the Adinath temple.
The Adinath temple is one of the later temples as a result of which the
sculptures of this temple are more detailed and delicately carved.
o This group also has the Vamana, Brahma and Javeri temples of the
Hindu pantheon. The Brahma temple is the only temple at Khajuraho
completely built of granite and has a four-faced lingam in its
sanctum sanctorum.
• Southern Group:
o The Southern Group of temples is situated slightly away from other
groups and consists of the Duladeo Temple, Chaturbhuj Temple and the
Beejamandal Temple. The Duladeo temple dedicated to Lord Shiva has
some of the best sculptures depicting apsaras and is the most famous
temple among the Southern Group of temples.
Refer: https://www.insightsonindia.com/2022/07/09/india-gets-elected-to-the-
intergovernmental-committee-of-unescos-2003-convention-for-the-safeguarding-of-the-
intangible-cultural-heritage/
72. Consider the following statements about the Sun Temple, Konark:
1. It is one of the best representations of Kalinga Architecture.
2. It was built by the King of the Ganga Dynasty, Narasimha Deva I in the 13th
century.
3. It is made of a stone called granite, widely available in the region.
Which of the statements given above is/are correct?
(a) 1 only
(b) 1 and 2 only
(c) 2 and 3 only
(d) 1, 2 and 3
Ans: (b)
Explanation:
• Situated close to the Bay of Bengal, the Sun Temple Konark is one of the best
representations of Kalinga Architecture. The word ‘Konark’ can be translated to
the ‘corner sun’; Kona (corner) and Arka (sun) and was so named as it lay on the
north east corner of the famous Jagannath Temple at Puri. Konark was also
called the Black Pagoda, the first mention of which is found in the diary of
Sir Streynsham Master, Governor of Fort St George, Madras. It was used as a
major landmark by most sailors and travellers. Dedicated to Sun God, Konark
was built by the King of the Ganga Dynasty, Narasimha Deva I in the 13th
century. It was declared a Unesco World Heritage Site in the year 1984 for its
architectural brilliance and for serving as an evidence of the spread of Sun
worship in India from Kashmir to the eastern parts.

Telegram: https://t.me/insightsIAStips
38
Youtube: https://www.youtube.com/channel/UCpoccbCX9GEIwaiIe4HLjwA
Revision Through MCQs (RTM) Compilation (July - 2022)

• Konark is made of a stone called Khondalite, widely available in the region.


The entire structure was constructed without the use of any binding material
such as mortar or cement. It was also one of the first structures in India to have
used iron beams and dowels. It is said that the temple was held together using
electromagnetic force with sixty three tons of magnet, ten tons at the bottom and
fifty three at the top of the structure.
Refer: https://www.insightsonindia.com/2022/07/09/india-gets-elected-to-the-
intergovernmental-committee-of-unescos-2003-convention-for-the-safeguarding-of-the-
intangible-cultural-heritage/
73. Consider the following statements:
1. The Great Living Chola Temples were built by kings of the Chola Empire, which
stretched over all of south India and the neighbouring islands.
2. The Temple of Gangaikondacholisvaram, built by Rajendra I, was completed in
1035.
3. The Airavatesvara temple complex, built by Rajaraja II, at Darasuram features a 24-
m vimana and a stone image of Vishnu.
Which of the statements given above is/are correct?
(a) 1 and 3 only
(b) 2 only
(c) 1 and 2 only
(d) 1, 2 and 3
Ans: (c)
Explanation:
• The Great Living Chola Temples were built by kings of the Chola Empire,
which stretched over all of south India and the neighbouring islands.
• The site includes three great 11th- and 12th-century Temples: the Brihadisvara
Temple at Thanjavur, the Brihadisvara Temple at Gangaikondacholisvaram and
the Airavatesvara Temple at Darasuram.
• The Temple of Gangaikondacholisvaram, built by Rajendra I, was completed in
1035. Its 53-m vimana (sanctum tower) has recessed corners and a graceful
upward curving movement, contrasting with the straight and severe tower at
Thanjavur.
• The Airavatesvara temple complex, built by Rajaraja II, at Darasuram
features a 24-m vimana and a stone image of Shiva.
• The temples testify to the brilliant achievements of the Chola in architecture,
sculpture, painting and bronze casting.
Refer: https://www.insightsonindia.com/2022/07/09/india-gets-elected-to-the-
intergovernmental-committee-of-unescos-2003-convention-for-the-safeguarding-of-the-
intangible-cultural-heritage/
74. Which one of the following heritage site is known especially for its rathas (temples in
the form of chariots), mandapas (cave sanctuaries), and giant open-air reliefs such as
the famous 'Descent of the Ganges'?
(a) Group of Monuments at Hampi
(b) Group of Monuments at Pattadakal
(c) Group of Monuments at Mahabalipuram
(d) Khajuraho Group of Monuments
Ans: (c)
Explanation: Group of Monuments at Mahabalipuram

Telegram: https://t.me/insightsIAStips
39
Youtube: https://www.youtube.com/channel/UCpoccbCX9GEIwaiIe4HLjwA
Revision Through MCQs (RTM) Compilation (July - 2022)

• Mahabalipuram
o Fifty six kilometre south of Chennai stands Mahabalipuram (traditionally
known as Mamallapuram) the ancient seaport of the Pallava kingdom
(7th - 8th centuries)
o Mahabalipuram,which was declared a UNESCO World Heritage Site in
1984 is a complex of rock cut temples, monolithic structures, rathas
(chariot temples), mandapas (caves) and numerous Siva sculptures.
• Cave temples
o The Cave Temples were built before the reign of the Pallava king
Mahendravarman I and are a complex of various temples that denote the
oldest forms of Pallava architecture.
o These consist of the Adi Paraha Perumal Cave Temple (dedicated to Lord
Vishnu), the Trimurti Cave Temple (dedicated to Brahma, Vishnu and
Siva), the Krishna Caves, Mahishasuramardini Cave (dedicated to
Goddess Durga) and the Yali or Tiger Caves (Narasimhavarman II or
Rajasimha - The Royal Seat).
• Pancha Ratha
o The Pancha Rathas or the five chariots is essentially an architectural
eclogue to the five Pandavas and their wife Draupadi from the
Mahabharata. Each ratha (Draupadi ratha, Arjuna ratha, Bhim ratha,
Dharmaraja ratha, Nakul-Sahadev ratha) structurally signifies the traits
of each of the Pandavas which makes them different from one another.
Carved from a single slab of rock, the walls of these rathas are adorned
with bas reliefs and murals such as elephants and Nandi the bull.
• Descent of the Ganges/Arjuna’s Penance
o The Descent of the Ganges also known as Arjuna’s penance is the second
largest ancient monolithic structure in Asia. The inscription depicts the
mythical story of Arjuna from the Mahabharata and scenes from
everyday South Indian life.
o The centre represents the nagas (snakes) descending from a once water-
filled cleft, representing the Ganges. The left has Arjuna performing self-
mortification (standing on one leg), in order to procure Pasupatastra, the
most powerful weapon from Lord Siva.
• Shore Temple
o The two towered Shore Temple at the very coast of Coromandel is one of
the most significant representations of Pallava architecture. It consists of
the seven pagodas that were built between 700 and 728 CE during the
reign of Narasimhavarman II and is dedicated to Lord Siva. The entrance
of the temple is marked by gopurams and the shikhara or the roof of the
temple resembles a pyramidal structure, which is one of a kind. A
complex of seven temples, there are a few along with other civil
structures that presently lie under the sea.
• Olakkanneshvara Temple
o The Olakkanneshvara Temple or the Olakkanatha Temple is a temple
dedicated to Lord Siva located on the top of a hill.
Refer: https://www.insightsonindia.com/2022/07/09/india-gets-elected-to-the-
intergovernmental-committee-of-unescos-2003-convention-for-the-safeguarding-of-the-
intangible-cultural-heritage/

Telegram: https://t.me/insightsIAStips
40
Youtube: https://www.youtube.com/channel/UCpoccbCX9GEIwaiIe4HLjwA
Revision Through MCQs (RTM) Compilation (July - 2022)

75. With reference to Ellora Caves, consider the following statements:


1. The Ellora caves are located in the district of Aurangabad in Maharashtra.
2. The caves were declared as a UNESCO world heritage site in 1983.
3. The topography of the area consists of a rocky plateau in the shape of a semicircle.
Which of the statements given above is/are correct?
(a) 1 only
(b) 2 and 3 only
(c) 1 and 3 only
(d) 1, 2 and 3
Ans: (d)
Explanation:
• The Ellora caves are located in the district of Aurangabad in Maharashtra. The
caves were excavated between the 6th to 11th centuries CE. There are more than
100 caves out of which only 34 are open to the public. Out of the 34, the first 12
are Buddhist, caves, 13 to 29 are Hindu caves, and 30 to 34 are Jain caves. The
caves were declared as a UNESCO world heritage site in 1983. The topography of
the area consists of a rocky plateau in the shape of a semicircle. The Buddhist
caves lie at the right arc on the south, the Jain caves lie at the left arc on the
north, while the Hindu caves are at the centre.
• The rocks are made of multi-layered basalt formations, known as the Deccan
Traps, the finer grains of which lend themselves to detailed sculpting. Due to
their close proximity to a trade route, the Ellora caves, unlike the ones at Ajanta,
were never lost to oblivion. There are written records which prove that these
caves were visited regularly by travellers and royal patronage continued well
after the last constructions took place.
• The constructions at Ellora, also locally known as Verul, were done under the
patronage of kings and wealthy merchants and traders. Although inscriptional
evidence is quite scanty, yet it is known that the Rashtrakuta dynasty got a part
of the Hindu and the Buddhist caves constructed, while the Yadava dynasty
constructed the Jain caves.
Refer: https://www.insightsonindia.com/2022/07/09/india-gets-elected-to-the-
intergovernmental-committee-of-unescos-2003-convention-for-the-safeguarding-of-the-
intangible-cultural-heritage/
76. With reference to Rani-ki-Vav (the Queen’s Stepwell), consider the following statements:
1. It was built on the banks of the river Saraswati, by the Gurjar Rani Udayamati of
the Chalukya Dynasty.
2. It was built to commemorate her husband Raja Bhimdev I.
3. It was declared as a World Heritage Site in 2014 by UNESCO.
Which of the statements given above is/are correct?
(a) 3 only
(b) 1 and 2 only
(c) 1, 2 and 3
(d) None of the above
Ans: (c)
Explanation:
• Rani ki Vav or precisely translated as the Queen’s stepwell was built in 1063,
on the banks of the river Saraswati, by the Gurjar Rani Udayamati of the
Chalukya Dynasty. The stepwell was built to commemorate her husband

Telegram: https://t.me/insightsIAStips
41
Youtube: https://www.youtube.com/channel/UCpoccbCX9GEIwaiIe4HLjwA
Revision Through MCQs (RTM) Compilation (July - 2022)

Raja Bhimdev I. This remarkably beautiful stepwell, located in Patan is the


oldest and most intricately created form of subterranean water architecture and
was declared as a World Heritage Site in 2014 by UNESCO.
• Initially created as a memorial for Raja Bhimdev I, the stepwell was also
constructed with a functional and structural aesthetic. Classified as a Nanda
style stepwell, the steps lead down through multiple levels, with queues of
carved walls, pillars, columns, brackets and beams ornamented with scroll work
and more than 800 sculptures, which mostly depict the ten different avatars of
Lord Vishnu (Dasaavatara).
Refer: https://www.insightsonindia.com/2022/07/09/india-gets-elected-to-the-
intergovernmental-committee-of-unescos-2003-convention-for-the-safeguarding-of-the-
intangible-cultural-heritage/
77. With reference to the International Union for Conservation of Nature and Natural
Resources (IUCN) and the Convention on International Trade in Endangered Species of
Wild Fauna and Flora (CITES), which of the following statements is/are correct?
1. IUCN is an organ of the United Nations and CITES is an international agreement
between governments
2. IUCN runs thousands of field projects around the world to better manage natural
environments.
3. CITES is legally binding on the States that have joined it, but this Convention does
not take the place of national laws.
Select the correct answer using the code given below.
(a) 1 only
(b) 2 and 3 only
(c) 1 and 3 only
(d) 1, 2 and 3
Ans: (b)
Explanation:
• IUCN is a voluntary organization, not an agency of United Nations. Even if you
knew this much, you could eliminate all other options to select B. Some facts
about IUCN:
• Founded in 1948 as the world’s first global environmental organization
o Today the largest professional global conservation network
o A leading authority on the environment and sustainable development
o More than 1,200 member organizations including 200+ government and
900+ non-government organizations
o A neutral forum for governments, NGOs, scientists, business and local
communities to find practical solutions to conservation and development
challenges
o Thousands of field projects and activities around the world
• CITES was drafted as a result of a resolution adopted in 1963 at a meeting of
members of the International Union for Conservation of Nature (IUCN). States
(countries) adhere voluntarily to the agreement. States that have agreed to be
bound by the Convention (‘joined’ CITES) are known as Parties.
• Although CITES is legally binding on the Parties – in other words they have to
implement the Convention – it does not take the place of national laws. Rather it
provides a framework to be respected by each Party, which has to adopt its own
domestic legislation to ensure that CITES is implemented at the national level.
• Sources: https://www.iucn.org/about/

Telegram: https://t.me/insightsIAStips
42
Youtube: https://www.youtube.com/channel/UCpoccbCX9GEIwaiIe4HLjwA
Revision Through MCQs (RTM) Compilation (July - 2022)

• https://cites.org/eng/disc/what.php
Refer: https://www.insightsonindia.com/2022/07/09/forest-landscape-restoration/
78. Open Acreage Licensing, often seen in news, is related to
(a) Hydrocarbon exploration
(b) Railways
(c) Mining of Iron ore
(d) Land Acquisition
Ans: (a)
Explanation:
• The OALP, a critical part of the Hydrocarbon Exploration and Licensing Policy,
provides uniform licences for exploration and production of all forms of
hydrocarbons, enabling contractors to explore conventional as well as
unconventional oil and gas resources. Fields are offered under a revenue-sharing
model and throw up marketing and pricing freedom for crude oil and natural gas
produced.
Refer: facts for prelims: https://www.insightsonindia.com/2022/07/09/mission-2023-
insights-daily-current-affairs-pib-summary-09-july-2022/
79. Global Liveability Index is released by which of the following?
(a) Economist Intelligence Unit
(b) Amnesty international
(c) World Economic Forum
(d) United Nations Development Programme
Ans: (a)
Explanation:
• Global Liveability Index 2022
• Findings:
o Best liveable city in the world: Vienna
o Best liveable city in India: Delhi
o Least: Bengaluru
• Released by: Economist Intelligence Unit (EIU)
• Theme: Recovery and Hardship
Refer: facts for prelims: https://www.insightsonindia.com/2022/07/09/mission-2023-
insights-daily-current-affairs-pib-summary-09-july-2022/
80. With reference to ‘Li-Fi’, recently in the news, which of the following statements is/are
correct?
1. It uses light as the medium for high-speed data transmission.
2. It is a wireless technology and is several times faster than ‘WiFi’.
Select the correct answer using the code given below.
(a) 1 only
(b) 2 only
(c) Both 1 and 2
(d) Neither 1 nor 2
Ans: (c)
Explanation:
• Statement 1: Li-fi is loosely expanded to Light-Fidelity. Using precisely controlled
light modulation, data can be transmitted using regular, visible light.
• Statement 2: It will be 10-100 times faster than current-generation WiFi, and
also very secure.

Telegram: https://t.me/insightsIAStips
43
Youtube: https://www.youtube.com/channel/UCpoccbCX9GEIwaiIe4HLjwA
Revision Through MCQs (RTM) Compilation (July - 2022)

Refer: UPSC CSE 2016

RTM- REVISION THROUGH MCQS –11th-Jul-2022

81. Consider the following statements :


1. Sustainable Development Goals (SDGs) were born at the United Nations Conference
on Sustainable Development in Rio de Janeiro in 2012.
2. The Sustainable Development Goals (SDGs) were adopted by all United Nations
Member States in 2015.
Which of the statements given above is/are correct?
(a) 1 only
(b) 2 only
(c) Both 1 and 2
(d) Neither 1 nor 2
Ans: (c)
Explanation:
• Sustainable Development Goals (SDGs) were born at the United Nations
Conference on Sustainable Development in Rio de Janeiro in 2012.
• The Sustainable Development Goals (SDGs) were adopted by all United
Nations Member States in 2015 to end poverty, reduce inequality and build
more peaceful, prosperous societies by 2030
Refer: https://www.insightsonindia.com/2022/07/11/232639/
82. Consider the following statements regarding Intergovernmental Science-Policy Platform
on Biodiversity and Ecosystem Services (IPBES).
1. Intergovernmental Science-Policy Platform on Biodiversity and Ecosystem Services
(IPBES) is a United Nations body established for the conservation and sustainable
use of biodiversity.
2. The United Nations Environment Programme (UNEP) provides secretariat services to
IPBES.
3. It is intended to serve a similar role to the Intergovernmental Panel on Climate
Change (IPCC).
Which of the above statements is/are correct?
(a) 1 and 2 only
(b) 1 and 3 only
(c) 2 and 3 only
(d) 1, 2 and 3
Ans: (c)
Explanation:
• The Intergovernmental Science-Policy Platform on Biodiversity and Ecosystem
Services (IPBES) is an independent intergovernmental body established by
States to strengthen the science-policy interface for biodiversity and ecosystem
services for the conservation and sustainable use of biodiversity, long-term
human well-being and sustainable development. It was established in Panama
City, on 21 April 2012 by 94 Governments. It is not a United Nations body.
However, at the request of the IPBES Plenary and with the authorization of the
UNEP Governing Council in 2013, the United Nations Environment Programme
(UNEP) provides secretariat services to IPBES.

Telegram: https://t.me/insightsIAStips
44
Youtube: https://www.youtube.com/channel/UCpoccbCX9GEIwaiIe4HLjwA
Revision Through MCQs (RTM) Compilation (July - 2022)

• The governing body of IPBES – made up of the representatives of IPBES


member States – usually meets once per year. Observers: Any State not yet a
member of IPBES; the Convention on Biological Diversity (CBD) and other
biodiversity-related conventions; related UN bodies; as well as many other
relevant organizations and agencies. It is intended to serve a similar role to
the Intergovernmental Panel on Climate Change.
Refer: https://www.insightsonindia.com/2022/07/11/ipbes-assessment-report-on-wild-
species/
83. Which one of the following best describes the term ‘Digital Nomads’?
(a) freelancers who typically do short-term work for multiple clients
(b) Independent contractor who earns wages on a per-job or per-task basis, typically for
short-term work.
(c) Individual who works for someone else in exchange for compensation.
(d) Remote workers who travel to different locations on a regular basis.
Ans: (d)
Explanation:
• Indonesia has announced “Digital Nomad Visas” for travellers, to attract more
foreign tourists.
• Digital Nomads are people who work remotely while travelling to different places
and spending their earned income in the country they are travelling to.
• Unlike regular remote workers, who tend to stay in one geographic area, digital
nomads travel and explore while working.
Refer: facts for prelims: https://www.insightsonindia.com/2022/07/11/mission-2023-
insights-daily-current-affairs-pib-summary-11-july-2022/
84. Consider the following statements about the Monkeypox disease:
1. It is a viral zoonotic disease with symptoms similar to those seen in the past in
smallpox patients.
2. It is usually a self-limited disease with the symptoms lasting from 2 to 4 weeks.
Which of the statements given above is/are correct?
(a) 1 only
(b) 2 only
(c) Both 1 and 2
(d) Neither 1 nor 2
Ans: (c)
Explanation:
• Monkeypox is a viral zoonosis (a virus transmitted to humans from animals) with
symptoms similar to those seen in the past in smallpox patients, although it is
clinically less severe.
• Monkeypox is usually a self-limited disease with the symptoms lasting from 2 to
4 weeks. Severe cases can occur. In recent times, the case fatality ratio has been
around 3–6%.
Refer: facts for prelims: https://www.insightsonindia.com/2022/07/11/mission-2023-
insights-daily-current-affairs-pib-summary-11-july-2022/
85. Consider the following statements:
1. India is the second largest tobacco producer behind Brazil.
2. The Tobacco Board is a statutory body.
Which of the statements given above is/are correct?
(a) 1 only

Telegram: https://t.me/insightsIAStips
45
Youtube: https://www.youtube.com/channel/UCpoccbCX9GEIwaiIe4HLjwA
Revision Through MCQs (RTM) Compilation (July - 2022)

(b) 2 only
(c) Both 1 and 2
(d) Neither 1 nor 2
Ans: (b)
Explanation:
• Andhra Pradesh government exported tobacco to the USA for 1 st time in the
country
• Status: India is the 2nd largest producer (1st is China) and 2nd largest export
(1st in Brazil) of tobacco.
• Nodal Agency: Tobacco Board (Ministry of Commerce and Industry)
o The Tobacco Board was constituted as a statutory body on 1st January,
1976 under Section (4) of the Tobacco Board Act, 1975.
Refer: facts for prelims: https://www.insightsonindia.com/2022/07/11/mission-2023-
insights-daily-current-affairs-pib-summary-11-july-2022/
86. Consider the following statements:
1. The muon is one of the fundamental subatomic particles.
2. Muons are similar to proton but weigh more than 207 times as much.
3. Muons are by-products of cosmic rays colliding with molecules in the upper
atmosphere.
Which of the above statements is/are correct?
(a) 1 and 2 only
(b) 1 and 3 only
(c) 2 and 3 only
(d) 1, 2 and 3
Ans: (b)
Explanation:
• The muon is one of the fundamental subatomic particles, the most basic
building blocks of the universe as described in the Standard Model of particle
physics.
• Muons are similar to electrons but weigh more than 207 times as much.
• Muons occur naturally when cosmic rays strike Earth's atmosphere, and
particle accelerators at Fermilab can produce them in large numbers.
• Muons are by-products of cosmic rays colliding with molecules in the upper
atmosphere. Muons reach earth with an average velocity of about 0.994c.
Refer: facts for prelims: https://www.insightsonindia.com/2022/07/11/mission-2023-
insights-daily-current-affairs-pib-summary-11-july-2022/
87. In which one of the following States is Singalila National Park located?
(a) Arunachal Pradesh
(b) Sikkim
(c) West Bengal
(d) Odisha
Ans: (c)
Explanation:
• Singalila National Park is a National park of India located on the Singalila
Ridge at an altitude of more than 7000 feet above sea level, in the Darjeeling
district of West Bengal.
Refer: facts for prelims: https://www.insightsonindia.com/2022/07/11/mission-2023-
insights-daily-current-affairs-pib-summary-11-july-2022/

Telegram: https://t.me/insightsIAStips
46
Youtube: https://www.youtube.com/channel/UCpoccbCX9GEIwaiIe4HLjwA
Revision Through MCQs (RTM) Compilation (July - 2022)

88. Consider the following statements about Red Panda:


1. It is the state animal of Sikkim.
2. It is listed as Endangered in the IUCN red list of Threatened Species.
3. It is threatened by poaching as well as destruction and fragmentation of habitat due
to deforestation.
Which of the above statements is/are correct?
(a) 1 and 2 only
(b) 1 and 3 only
(c) 2 and 3 only
(d) 1, 2 and 3
Ans: (d)
Explanation:
• Category: endangered species (IUCN) and comes under Schedule I of the
Indian Wildlife (Protection) Act, 1972.
• Red pandas are shy, solitary and arboreal animals and are considered
an indicator species for ecological change.
• They are also one of the most iconic species in terms of their importance to
global conservation.
• The red panda’s survival is crucial for the eastern and north-eastern
Himalayan subalpine conifer forests and the eastern Himalayan broadleaf
forests. (2200-4800m under dense bamboo)
• It is also the state animal of Sikkim.
• Protection measure: Transboundary law enforcement cooperation through the
use of multi-government platforms like SAWEN (South Asia Wildlife Enforcement
Network).
Refer: facts for prelims: https://www.insightsonindia.com/2022/07/11/mission-2023-
insights-daily-current-affairs-pib-summary-11-july-2022/
89. In which one of the following States is Neora Valley National Parks located?
(a) Himachal Pradesh
(b) Uttarakhand
(c) Assam
(d) West Bengal
Ans: (d)
Explanation:
• The number of red pandas has been declining in the wild, even in the Singalila
and Neora Valley National Parks (West Bengal)
Refer: facts for prelims: https://www.insightsonindia.com/2022/07/11/mission-2023-
insights-daily-current-affairs-pib-summary-11-july-2022/
90. “The crop is semi-tropical crop in nature. It needs 100-120 days of frost-free climate,
medium rainfall, loamy soil with high potash and iron content. It cannot tolerate high-
speed winds and storms.” Which one of the following is that crop?
(a) Cotton
(b) Sugar cane
(c) Tobacco
(d) Wheat
Ans: (c)
Explanation:

Telegram: https://t.me/insightsIAStips
47
Youtube: https://www.youtube.com/channel/UCpoccbCX9GEIwaiIe4HLjwA
Revision Through MCQs (RTM) Compilation (July - 2022)

• Andhra Pradesh government exported tobacco to the USA for 1st time in the
country
• Status: India is the 2nd largest producer (1st is China) and 2nd largest export
(1st in Brazil) of tobacco.
• Climate: Semi-tropical crop, needs 100-120 days of frost-free climate, medium
rainfall, loamy soil with high potash and iron content. It cannot tolerate high-
speed winds and storms.
• Nodal Agency: Tobacco Board (Ministry of Commerce and Industry)
Refer: facts for prelims: https://www.insightsonindia.com/2022/07/11/mission-2023-
insights-daily-current-affairs-pib-summary-11-july-2022/

RTM- REVISION THROUGH MCQS –12th-Jul-2022

91. The national motto of India, ‘Satyameva Jayate’ inscribed below the Emblem of India is
taken from:
(a) Katha Upanishad
(b) Chandogya Upanishad
(c) Aitareya Upanishad
(d) Mundaka Upanishad
Ans: (d)
Explanation:
• Satyameva Jayate ('Truth alone triumphs') is a part of a mantra from the
Hindu scripture Mundaka Upanishad. Following the independence of India, it
was adopted as the national motto of India on 26 January 1950, the day India
became a republic.
Refer: https://www.insightsonindia.com/2022/07/12/pm-modi-unveils-national-emblem-
on-new-parliament-building/
92. “To abide by the Constitution and respect its ideals and institutions, the National Flag
and the National Anthem ” is a provision made in the
(a) Preamble of the Constitution
(b) Directive Principles of State Policy
(c) Fundamental Rights
(d) Fundamental Duties
Ans: (d)
Explanation:
• Constitutional & Statutory Provisions regarding National Flag of India:
• Art 51A (a) –To abide by the Constitution and respect its ideals and institutions,
the National Flag and the National Anthem.
• Statutes Governing Use of Flag:
o Emblems and Names (Prevention of Improper Use) Act, 1950.
o Prevention of Insults to National Honor Act, 1971.
Refer: https://www.insightsonindia.com/2022/07/12/pm-modi-unveils-national-emblem-
on-new-parliament-building/
93. With reference to ‘World Population Prospects’ (WPP) report, consider the following
statements:
1. The Population Division of the UN has been publishing the WPP in a biennial cycle
since 1971.

Telegram: https://t.me/insightsIAStips
48
Youtube: https://www.youtube.com/channel/UCpoccbCX9GEIwaiIe4HLjwA
Revision Through MCQs (RTM) Compilation (July - 2022)

2. Each revision of the WPP provides a historical time series of population indicators
starting in 1970.
Which of the statements given above is/are correct?
(a) 1only
(b) 2 only
(c) Both 1 and 2
(d) Neither 1 nor 2
Ans: (d)
Explanation: World Population Prospects:
• The Population Division of the UN has been publishing the WPP in a biennial
cycle since 1951.
• Each revision of the WPP provides a historical time series of population
indicators starting in 1950.
• It does so by taking into account newly released national data to revise
estimates of past trends in fertility, mortality or international migration.
Refer: https://www.insightsonindia.com/2022/07/12/india-to-surpass-china-as-the-most-
populous-country-in-a-year-un/
94. Consider the following statements regarding Dark energy.
1. Dark energy is the mysterious form of energy that makes up less than 1% of the
universe.
2. While dark matter attracts and holds galaxies together, dark energy repels and
causes the expansion of our universe.
Which of the statements given above is/are correct?
(a) 1 only
(b) 2 only
(c) Both 1 and 2
(d) Neither 1 nor 2
Ans: (b)
Explanation:
• Dark energy, the mysterious form of energy that makes up about 68% of the
universe, has intrigued physicists and astronomers for decades. Dark energy has
been noted as “the most profound mystery in all of science”. With advanced
technologies and newer experiments, scientists have found certain clues about it
and, recently an international team of researchers made the first putative direct
detection of dark energy. Everything we see – the planets, moons, massive
galaxies, you, me, this website – makes up less than 5% of the universe. About
27% is dark matter and 68% is dark energy. While dark matter attracts and
holds galaxies together, dark energy repels and causes the expansion of our
universe.
Refer: https://www.insightsonindia.com/2022/07/12/dark-matter/
95. Consider the following statements:
1. Jet Streams occur in the Northern Hemisphere only.
2. Only some cyclones develop an eye.
3. The temperature inside the eye of a cyclone is nearly 10 degree Celsius lesser than
that of the surroundings.
Which of the statements given above is/are correct?
(a) 1 only
(b) 2 and 3 only

Telegram: https://t.me/insightsIAStips
49
Youtube: https://www.youtube.com/channel/UCpoccbCX9GEIwaiIe4HLjwA
Revision Through MCQs (RTM) Compilation (July - 2022)

(c) 2 only
(d) 1 and 3 only
Ans: (c)
Explanation:
• S1: This is obviously incorrect, they occur in upper atmospheres of both
hemispheres.
• S2: Correct. Extra-tropical cyclones may not always have an eye, whereas mostly
mature storms have well developed eye. Rapidly intensifying storms may develop
an extremely small, clear, and circular eye, sometimes referred to as a pinhole
eye.
• S3: It is warmer and not colder for a tropical cyclone. The warmer temperature is
what drives the storm.
• Context:
o An extremely large ‘Azores High’ (a subtropical weather
phenomenon) has resulted in abnormally dry conditions across the
western Mediterranean.
o Azores High is a subtropical high-pressure system that extends over the
eastern subtropical North Atlantic and Western Europe during winter. It
is associated with anticyclonic winds in the subtropical North Atlantic. It
is formed by dry air aloft descending the subtropics and coincides with
the downward branch of the Hadley Circulation.
Refer: Facts For Prelims: https://www.insightsonindia.com/2022/07/12/mission-2023-
insights-daily-current-affairs-pib-summary-12-july-2022/
96. Consider the following statements about Acharya Prafulla Chandra Ray:
1. He pioneered the investigation of radio and microwave optics.
2. He is popularly known as the 'father of Indian Chemistry'.
3. Bengal Chemicals & Pharmaceuticals Limited was established in 1901 by Acharya
P.C. Ray.
Which of the statements given above is/are correct?
(a) 1 only
(b) 2 and 3 only
(c) 2 only
(d) 1 and 3 only
Ans: (b)
Explanation:
• PC Ray’s contribution to the field of Science was a step toward nation-building.
He was also a scholar who championed educational reforms, promoted
employment through industry, and rallied for political advancement.
o Contributions:
o Discovered several ground-breaking chemical compounds (eg:
mercurous nitrite), published more than a hundred and fifty research
papers in famous science journals (eg: Journal of the Chemical Society
of London); authored many books on chemical sciences (eg: Life and
Experience of a Bengali Chemist).
• Bengal Chemicals & Pharmaceuticals Limited is a Public Sector Undertaking.
Established in Kolkata, in 1901 by Prafulla Chandra Ray (1861-1944), it is
India's first pharmaceutical company.

Telegram: https://t.me/insightsIAStips
50
Youtube: https://www.youtube.com/channel/UCpoccbCX9GEIwaiIe4HLjwA
Revision Through MCQs (RTM) Compilation (July - 2022)

• In 1892, Prafulla Chandra Ray founded Bengal Chemical Works with a capital of
Rs. 700 (US$10).
Refer: Facts For Prelims: https://www.insightsonindia.com/2022/07/12/mission-2023-
insights-daily-current-affairs-pib-summary-12-july-2022/
97. Recently, which one of the following has inaugurated the world's first-ever facility
dedicated to monitoring space debris and safeguarding assets in space?
(a) SpaceX
(b) NASA
(c) ISRO
(d) JAXA
Ans: (c)
Explanation:
• The Indian Space Research Organisation (ISRO) inaugurated the world's
first-ever facility dedicated to monitoring space debris and safeguarding
assets in space, which is expected to significantly help combat the growing
threat to orbiting satellites and space stations.
Refer: Facts For Prelims: https://www.insightsonindia.com/2022/07/12/mission-2023-
insights-daily-current-affairs-pib-summary-12-july-2022/
98. The identity platform ‘Aadhaar’ provides open “Application Programming Interfaces
(APIs)”. What does it imply?
1. It can be integrated into any electronic device.
2. Online authentication using iris is possible.
Which of the statements given above is/are correct?
(a) 1 only
(b) 2 only
(c) Both 1 and 2
(d) Neither 1 nor 2
Ans: (c)
Explanation:
• Aadhaar provides open Application Programming Interfaces or APIs, which can
be integrated easily into any electronic device.
• These APIs enable online authentication using a fingerprint or iris.
Refer: Facts For Prelims: https://www.insightsonindia.com/2022/07/12/mission-2023-
insights-daily-current-affairs-pib-summary-12-july-2022/
99. Mukundara Hills National Park, recently seen in news is located in
(a) Maharashtra
(b) Madhya Pradesh
(c) Rajasthan
(d) Gujarat
Ans: (c)
Explanation:
• Mukundara Hills National Park is a national park in Rajasthan.
• It was established in 2004 and consists of three wildlife sanctuaries: Darrah
Wildlife Sanctuary, National Chambal Sanctuary, and Jawahar Sagar Wildlife
Sanctuary.
Refer: https://en.wikipedia.org/wiki/Mukundara_Hills_National_Park
100. Consider the following statements regarding Manda buffalo, recently seen in news.
1. It is found in the Eastern Ghats and parts of Odisha.

Telegram: https://t.me/insightsIAStips
51
Youtube: https://www.youtube.com/channel/UCpoccbCX9GEIwaiIe4HLjwA
Revision Through MCQs (RTM) Compilation (July - 2022)

2. They are less prone to diseases and are resistant to parasitic infections.
3. It is the first animal to be recognised by National Bureau of Animal Genetic
Resources (NBAGR).
Which of the above statements is/are correct?
(a) 1 and 2 only
(b) 2 only
(c) 1 and 3 only
(d) 1, 2 and 3
Ans: (a)
Explanation:
• The National Bureau of Animal Genetic Resources (NBAGR) has recognised the
Manda buffalo, found in the Eastern Ghats and plateau of Koraput region of
Odisha, as the 19th unique breed of buffaloes found in India.
• The Manda are resistant to parasitic infections, less prone to diseases and can
live, produce and reproduce at low or nil input system.
• This buffalo germ-plasm was first identified through a detail survey conducted
by the Animal Resource Development (ARD) department of Odisha in
collaboration with Orissa University of Agriculture and Technology (OUAT).
• With Manda buffalo recognised by NBAGR, it will be incumbent upon both
Centre and State governments to devise a strategy for conservation of these
buffaloes.
• Four breeds of cattle — Binjharpuri, Motu, Ghumusari and Khariar — and two
breeds of buffalo — Chilika and Kalahandi — and one breed of sheep,
Kendrapada, have already received NBAGR recognition.


Refer: https://www.thehindu.com/news/national/other-states/koraputs-manda-buffalo-
gets-unique-indigenous-tag/article36319950.ece

RTM- REVISION THROUGH MCQS –13th-Jul-2022

101. “World Population Prospects” is released by which of the following?


(a) World Bank Group

Telegram: https://t.me/insightsIAStips
52
Youtube: https://www.youtube.com/channel/UCpoccbCX9GEIwaiIe4HLjwA
Revision Through MCQs (RTM) Compilation (July - 2022)

(b) UN Department of Economics and Social Affairs


(c) World Economic Forum
(d) United Nations Population Fund
Ans: (b)
Explanation:
• The world Population prospects are released by the UN Department of
Economics and Social Affairs(UNDESA).
Refer: https://www.insightsonindia.com/2022/07/13/indias-demography-at-crossroads-
elderly-may-outnumber-the-youth/
102. With reference to ‘National Institution for Transforming India’ (NITI Aayog), consider
the following statements:
1. It was formed via a resolution of the Union Cabinet on 1 January 2014.
2. It comprises all the state Governors and Chief Ministers, along with the Chief
Ministers of Delhi and Puducherry, Lieutenant Governors of all UTs, and a vice
chairman nominated by the Prime Minister.
Which of the given above statements is/are correct?
(a) 1 only
(b) 2 only
(c) Both 1 and 2
(d) Neither 1 nor 2
Ans: (d)
Explanation:
• S1: The NITI Aayog is a policy think tank of the Government of India, established
in 2015 to replace the Planning Commission.
• S2: It comprises all the state Chief Ministers (not Governors), along with the
Chief Ministers of Delhi and Puducherry, Lieutenant Governors of all UTs, and a
vice-chairman nominated by the Prime Minister.
Refer: https://www.insightsonindia.com/2022/07/13/niti-aayogs-role-in-ensuring-
collaboration-between-states-and-centre/
103. Consider the following statements regarding CRISPR–Cas9.
1. CRISPR technology allows to easily alter DNA sequences and modify gene function.
2. It can be used to correct sickle cell anemia, a genetic blood disorder.
3. Gene editing system does not occur naturally in organisms.
Which of the above statements is/are correct?
(a) 1 and 2 only
(b) 2 and 3 only
(c) 1 only
(d) 1, 2 and 3
Ans: (a)
Explanation:
• CRISPR-Cas9 stands for ‘Clustered regularly interspaced short palindromic
repeats and CRISPR-associated protein 9.’ This protein can be programmed to
go to a desired location in the genome and correct or edit defective strands (such
as those involved in certain diseases) of DNA. The technology, when perfected,
may be used to treat several genetic disorders.
o Indian scientists have developed a new variant of currently popular gene
editing tool, CRISPR-Cas9, and have shown that this variant can

Telegram: https://t.me/insightsIAStips
53
Youtube: https://www.youtube.com/channel/UCpoccbCX9GEIwaiIe4HLjwA
Revision Through MCQs (RTM) Compilation (July - 2022)

increase precision in editing genome while avoiding unintended changes


in DNA.
o The researchers have also shown that this type of gene editing can
be used to correct sickle cell anemia, a genetic blood disorder.
o By reprogramming and using a naturally occurring gene editing
system – CRISPR-Cas9 – found in bacteria, scientists globally have
been engaged in ‘editing’ genome of various organisms.
o Read more>>
Refer: https://www.insightsonindia.com/2022/07/13/10-years-of-crispr-gene-editing/
104. CRISPR-based genetic engineering technology can be used in which of the following
areas
1. Improve the production of biofuels
2. Restrain populations of mosquitoes that infect humans
3. Transcriptional activation and repression
Select the correct answer code:
(a) 1, 2 and 3 only
(b) 2, 3 and 4 only
(c) 1, 2 and 4 only
(d) 1, 2, 3 and 4
Ans: (d)
Explanation:
• Leveraging advancements in CRISPR-based genetic engineering, researchers
have created a system that restrains populations of mosquitoes that infect
millions each year with debilitating diseases.
• Applications of CRISPR
o Using CRISPR for genome editing
o Using CRISPR libraries for screening
o CRISPR/Cas9-mediated chromatin immunoprecipitation
o Transcriptional activation and repression
o Epigenetic editing with CRISPR/Cas9
o Live imaging of DNA/mRNA
o Therapeutic Applications
o Enhancing crop production
o Fighting antimicrobial resistance
• Gene editing could improve the production of biofuels by algae. Using
CRISPR-Cas9, the strains of algae that produce twice as much fat can be
created, which is then used to produce biodiesel.
Refer: https://www.insightsonindia.com/2022/07/13/10-years-of-crispr-gene-editing/
105. Consider the following statements about the James Webb Space Telescope:
1. It is a space telescope that was launched into low Earth orbit in 2000.
2. It is a joint venture between the US (NASA) and Japan (JAXA) space agencies.
Which of the statements given above is/are correct?
(a) 1 only
(b) 2 only
(c) Both 1 and 2
(d) Neither 1 nor 2
Ans: (d)
Explanation:

Telegram: https://t.me/insightsIAStips
54
Youtube: https://www.youtube.com/channel/UCpoccbCX9GEIwaiIe4HLjwA
Revision Through MCQs (RTM) Compilation (July - 2022)

• S1: NASA's James Webb Space Telescope (JWST) launched on Dec 25, 2021,
from ESA's launch site at Kourou in French Guiana.
• S2: JWST is a joint venture between the US (NASA), European (ESA) and
Canadian space agencies (CSA).
Refer: https://www.insightsonindia.com/2022/07/13/the-1st-image-from-nasas-james-
webb/
106. Consider the following statements about International Criminal Police Organisation, or
Interpol:
1. It is the global money laundering and terrorist financing watchdog.
2. It was formed by the 1949 UN Security Summit in Paris.
Which of the statements give above is/are correct?
(a) 1 only
(b) 2 only
(c) Both 1 and 2
(d) Neither 1 nor 2
Ans: (d)
Explanation:
• The International Criminal Police Organisation, or Interpol, is a 194-member
intergovernmental organisation.
• Headquartered in Lyon, France.
• Formed in 1923 as the International Criminal Police Commission, and started
calling itself Interpol in 1956.
• India joined the organisation in 1949, and is one of its oldest members.
• The Financial Action Task Force (FATF) is the global money laundering and
terrorist financing watchdog.
Refer: Facts For Prelims: https://www.insightsonindia.com/2022/07/13/mission-2023-
insights-daily-current-affairs-pib-summary-13-july-2022/
107. Regarding Foreign Contribution (Regulation) Act, which of the following statements
is/are true?
1. FCRA is implemented by the Ministry of Home Affairs.
2. Once granted, FCRA registration is valid for three years.
3. The entities covered by the Act does not include an individual.
Select the correct answer using the code below:
(a) 1 only
(b) 2 and 3 only
(c) 1 and 2 only
(d) 1, 2 and 3
Ans: (a)
Explanation:
• FCRAregulates foreign donations and ensures that such contributions do not
adversely affect the internal security of the country.
• The Act, first enacted in 1976 was amended in the year 2010 and then 2020.
• Section 5 of the Foreign Contribution (Regulation) Act, 2010 gives the Union
government “unchecked and unbridled powers” to declare an organisation as
being one of political nature and deny it access to funds from sources abroad.
• FCRA is implemented by the Ministry of Home Affairs.
• Applicability:

Telegram: https://t.me/insightsIAStips
55
Youtube: https://www.youtube.com/channel/UCpoccbCX9GEIwaiIe4HLjwA
Revision Through MCQs (RTM) Compilation (July - 2022)

• The provisions of the Act apply to the territory of India, to citizens of India who
may be outside India and to companies or their branches outside India that are
registered or incorporated in India.
• The entities covered by the Act include an individual, a Hindu undivided family,
an association, or a registered company.
• For how long is approval granted?
• Once granted, FCRA registration is valid for five years. NGOs are expected to
apply for renewal within six months of the date of expiry of registration. In case
of failure to apply for renewal, the registration is deemed to have expired, and
the NGO is no longer entitled to receive foreign funds or utilise its existing funds
without permission from the ministry.
Refer: Facts For Prelims: https://www.insightsonindia.com/2022/07/13/mission-2023-
insights-daily-current-affairs-pib-summary-13-july-2022/
108. Consider the following statements:
1. As per recent amendment to the Indian Forest Act, 1927, forest dwellers have the
right to fell the bamboos grown on forest areas.
2. As per the Scheduled Tribes and Other Traditional Forest Dwellers (Recognition of
Forest Rights) Act, 2006, bamboo is a minor forest produce.
3. The Scheduled Tribes and Other Traditional Forest Dwellers (Recognition of Forest
Rights) Act, 2006 allows ownership of minor forest produce to forest dwellers.
Which of the statements given above is / are correct?
(a) 1 and 2 only
(b) 2 and 3 only
(c) 3 only
(d) 1, 2 and 3
Ans: (b)
Explanation:
• S1: Under the original Act, the definition of tree includes palms, bamboos,
stumps, brush-wood, and canes. The amendment act amends this definition of
tree to remove the word bamboos.
• Since bamboo is defined as a tree under the Act, its inter-state movement
requires permit when in transit in other states. Consequent to the amendment,
felling or transportation of bamboos growing in non-forest areas will not require
any permits.
• See https://www.prsindia.org/billtrack/indian-forest-amendment-bill-2017
• S2: The Act recognises bamboo as an MFP and vests the “right of ownership,
access to collect, use and dispose of minor forest pr oduce” with Scheduled
Tribes and traditional forest dwellers
• See https://www.downtoearth.org.in/news/bamboo-now-a-minor-forest-
produce-33239
• S3: Section 3(1) of the act provides a right of ownership, access to collect, use,
and dispose of minor forest produce( includes all non-timber forest produce of
plant origin) which has been traditionally collected within or outside village
boundaries.
Refer: Facts For Prelims: https://www.insightsonindia.com/2022/07/13/mission-2023-
insights-daily-current-affairs-pib-summary-13-july-2022/

Telegram: https://t.me/insightsIAStips
56
Youtube: https://www.youtube.com/channel/UCpoccbCX9GEIwaiIe4HLjwA
Revision Through MCQs (RTM) Compilation (July - 2022)

109. Nord Stream 2, an export gas pipeline running from


(a) Russia to Germany
(b) Russia to China
(c) Europe to North America
(d) North America to Japan
Ans: (a)
Explanation: Importance of Nord Stream:
• The pipeline is the primary route through which its gas enters Germany and
then travels west and southwards through onshore links to other European
countries.
• Nord Stream 2 Pipeline that would have doubled the flow of Russian gas to
Germany was suspended in February 2022 (due to the Russian invasion of
Ukraine)
Refer: Facts For Prelims: https://www.insightsonindia.com/2022/07/13/mission-2023-
insights-daily-current-affairs-pib-summary-13-july-2022/
110. Consider the following countries:
1. Finland
2. Lithuania
3. Norway
4. Azerbaijan
5. Latvia
Which of the above have borders with Russia?
(a) 1, 2 and 5 only
(b) 1, 2, 3 and 4 only
(c) 3, 4 and 5 only
(d) 1, 2, 3, 4 and 5
Ans: (d)
Explanation:
• Russia borders 14 countries, Azerbaijan, Belarus, China, Estonia, Finland,
Georgia, Kazakhstan, North Korea, Latvia, Lithuania, Mongolia, Norway,
Poland, and Ukraine. It shares maritime borders with Japan, Sweden, Turkey,
and the United States.


Refer: Facts For Prelims: https://www.insightsonindia.com/2022/07/13/mission-2023-
insights-daily-current-affairs-pib-summary-13-july-2022/

Telegram: https://t.me/insightsIAStips
57
Youtube: https://www.youtube.com/channel/UCpoccbCX9GEIwaiIe4HLjwA
Revision Through MCQs (RTM) Compilation (July - 2022)

RTM- REVISION THROUGH MCQS –14th-Jul-2022

111. Consider the following statements:


1. The 73th constitutional amendment act mandated the setting up and devolution of
powers to Urban local bodies (ULBs).
2. Urban governance is part of the state list under the Constitution.
Which of the statements given above is/are correct?
(a) 1 only
(b) 2 only
(c) Both 1 and 2
(d) Neither 1 nor 2
Ans: (b)
Explanation:
• The 74th Constitution Amendment Act was passed in 1992 mandating the
setting up and devolution of powers to urban local bodies (ULBs) as the lowest
unit of governance in cities and towns.
• Constitutional provisions were made for ULBs’ fiscal empowerment.
o Urban governance is part of the state list under the Constitution
Refer: https://www.insightsonindia.com/2022/07/14/the-scale-of-municipal-finances-is-
inadequate/
112. Global Gender Gap Index is released by which of the following?
(a) WEF
(b) UNDP
(c) UNESCO
(d) IMF
Ans: (a)
Explanation:
• India ranks 135 among a total of 146 countries in the Global Gender Gap Index,
2022, released by the World Economic Forum.
• The country is the worst performer in the world in the “health and survival” sub-
index in which it is ranked 146.
Refer: https://www.insightsonindia.com/2022/07/14/india-ranks-135-out-of-146-on-
gender-gap-index/
113. Consider the following statements:
1. The Juvenile Justice Board is an institutional body constituted under Section 4 of
the Juvenile Justice Act, 2015.
2. One or more than one Juvenile Justice Board(s) are established by the National
Commission for Protection of Child Rights (NCPCR) for each state.
Which of the given above statements is/are correct?
(a) 1 only
(b) 2 only
(c) Both 1 and 2
(d) Neither 1 nor 2
Ans: (a)
Explanation:
• The Juvenile Justice Board is an institutional body constituted under Section
4 of the JJ Act, 2015. According to the division of powers, the subject of
administration of criminal justice has been included in the State List (List II,

Telegram: https://t.me/insightsIAStips
58
Youtube: https://www.youtube.com/channel/UCpoccbCX9GEIwaiIe4HLjwA
Revision Through MCQs (RTM) Compilation (July - 2022)

Schedule VII) of the Indian Constitution. Therefore, one or more than one
Juvenile Justice Board(s) are established by the State Government for each
district. The Board exercises its powers and discharges functions relating to the
‘child in conflict with law’ as has been defined under Section 2(13) of this Act.
Refer: https://www.insightsonindia.com/2022/07/14/assessing-juvenility-a-delicate-task-
sc/
114. Which of the following are the reasons for the occurrence of multi-drug resistance in
microbial pathogens in India?
1. Genetic predisposition of some people
2. Taking incorrect doses of antibiotics to cure diseases
3. Using antibiotics in livestock farming
4. Multiple chronic diseases in some people
Select the correct answer using the code given below.
(a) 1 and 2
(b) 2 and 3 only
(c) 1, 3 and 4
(d) 2, 3 and 4
Ans: (d)
Explanation:
• S1 and S2: Selection of resistant microorganisms is exacerbated by
inappropriate use of antimicrobials since a number of microbes are resistant to
these anti-biotics. So, S1 is irrelevant to MDR.
• The practice of adding antibiotics to agricultural feed promotes drug resistance.
• S3: As per WHO, the high volume of antibiotics in food-producing animals
contributes to the development of antimicrobial-resistant bacteria, particularly
in settings of intensive animal production. These bacteria can be transmitted
from animals to humans via direct contact between animals and humans, or
through the food chain and the environment.
See https://www.who.int/foodsafety/areas_work/antimicrobial-
resistance/amrfoodchain/en/
• S4: This is a confusing option, nonetheless seems correct.
• Many medical advances are dependent on the ability to fight infections using
antibiotics, such as for treatment of chronic diseases like diabetes, asthma, and
rheumatoid arthritis (See https://www.cdc.gov/drugresistance/about.html), and
thus, multiple chronic diseases in some people may lead them to take more anti-
biotics than others exacerbating the problem.
• Sometimes healthcare providers prescribe antimicrobials inappropriately,
wishing to placate an insistent patient who has a viral infection or an as-yet
undiagnosed condition. At times there could be a wrong identification of the
disease. All these worsen the problem. Whether this is actually a reason or not is
hard to testify unless we find a solid evidence, but based on reasoning alone this
would be correct.
• Also, MDR provokes obstruction in disease control by intensifying the possibility
of spreading of resistant pathogens, thus, declining efficacy of treatment and,
hence, resulting in prolonged time of infection in patient.
See https://www.hindawi.com/journals/ipid/2014/541340/
Refer: https://www.insightsonindia.com/2022/07/14/antimicrobial-resistance-vaccines/

Telegram: https://t.me/insightsIAStips
59
Youtube: https://www.youtube.com/channel/UCpoccbCX9GEIwaiIe4HLjwA
Revision Through MCQs (RTM) Compilation (July - 2022)

115. With reference to chemical fertilizers in India, consider the following statements:
1. At present, the retail price of chemical fertilizers is market-driven and not
administered by the Government.
2. Ammonia, which is an input of urea, is produced from natural gas.
3. Sulphur, which is a raw material for phosphoric acid fertilizer, is a by-product of all
oil refineries.
Which of the statements given above is/are correct?
(a) 1 only
(b) 2 and 3 only
(c) 2 only
(d) 1, 2 and 3
Ans: (b)
Explanation:
• S1: The Union Government subsidies the urea manufacturing units for the cost
of transportation to facilitate the availability of urea at the same maximum retail
price all over the country. Also, the Government is subsidising fertilizers for their
availability at reasonable prices for agricultural purposes.
See https://fert.nic.in/faq
• S2: Common knowledge.
• S3: Sulphur is a by-product of processing natural gas and refining high-sulphur
crude oils. Even if they use low sulphur crude oil, there is still sulphur emission,
so S3 is correct.
• The primary sources of sulfuric acid emissions are combustion of coal, and the
industries that manufacture or use it in production, including metal smelters,
phosphate fertilizer producers, oil refineries, the chemical industry, battery
manufacturers, manufacturers of fabricated metal products, manufacturers of
electronic components, and manufacturers of measuring and controlling devices.
See https://www.sciencedirect.com/topics/chemistry/phosphate-
fertilizer and https://ibm.gov.in/writereaddata/files/12152016105127IMYB201
5_Sulphur_15122016_Adv.pdf
Refer: facts for prelims: https://www.insightsonindia.com/2022/07/14/mission-2023-
insights-daily-current-affairs-pib-summary-14-july-2022/
116. Consider the following statements about International North–South Transport
Corridor (INSTC):
1. It is a multi-modal transportation established in 2000 for the purpose of promoting
transportation cooperation among the Member States.
2. This corridor connects India Ocean and Persian Gulf to the Caspian Sea via
Pakistan.
Which of the given above statements is/are correct?
(a) 1 only
(b) 2 only
(c) Both 1 and 2
(d) Neither 1 nor 2
Ans: (a)
Explanation:
• It is a 7,200-km-long multi-mode network of ship, rail, and road route for
moving freight.

Telegram: https://t.me/insightsIAStips
60
Youtube: https://www.youtube.com/channel/UCpoccbCX9GEIwaiIe4HLjwA
Revision Through MCQs (RTM) Compilation (July - 2022)

• Regions involved: India, Iran, Afghanistan, Azerbaijan, Russia, Central Asia


and Europe.
• This corridor connects India Ocean and Persian Gulf to the Caspian Sea via Iran,
Islamic Rep., and is then connected to Saint Petersburg and North European via
Russia.


Refer: facts for prelims: https://www.insightsonindia.com/2022/07/14/mission-2023-
insights-daily-current-affairs-pib-summary-14-july-2022/
117. Consider the following statements about Ashgabat Agreement:
1. The Agreement was first signed by Uzbekistan, Turkmenistan, Iran, Oman and
Qatar in 2011.
2. Its objective is to enhance connectivity within the Eurasian region.
3. India recently joined the Ashgabat Agreement.
Which of the given above statements is/are correct?
(a) 1 and 3 only
(b) 2 only
(c) 1 and 2 only
(d) 1, 2 and 3
Ans: (c)
Explanation:
• S1 and S3: India on February 1, 2018 joined the Ashgabat agreement, which
was instituted in April 2011 to establish an international multimodal transport
and transit corridor between Central Asia and the Persian Gulf.
• The Agreement was first signed by Uzbekistan, Turkmenistan, Iran, Oman
and Qatar on 25 April 2011. While Qatar subsequently withdrew from the
agreement in 2013, Kazakhstan and Pakistan joined the grouping in 2016.
The Ashgabat Agreement came into force in April 2016.
• S2: Its objective is to enhance connectivity within the Eurasian region and
synchronize it with other regional transport corridors, including the
International North–South Transport Corridor (INSTC).
Refer: link
118. Consider the following statements about the International Space Station:
1. It is the biggest space laboratory of USA.
2. It orbits around the Earth at the same time it moves around the Sun.
Which of the given above statements is/are correct?
(a) 1 only

Telegram: https://t.me/insightsIAStips
61
Youtube: https://www.youtube.com/channel/UCpoccbCX9GEIwaiIe4HLjwA
Revision Through MCQs (RTM) Compilation (July - 2022)

(b) 2 only
(c) Both 1 and 2
(d) Neither 1 nor 2
Ans: (d)
Explanation:
• S1: It is a multinational collaborative project involving five participating space
agencies: NASA (United States), Roscosmos (Russia), JAXA (Japan), ESA
(Europe), and CSA (Canada). The ownership and use of the space station is
established by intergovernmental treaties and agreements.
• S2: It orbits around Earth. link
Refer: https://www.nasa.gov/mission_pages/station/main/index.html
119. In the context of any country, which one of the following would be considered as part
of its social capital?
(a) The proportion of literates in the population
(b) The stock of its buildings, other infrastructure and machines
(c) The size of population in the working age group
(d) The level of mutual trust and harmony in the society
Ans: (d)
Explanation:
• Option A and D are examples of human capital and option B is physical capital.
• Straight from Wikipedia: “Social capital broadly refers to those factors of
effectively functioning social groups that include such things as interpersonal
relationships, a shared sense of identity, a shared understanding, shared norms,
shared values, trust, cooperation, and reciprocity.” Option D is the correct
answer.
Refer: UPSC CSE 2019
120. With reference to the management of minor minerals in India, consider the following
statements:
1. Sand is a ‘minor mineral’ according to the prevailing law in the country.
2. State Governments have the power to grant mining leases of minor minerals, but the
powers regarding the formation of rules related to the grant of minor minerals lie
with the Central Government.
3. State Governments have the power to frame rules to prevent illegal mining of minor
minerals.
Which of the statements given above is / are correct?
(a) 1 and 3 only
(b) 2 and 3 only
(c) 3 only
(d) 1, 2 and 3
Ans: (a)
Explanation:
• S1: There is some ambiguity in sand being a minor mineral. Sand can be a major
mineral or a minor mineral depending on where it is used; same is the case for
limestone. A brief discussion follows.
See http://www.arthapedia.in/index.php?title=Minor_Minerals
• According to section 3(e) of the Mines and Minerals (Development and Regulation)
Act, 1957 “Minor Minerals” means building stones, gravel, ordinary
clay, ordinary sand other than sand used for prescribed purposes.

Telegram: https://t.me/insightsIAStips
62
Youtube: https://www.youtube.com/channel/UCpoccbCX9GEIwaiIe4HLjwA
Revision Through MCQs (RTM) Compilation (July - 2022)

• The term ordinary sand used in clause (e) of Section-3 of the MMDR Act, 1957
has been further clarified in Rule 70 of the Mineral Concession Rule, 1960.
• It is stated that sand shall not be treated as minor mineral when used for any
of the following purposes namely; (i) purpose of refractory and manufacturer of
ceramic, (ii) metallurgical purposes, (iii)optical purposes, (iv) purposes of stowing
in coal mines, (v) for manufacture of silvicrete cement, (vi) manufacture of
sodium silicate and (vii) manufacture of pottery and glass.
• However, given the ambiguity, we need to assume that UPSC is perhaps referring
to ordinary sand. S1 is provisionally correct.
• S2 and S3: The central government has the power to notify “minor minerals”
under section 3 (e) of the MMDR Act, 1957. On the other hand, as per Section 15
of the MMDR Act, 1957 State Governments have complete powers for making
Rules for grant of concessions in respect of extraction of minor minerals and levy
and collection of royalty on minor minerals.
• The power to frame policy and legislation relating to minor minerals is entirely
delegated to the State Governments while policy and legislation relating to the
major minerals are dealt by the Ministry of Mines under Union /Central
Government. So, S2 is incorrect and S3 is correct.
Refer: UPSC CSE 2019

RTM- REVISION THROUGH MCQS –15th-Jul-2022

121. With reference to Beti Bachao Beti Padhao, consider the following statements:
1. It is a joint initiative of the Ministry of Women and Child Development and Ministry
of Finance.
2. It aims to address sex selective abortion and the declining child sex ratio in India.
Which of the statements given above is/are correct?
(a) 1 only
(b) 2 only
(c) Both 1 and 2
(d) Neither 1 nor 2
Ans: (b)
Explanation:
• Beti Bachao Beti Padhao will now be extended across the country.
• BBBP is the Centre’s flagship programme (under the Women and Child
Development Ministry) for women’s empowerment, which focuses on
the education of girls child and improving the sex ratio.
o It was launched in January 2015 to address sex-selective abortion and
the declining child sex ratio which was at 918 girls for every 1,000
boys in 2011.
o It is a joint initiative of the Ministry of Women and Child Development,
Ministry of Health and Family Welfare and Ministry of Human
Resource Development.
o The programme is operational in 405 districts at present.
Refer: https://www.insightsonindia.com/2022/07/15/bbbp-to-be-extended-to-all-
districts/
122. What is ‘I2U2’, sometime mentioned in the news?
(a) A joint project between NASA and ISRO

Telegram: https://t.me/insightsIAStips
63
Youtube: https://www.youtube.com/channel/UCpoccbCX9GEIwaiIe4HLjwA
Revision Through MCQs (RTM) Compilation (July - 2022)

(b) A strategy initiated by the India that seeks to connect Asia with USA
(c) A new grouping of India with some middle East nations
(d) An agreement on the Indian nuclear program
Ans: (c)
Explanation:
• It was initially formed in October, 2021 following the Abraham Accords between
Israel and the UAE, to deal with issues concerning maritime security,
infrastructure and transport in the region.
• At that time, it was called the ‘International Forum for Economic
Cooperation’, referred to as the ‘West Asian Quad’.
• I2U2 initiative is a new grouping of India, Israel, USA and UAE.
• This will not only revitalize and re-energize the system of alliances and
partnerships around the world, but also stitch together partnerships that did not
exist previously or were not utilized to their full extent.
Refer: https://www.insightsonindia.com/2022/07/15/i2u2-summit-uae-to-invest-2-
billion-to-develop-integrated-food-parks-across-india/
123. With reference to ‘International Renewable Energy Agency’ (IRENA), consider the
following statements:
1. It is the first international organisation to focus exclusively on renewable energy
addressing needs in both industrialised and developing countries.
2. It was founded in 2009 and its statute entered into force on 8 July 2010.
3. The agency is headquartered in Paris, France.
Which of the statements given above is/are correct?
(a) 1 and 2 only
(b) 2 and 3 only
(c) 1 and 3 only
(d) 1, 2 and 3
Ans: (a)
Explanation:
• The International Renewable Energy Agency (IRENA) is an intergovernmental
organization mandated to facilitate cooperation, advance knowledge, and
promote the adoption and sustainable use of renewable energy.
• It is the first international organisation to focus exclusively on renewable
energy, addressing needs in both industrialised and developing countries.
• It was founded in 2009 and its statute entered into force on 8 July 2010. The
agency is headquartered in Masdar City, Abu Dhabi.
• IRENA is an official United Nations observer.
Refer: https://www.insightsonindia.com/2022/07/15/i2u2-summit-uae-to-invest-2-
billion-to-develop-integrated-food-parks-across-india/
124. Consider the following statements regarding Off-budget borrowing.
1. Off-budget borrowing helps keep the country’s fiscal deficit within acceptable limits.
2. Off-budget borrowings is not part of the calculation of the fiscal indicators and does
not have any fiscal implications.
3. Public sector banks are not authorised to fund off-budget expenses.
Which of the statements given above is/are correct?
(a) 1 only
(b) 2 and 3 only
(c) 1 and 3 only

Telegram: https://t.me/insightsIAStips
64
Youtube: https://www.youtube.com/channel/UCpoccbCX9GEIwaiIe4HLjwA
Revision Through MCQs (RTM) Compilation (July - 2022)

(d) 1, 2 and 3
Ans: (a)
Explanation:
• Off-budget borrowings are loans that are taken not by the Centre directly, but by
another public institution which borrows on the directions of the central
government. Such borrowings are used to fulfil the government’s expenditure
needs. But since the liability of the loan is not formally on the Centre, the loan is
not included in the national fiscal deficit. This helps keep the country’s fiscal
deficit within acceptable limits. Comptroller and Auditor General report of 2019
points out, this route of financing puts major sources of funds outside the
control of Parliament. “Such off-budget financing is not part of the calculation of
the fiscal indicators despite fiscal implications,” said the report.
• The government can ask an implementing agency to raise the required funds
from the market through loans or by issuing bonds. In the Budget presentation
for 2020-21, the government paid only half the amount budgeted for the food
subsidy bill to the Food Corporation of India. The shortfall was met through a
loan from the National Small Savings Fund. Public sector banks are also used to
fund off-budget expenses. For example, loans from PSU banks were used to
make up for the shortfall in the release of fertiliser subsidy.
Refer: https://www.insightsonindia.com/2022/07/15/centre-relaxes-norms-for-adjusting-
states-off-budget-borrowings/
125. Consider the following statements:
1. Whenever forest land is diverted for non-forest purposes, it is mandatory under the
Forest Rights Act (FRA), 2006 that an equivalent area of non-forest land has to be
taken up for compensatory afforestation.
2. As per the rules, 90% of the Compensatory Afforestation Fund money is to be given
to the states while 10% is to be retained by the Centre.
3. As per recent notification by GOI, states which have a forest land of more than 75%
of their geographical area need not provide non-forest land for compensatory
afforestation.
Which of the given above statements is/are correct?
(a) 1 and 2 only
(b) 2 only
(c) 1 and 3 only
(d) 2 and 3 only
Ans: (d)
Explanation:
• S3: the Ministry of Environment notified that states which have a forest land of
more than 75% of their geographical area need not provide non-forest land for
compensatory afforestation. Instead, land can be taken up in states with lesser
forest cover. Further, it was also notified that the minimum area of
compensatory land should be five hectares if the land is not contiguous to a
forest.
• S1: According to the Forest (Conservation) Act, 1980, each time forest land is
diverted, the project proponent has to pay the state to undertake plantation and
for the ecosystem services lost due to diverting forest land, called Net Present
Value (NPV). Whenever forest land is diverted for non-forest purposes, it is
mandatory under the Forest (Conservation) Act, 1980 that an equivalent area of
non-forest land has to be taken up for compensatory afforestation.

Telegram: https://t.me/insightsIAStips
65
Youtube: https://www.youtube.com/channel/UCpoccbCX9GEIwaiIe4HLjwA
Revision Through MCQs (RTM) Compilation (July - 2022)

• S2: As per the rules, 90% of the Compensatory Afforestation Fund money is
to be given to the states while 10% is to be retained by the Centre.
Refer: https://www.insightsonindia.com/2022/07/15/forest-conservation-rules/
126. With reference to agriculture in India, how can the technique of `genome sequencing’,
often seen in the news, be used in the immediate future?
1. Genome sequencing can be used to identify genetic markers for disease resistance
and drought tolerance in various crop plants.
2. This technique helps in reducing the time required to develop new varieties of crop
plants.
3. It can be used to decipher the host-pathogen relationships in crops.
Select the correct answer using the code given below:
(a) 1 only
(b) 2 and 3 only
(c) 1 and 3 only
(d) 1, 2 and 3
Ans: (d)
Explanation:
• Genome sequencing is figuring out the order of DNA nucleotides, or bases, in a
genome—the order of As, Cs, Gs, and Ts that make up an organism’s DNA.
• Justification: Statement 1: Currently available newborn screening (genome) for
childhood diseases allows detection of rare disorders that can be prevented or
better treated by early detection and intervention.
• https://www.scientificamerican.com/article/whole-genome-sequencing-predict-
disease/
• Statement 2: Naturally if the gene potential can be identified by screening and
sequencing, it will help in better genetic engineering.
• Statement 3: This can be reasoned logically based on the above.
• https://en.wikipedia.org/wiki/Whole_genome_sequencing
Refer: https://www.insightsonindia.com/2022/07/15/genomics/

127. Consider the following statements regarding IndiGen programme.


1. It aims to undertake whole genome sequencing of thousands of Indian individuals
representing diverse ethnic groups from India.
2. The human genome data sets could be utilized for prioritizing Pharmacogenomics
variants specific for Indian population for optimizing therapy and minimizing
adverse events.
3. It is the initiative of Council of Scientific and Industrial Research (CSIR).
Which of the above statements is/are correct?
(a) 1 only
(b) 1 and 2 only
(c) 2 and 3 only
(d) 1, 2 and 3
Ans: (d)
Explanation: Genomics for Public Health in India (IndiGen) programme:
• The IndiGen initiative was undertaken by CSIR in April 2019, which was
implemented by the CSIR-Institute of Genomics and Integrative Biology (IGIB),
Delhi and CSIR-Centre for Cellular and Molecular Biology (CCMB), Hyderabad.

Telegram: https://t.me/insightsIAStips
66
Youtube: https://www.youtube.com/channel/UCpoccbCX9GEIwaiIe4HLjwA
Revision Through MCQs (RTM) Compilation (July - 2022)

• It aims to undertake whole genome sequencing of thousands of individuals


representing diverse ethnic groups from India.
• The objective is to enable genetic epidemiology and develop public health
technologies applications using population genome data.
• The human genome data sets would also be utilized for prioritizing
Pharmacogenomics variants specific for Indian population for optimizing therapy
and minimizing adverse events.
Refer: https://www.insightsonindia.com/2022/07/15/genomics/
128. “It needs plain alluvial soil and standing water. During the monsoon season, the
monsoon climate offers a suitable environment for growing jute (warm and wet).
Temperatures from 20 to 40 °C and relative humidity of 70%–80% are favorable for
successful cultivation. Jute requires 5–8 cm (2–3 in) of rainfall weekly and more during
the sowing time.” Which one of the following is that crop?
(a) Cotton
(b) Jute
(c) Sugarcane
(d) Tea
Ans: (b)
Explanation:
• The jute plant needs plain alluvial soil and standing water. During the monsoon
season, the monsoon climate offers a suitable environment for growing jute
(warm and wet). Temperatures from 20 to 40 °C (68–104 °F) and relative
humidity of 70%–80% are favorable for successful cultivation. Jute requires 5–8
cm (2–3 in) of rainfall weekly and more during the sowing time. Soft water is
necessary for jute production.
Refer: facts for prelims: https://www.insightsonindia.com/2022/07/15/mission-2023-
insights-daily-current-affairs-pib-summary-15-july-2022/
129. Consider the following statements about Commission for air quality management
(CAQM):
1. It is a statutory body.
2. Its authority supersedes the central and state pollution control board of all states.
Which of the statements given above is/are correct?
(a) 1 only
(b) 2 only
(c) Both 1 and 2
(d) Neither 1 nor 2
Ans: (a)
Explanation:
• It released the policy to curb air pollution in the Delhi-NCR region
• Background: The Commission for Air Quality Management (CAQM) was
established as a statutory body by the government in August 2021 as an
overarching body to carry out air quality management in Delhi NCR.
• Its authority supersedes the central and state pollution control board of
Delhi, Haryana, UP, Punjab and Rajasthan.
Refer: facts for prelims: https://www.insightsonindia.com/2022/07/15/mission-2023-
insights-daily-current-affairs-pib-summary-15-july-2022/

Telegram: https://t.me/insightsIAStips
67
Youtube: https://www.youtube.com/channel/UCpoccbCX9GEIwaiIe4HLjwA
Revision Through MCQs (RTM) Compilation (July - 2022)

130. With reference to India’s Desert National Park, which of the following statements are
correct?
1. It is spread over two districts.
2. There is no human habitation inside the Park.
3. It is one of the natural habitats of Great Indian Bustard.
Select the correct answer using the code given below:
(a) 1 and 2 only
(b) 2 and 3 only
(c) 1 and 3 only
(d) 1, 2 and 3
Ans: (c)
Explanation:
• S1: Situated in Jaisalmer and Barmer districts of Indian state Rajasthan.
• S2: This is clearly incorrect. A national park can have human habitation.
• S3: This is common knowledge.


Refer: facts for prelims: https://www.insightsonindia.com/2022/07/15/mission-2023-
insights-daily-current-affairs-pib-summary-15-july-2022/

RTM- REVISION THROUGH MCQS –16th-Jul-2022

131. With reference to Shanghai Cooperation Organization (SCO), consider the following
statements:
1. It is a permanent intergovernmental international organization.
2. It founded in Shanghai on 15 June 2001.
3. The Heads of Government Council (HGC) is the supreme decision-making body in
the SCO.
Which of the statements given above is/are correct?
(a) 1 only
(b) 1 and 2 only
(c) 2 and 3 only
(d) 1, 2 and 3

Telegram: https://t.me/insightsIAStips
68
Youtube: https://www.youtube.com/channel/UCpoccbCX9GEIwaiIe4HLjwA
Revision Through MCQs (RTM) Compilation (July - 2022)

Ans: (b)
Explanation:
• The Shanghai Cooperation Organization (SCO) is an intergovernmental
organization founded in Shanghai on 15 June 2001.
• The Heads of State Council (HSC) is the supreme decision-making body
in the SCO. It meets once a year and adopts decisions and guidelines on all
important matters of the organisation. The SCO Heads of Government Council
(HGC) meets once a year to discuss the organisation's multilateral cooperation
strategy and priority areas, to resolve current important economic and other
cooperation issues, and also to approve the organisation's annual budget. The
SCO's official languages are Russian and Chinese.
Refer: https://www.insightsonindia.com/2022/07/16/iran-and-belarus-to-be-the-newest-
sco-members/
132. S-400 air defence missile system, recently seen in news is developed by
(a) United States
(b) China
(c) Israel
(d) Russia
Ans: (d)
Explanation:
• The S-400 Triumf, (NATO calls it SA-21 Growler), is a mobile, surface-to-air
missile system (SAM) designed by Russia. It is the most dangerous
operationally deployed modern long-range SAM (MLR SAM) in the world,
considered much ahead of the US-developed Terminal High Altitude Area
Defense system (THAAD).
Refer: https://www.insightsonindia.com/2022/07/16/waiver-to-india-for-purchase-of-s-
400-missile-system/
133. Consider the following statements regarding Countering America’s Adversaries
through Sanctions Act (CAATSA).
1. Its core objective is to counter Iran, Russia and North Korea through punitive
measures.
2. It empowers the US President to impose some sanctions on persons engaged in a
“significant transaction” with Russian defence and intelligence sectors.
Which of the above statements is/are correct?
(a) 1 only
(b) 2 only
(c) Both 1 and 2
(d) Neither 1 nor 2
Ans: (c)
Explanation:
• Countering America’s Adversaries through Sanctions Act (CAATSA) was passed
unanimously by the US Congress and signed reluctantly by Donald Trump.
Enacted on August 2, 2017, its core objective is to counter Iran, Russia and
North Korea through punitive measures.
• Title II of the Act primarily deals with sanctions on Russian interests such as its
oil and gas industry, defence and security sector, and financial institutions, in
the backdrop of its military intervention in Ukraine and its alleged meddling in
the 2016 US Presidential elections.

Telegram: https://t.me/insightsIAStips
69
Youtube: https://www.youtube.com/channel/UCpoccbCX9GEIwaiIe4HLjwA
Revision Through MCQs (RTM) Compilation (July - 2022)

• Section 231 of the Act empowers the US President to impose at least five of the
12 listed sanctions — enumerated in Section 235 of the Act — on persons
engaged in a “significant transaction” with Russian defence and intelligence
sectors.
Refer: https://www.insightsonindia.com/2022/07/16/waiver-to-india-for-purchase-of-s-
400-missile-system/
134. ‘Namsai Declaration’ was in news recently, is related to
(a) good governance
(b) interlinking of rivers
(c) disaster management
(d) settling border dispute
Ans: (d)
Explanation: Namsai Declaration:
• This declaration is signed between the states of Assam and Arunachal Pradesh
on July 15, 2022, at Namsai, Arunachal Pradesh to minimize the border
dispute between the two states in respect of 123 villages placed before the
Local Commission by Arunachal Pradesh.
• According to the declaration, all border issues between Assam and Arunachal
Pradesh will be confined to those raised before the local commission in 2007.
Refer: https://www.insightsonindia.com/2022/07/16/assam-arunachal-pradesh-sign-
pact-aiming-to-solve-the-border-dispute/
135. “Global Findex Database” is released by which of the following?
(a) IMF
(b) WEF
(c) WTO
(d) WBG
Ans: (d)
Explanation: The Global Findex Database 2021 - World Bank Group
• World Bank’s Global Findex database surveyed how people in 123 economies
use formal and informal financial services e.g., cards, ATMs, mobile phones, and
the internet.
• India-specific findings:
o Low access to formal banking in India and there is a lack of trust.
o Aadhar contributed to account ownership in about 80% of adults (from
35% in 2011)
o Drop-in fraud and leakage: Transitioning from cash to biometric smart
cards has reduced leakage by 47% in pension payments.
Refer: https://www.insightsonindia.com/2022/07/16/financial-inclusion/
136. India's first World Heritage City Status was accorded to which of the following city by
UNESCO?
(a) Bodh Gaya
(b) Jaipur
(c) Ahmedabad
(d) Varanasi
Ans: (c)
Explanation:
• Indian city Ahmedabad and the state of Kerala have been mentioned
in TIME magazine’s list of the world’s greatest places of 2022.

Telegram: https://t.me/insightsIAStips
70
Youtube: https://www.youtube.com/channel/UCpoccbCX9GEIwaiIe4HLjwA
Revision Through MCQs (RTM) Compilation (July - 2022)

• The commercial capital of Gujarat, Ahmedabad, has been formally accorded the
status of India's first World Heritage City by UNESCO.
Refer: facts for prelims: https://www.insightsonindia.com/2022/07/16/mission-2023-
insights-daily-current-affairs-pib-summary-16-july-2022/
137. SAMARTH was launched by the
(a) Ministry of Micro, Small & Medium Enterprises
(b) Ministry of Coal
(c) Ministry of Power
(d) Ministry of Social Justice and Empowerment
Ans: (c)
Explanation:
• SAMARTH was launched by the Ministry of power (2021) and mandates all
thermal power plants in India use 5-10% of biomass alongside coal to produce
power. Consequently, it promotes research on modern boilers which can handle
a greater amount of silica and alkalis in biomass pellets.
• *Don’t get confused with the SAMARTH Scheme of the Ministry of textile (for
capacity building in the textile sector)
Refer: facts for prelims: https://www.insightsonindia.com/2022/07/16/mission-2023-
insights-daily-current-affairs-pib-summary-16-july-2022/
138. With reference to the carbon nanotubes, consider the following statements:
1. They can be used as carriers of drugs and antigens in the human body.
2. They can be made into artificial blood capillaries for an injured part of human body.
3. They can be used in biochemical sensors.
4. Carbon nanotubes are biodegradable
Which of the statements given above are correct?
(a) 1 and 2 only
(b) 2, 3 and 4 only
(c) 1, 3 and 4 only
(d) 1, 2, 3, and 4
Ans: (c)
Explanation:
• S1: The nanotube’s structure allows it to be used as a container for transporting
a drug in the body. A molecule of the drug can be placed inside the nanotube
cage. This keeps the drug ‘wrapped up’ until it reaches the site where it is
needed. In this way, a dose that might be damaging to other parts of the body
can be delivered safely to, for example, a tumour.
o See https://www.scielo.br/pdf/bjps/v49n4/v49n4a02.pdf
• S3: This is correct. See https://technology.nasa.gov/patent/TOP2-104
o NASA has successfully demonstrated a miniaturized electronics
technology for in-vitro detecting specific biomarker signatures, which is
based on incorporating embedded vertically aligned carbon
nanotubes…in diagnostics devices.
• S4: Recently, it has been demonstrated that functionalized carbon nanotubes
can be degraded by oxidative enzymes.
Refer: facts for prelims: https://www.insightsonindia.com/2022/07/16/mission-2023-
insights-daily-current-affairs-pib-summary-16-july-2022/
139. Consider the following statements regarding Asiatic black bear (Ursus thibetanus):
1. It is listed as Vulnerable on the IUCN Red List.

Telegram: https://t.me/insightsIAStips
71
Youtube: https://www.youtube.com/channel/UCpoccbCX9GEIwaiIe4HLjwA
Revision Through MCQs (RTM) Compilation (July - 2022)

2. It is one of the largest carnivores of Senchal Wildlife Sanctuary (WLS).


Which of the above statements is/are correct?
(e) 1 only
(f) 2 only
(g) Both 1 and 2
(h) Neither 1 nor 2
Ans: (c)
Explanation:
• Asiatic black bear (Ursus thibetanus) is one of the largest carnivores of Senchal
Wildlife Sanctuary (WLS), West Bengal.


Refer: facts for prelims: https://www.insightsonindia.com/2022/07/16/mission-2023-
insights-daily-current-affairs-pib-summary-16-july-2022/
140. Consider the following pairs
Famous place River
1. Pandharpur Chandrabhaga
2. Tiruchirappalli Cauvery
3. Hampi Malaprabha
Which of the pairs given above are correctly matched?
(a) 1 and 2 only
(b) 2 and 3 only
(c) 1 and 3 only
(d) 1, 2 and 3
Ans: (a)
Explanation:
• The Bhima River is a major river in Western India and South India. The river is
also referred to as Chandrabhaga River, especially at Pandharpur, as it
resembles the shape of the Moon.

Telegram: https://t.me/insightsIAStips
72
Youtube: https://www.youtube.com/channel/UCpoccbCX9GEIwaiIe4HLjwA
Revision Through MCQs (RTM) Compilation (July - 2022)

• Hampi is situated on the banks of the Tungabhadra River in the eastern part of
central Karnataka.
• The two major rivers draining Tiruchirappalli are the Kaveri and its tributary the
Kollidam
Refer: facts for prelims: https://www.insightsonindia.com/2022/07/16/mission-2023-
insights-daily-current-affairs-pib-summary-16-july-2022/

RTM- REVISION THROUGH MCQS –18th-Jul-2022

141. Which one of the following is/are best describes the ‘Zombie fires’?
(a) Fires burn trees up their entire length to the top.
(b) Fires burn only surface litter and duff.
(c) Fires occur in deep accumulations of humus, peat and similar dead vegetation that
become dry enough to burn.
(d) Fires that continues to burn underground and then reignites on the surface after a
period of time.
Ans: (d)
Explanation: Types of wildland fires: There are three basic types of forest fires:
• Crown fires burn trees up their entire length to the top. These are the most
intense and dangerous wildland fires.
• Surface fires burn only surface litter and duff. These are the easiest fires to put
out and cause the least damage to the forest.
• Ground fires (sometimes called underground or subsurface fires) occur in deep
accumulations of humus, peat and similar dead vegetation that become dry
enough to burn. These fires move very slowly, but can become difficult to fully
put out, or suppress. Occasionally, especially during prolonged drought, such
fires can smoulder all winter underground and then emerge at the surface again
in spring.
• A zombie fire is a fire that continues to burn underground and then reignites on
the surface after a period of time.
Refer: https://www.insightsonindia.com/2022/07/18/forest-fires-2/
142. Consider the following statements:
1. Constitution of India does not define the word bail but only categories offences as
‘bailable’ and ‘non-bailable’.
2. Constitution of India empowers magistrates to grant bail for bailable offences as a
matter of right.
Which of the statements given above is/are correct?
(a) 1 only
(b) 2 only
(c) Both 1 and 2
(d) Neither 1 nor 2
Ans: (d)
Explanation:
• The CrPC does not define the word bail but only categories offences under
the Indian Penal Code as ‘bailable’ and ‘non-bailable’.
• The CrPC empowers magistrates to grant bail for bailable offences as a
matter of right.

Telegram: https://t.me/insightsIAStips
73
Youtube: https://www.youtube.com/channel/UCpoccbCX9GEIwaiIe4HLjwA
Revision Through MCQs (RTM) Compilation (July - 2022)

• This would involve release on furnishing a bail bond, without or without


security.
• In the case of Non-bailable offences, a magistrate would determine if the accused
is fit to be released on bail.
• Non-bailable offences are cognisable, which enables the police officer to arrest
without a warrant.
Refer: https://www.insightsonindia.com/2022/07/18/hasty-arrests-near-impossible-bail-
show-need-for-overhaul-cji/
143. Consider the following statements
1. Vice-President is a member and chairman of Council of States.
2. Vice-President can be removed only through a formal impeachment process similar
to that of President.
3. Dr S. Radhakrishnan was elected as Vice President continuously for two terms.
Which of the above statements is/are correct?
(a) 3 only
(b) 1 and 3 only
(c) 2 and 3 only
(d) 1 only
Ans: (a)
Explanation:
• Vice-President is not a member, however he/she is the chairman of Council of
States. There is no exact procedure mentioned in Indian Constitution for the
removal of Vice President.
Refer: https://www.insightsonindia.com/2022/07/18/how-the-vice-president-of-india-is-
elected-what-the-constitution-says-about-the-post/
144. Consider the following statements regarding the Office of Vice President of India.
1. The Vice President takes over the office of the President when there is a vacancy.
2. The Vice President is removed from his office by a resolution of both the houses of
the parliament by a special majority.
Which of the above statements is/are correct?
(a) 1 only
(b) 2 only
(c) Both 1 and 2
(d) Neither 1 nor 2
Ans: (a)
Explanation:
• The Vice President is elected for five years. His election method is similar to that
of the President, the only difference is that members of State legislatures are not
part of the electoral college.
• The Vice President acts as the President only until a new President is
elected. B. D. Jatti acted as President on the death of Fakhruddin Ali Ahmed
until a new President was elected.
• The impeachment of Vice President is different from that of the President. In the
latter case, both houses must pass the resolution by a special majority.
• But the VP may be removed from his office by a resolution of the Rajya
Sabha passed by an effective majority and agreed to by the Lok Sabha with
simple majority.
Refer: https://www.insightsonindia.com/2022/07/18/how-the-vice-president-of-india-is-
elected-what-the-constitution-says-about-the-post/

Telegram: https://t.me/insightsIAStips
74
Youtube: https://www.youtube.com/channel/UCpoccbCX9GEIwaiIe4HLjwA
Revision Through MCQs (RTM) Compilation (July - 2022)

145. To be eligible for election as Vice-President, a person should fulfil which of the
following conditions?
1. He should not be less than 30 years of age.
2. He should be qualified for election as a member of the Rajya Sabha.
3. He should not hold any office of profit under the Union government or any state
government or any local authority or any other public authority.
Select the correct answer code:
a) 1 and 2 only
b) 2 only
c) 2 and 3 only
d) 1, 2 and 3
Ans: (c)
Explanation:
• To be eligible for election as Vice-President, a person should fulfil the following
qualifications:
o He should be a citizen of India.
o He should have completed 35 years of age.
o He should be qualified for election as a member of the Rajya Sabha.
o He should not hold any office of profit under the Union government
or any state government or any local authority or any other public
authority.
Refer: https://www.insightsonindia.com/2022/07/18/how-the-vice-president-of-india-is-
elected-what-the-constitution-says-about-the-post/
146. Which of the following factors/policies were affecting the price of rice in India in the
recent past?
1. Minimum Support Price
2. Government’s trading
3. Government’s stockpiling
4. Consumer subsidies
Select the correct answer using the code given below:
(a) 1,2 and 4 only
(b) 1,3 and 4 only
(c) 2 and 3 only
(d) 1, 2, 3 and 4
Ans: (d)
Explanation:
• All of the above activities will affect the price of rice in India.
• Context: India has witnessed a shortfall in rice acreage while the overall crop
coverage has risen.
Refer: https://www.insightsonindia.com/2022/07/18/explained-is-there-a-crisis-in-rice/
147. Among the following, which one is the largest exporter of rice in the world in the last
five years?
(a) China
(b) India
(c) Myanmar
(d) Vietnam
Ans: (b)
Explanation: Data about Rice:

Telegram: https://t.me/insightsIAStips
75
Youtube: https://www.youtube.com/channel/UCpoccbCX9GEIwaiIe4HLjwA
Revision Through MCQs (RTM) Compilation (July - 2022)

• It is India’s largest agricultural crop (accounting for over 40% of the total
foodgrain output)
• India is the world’s biggest exporter (around 40% of the world’s export)
• India is 2nd largest rice producer in the world after China.
Refer: https://www.insightsonindia.com/2022/07/18/explained-is-there-a-crisis-in-rice/
148. Which one of the following statements best describes the role of B cells and T cells in
the human body?
(a) They protect the environmental allergens. body
(b) They alleviate the body’s pain and inflammation.
(c) They act as immunosuppressants in the body.
(d) They protect the body from the diseases caused by pathogens.
Ans: (d)
Explanation:
• T cells and B cells are the most critical components of the adaptive immune
system. These cells are essential for fighting disease and play an important role
in regulating hypersensitivity to harmless or “self” antigens.

• T cells and B cells both recognize specific antigens via a complementary
receptor, followed by activation and proliferation to specifically bind to the
antigen of the infecting pathogen.
Refer: facts for prelims: https://www.insightsonindia.com/2022/07/18/mission-2023-
insights-daily-current-affairs-pib-summary-18-july-2022/
149. Regarding “Jagriti mascot”, consider the following statements:
1. It is a mascot created by the Reserve Bank of India.
2. It aims to "empower consumers and generate awareness of their rights".
Which of the above statements is/are correct?
(e) 1 only
(f) 2 only
(g) Both 1 and 2
(h) Neither 1 nor 2
Ans: (b)
Explanation:
• It is a mascot for empowering consumers and generating awareness of their
rights. It will spread awareness about consumer rights, the consumer protection
act 2019, Hallmarking, the provision of weights and measures act etc.
• Other initiative: “Jago Grahak Jago” campaign, National Consumer Helpline
(NCH), Consumer welfare fund
• Launched by the Department of Consumer Affairs (DoCA)
Refer: facts for prelims: https://www.insightsonindia.com/2022/07/18/mission-2023-
insights-daily-current-affairs-pib-summary-18-july-2022/
150. Which one of the following is the best description of ‘Dunagiri’, that was in the news
recently?
(a) Amphibious warfare ship
(b) Nuclear-powered submarine
(c) Torpedo launch and recovery vessel
(d) Multi-role stealth frigate
Ans: (d)
Explanation:

Telegram: https://t.me/insightsIAStips
76
Youtube: https://www.youtube.com/channel/UCpoccbCX9GEIwaiIe4HLjwA
Revision Through MCQs (RTM) Compilation (July - 2022)

• Defence Minister recently launched Dunagiri (latest ship of 17A frigate)-


Nilgiri Class. This is the fourth and latest ship in this class.
• It has been designed in-house by the Indian Navy’s Directorate of Naval Design
(DND)
• The Shivalik class or Project 17 class is a class of multi-role stealth
frigates in service with the Indian Navy. They are the first stealth warships built
in India.
• P17A Frigates (Nilgiri Class ) are follow-on class of the P17 (Shivalik Class)
Frigates with improved stealth features
Refer: facts for prelims: https://www.insightsonindia.com/2022/07/18/mission-2023-
insights-daily-current-affairs-pib-summary-18-july-2022/

RTM- REVISION THROUGH MCQS –19th-Jul-2022

151. With reference to ‘Minorities’ (or ‘Minority’) in India, which of the following statements
is/are correct?
1. All minorities have the right to establish and administer educational institutions of
their choice.
2. The protection under Article 29 is confined only to minorities and does not extend to
any section of citizens.
Select the correct answer using the codes given below:
(a) 1 only
(b) 2 only
(c) Both 1 and 2
(d) Neither 1 nor 2
Ans: (a)
Explanation:
• Article 29:
o It provides that any section of the citizens residing in any part of India
having a distinct language, script or culture of its own, shall have the
right to conserve the same.
o It grants protection to both religious minorities as well as linguistic
minorities.
o However, the SC held that the scope of this article is not necessarily
restricted to minorities only, as the use of the word ‘section of citizens’ in
the Article includes minorities as well as the majority.
• Article 30:
o S1: All minorities shall have the right to establish and administer
educational institutions of their choice.
o S2: The protection under Article 30 is confined only to minorities
(religious or linguistic) and does not extend to any section of citizens (as
under Article 29).
Refer: https://www.insightsonindia.com/2022/07/19/minority-status-of-religious-and-
linguistic-communities-is-state-dependent-sc/

Telegram: https://t.me/insightsIAStips
77
Youtube: https://www.youtube.com/channel/UCpoccbCX9GEIwaiIe4HLjwA
Revision Through MCQs (RTM) Compilation (July - 2022)

152. With reference to the Special Officer for Linguistic Minorities, consider the following
statements:
1. Special Officer for Linguistic Minorities has been inserted to the constitution by the
42nd Constitutional Amendment Act.
2. It would be the duty of the Special Officer to investigate all matters relating to the
safeguards provided for linguistic minorities under the Constitution.
Which of the statements given above is/are correct?
(a) 1 only
(b) 2 only
(c) Both 1 and 2
(d) Neither 1 nor 2
Ans: (b)
Explanation:
• Originally, the Constitution of India did not make any provision with respect to
the Special Officer for Linguistic Minorities. Later, the States Reorganization
Commission (1953-55) made a recommendation in this regard. Accordingly, the
7th Constitutional Amendment Act of 1956 inserted a new Article 350-B in Part
XVII of the Constitution. This article contains the following provisions: There
should be a Special Officer for Linguistic Minorities. He is to be appointed by the
President of India.
• It would be the duty of the Special Officer to investigate all matters relating to
the safeguards provided for linguistic minorities under the Constitution. He
would report to the President upon those matters at such intervals as the
President may direct. The President should place all such reports before each
House of Parliament and send to the governments of the states concerned.
Refer: https://www.insightsonindia.com/2022/07/19/minority-status-of-religious-and-
linguistic-communities-is-state-dependent-sc/
153. In India, legal service authorities provide free legal services to which of the following
type of citizens?
1. Person with an annual income of less than Rs 1,00,000
2. Transgender with an annual income of less than Rs2,00,000
3. Member of other backward classes(OBC) with an annual income of less than Rs
3,00,000
4. All senior citizens
Select the correct answer using the given code below-
(a) 1 and 2 only
(b) 3 and 4 only
(c) 2 and 3 only
(d) 1 and 4 only
Ans: (a)
Explanation:
• S1 and S3: The sections of the society as enlisted under Section 12 of the Legal
Services Authorities Act are entitled for free legal services, they are :
o A member of a Scheduled Caste or Scheduled Tribe;
o A victim of trafficking in human beings or begar as referred to in Article
23 of the Constitution;
o A woman or a child;
o A mentally ill or otherwise disabled person;

Telegram: https://t.me/insightsIAStips
78
Youtube: https://www.youtube.com/channel/UCpoccbCX9GEIwaiIe4HLjwA
Revision Through MCQs (RTM) Compilation (July - 2022)

o A person under circumstances of undeserved want such as being a victim


of a mass disaster, ethnic violence, caste atrocity, flood, drought,
earthquake or industrial disaster; or
o An industrial workman; or
o In custody, including custody in a protective home within the meaning of
clause (g) of Section 2 of the Immoral Traffic (Prevention) Act, 1956(104 of
1956); or in a juvenile home within the meaning of clause(j) of Section 2
of the Juvenile Justice Act, 1986 (53 of 1986); or in a psychiatric hospital
or psychiatric nursing home within the meaning of clause (g) of Section 2
of the Mental Health Act, 1987(14 of 1987);or
o a person in receipt of annual income less than the amount mentioned in
the following schedule (or any other higher amount as may be prescribed
by the State Government), if the case is before a Court other than the
Supreme Court, and less than Rs 5 Lakh, if the case is before the
Supreme Court.
• The Income Ceiling Limit prescribed u/S 12(h) of the Act for availing free legal
services in different States has been stated here https://nalsa.gov.in/faqs
• S2: Limit for Transgender – Rs. 2,00,000 see https://nalsa.gov.in/faqs
• S4: Senior citizens’ eligibility for free legal aid depends on the Rules framed by
the respective State Governments in this regard.
• In Delhi for example, senior citizens are eligible for free legal aid subject to
prescribed ceiling of annual income. Any individual above the age of 60 can
apply for free legal aid/services.
Refer: https://www.insightsonindia.com/2022/07/19/tele-law-service-is-being-made-free-
of-cost-for-citizens-from-this-year-minister-of-law-and-justice/
154. Consider the following statements regarding National Legal Services Authority
(NALSA).
1. The National Legal Services Authority (NALSA) has been constituted under the Legal
Services Authorities Act, 1987 to provide free Legal Services to the weaker sections
of the society.
2. The President of India is the Patron-in-Chief of the Authority.
Which of the above statements is/are correct?
(a) 1 only
(b) 2 only
(c) Both 1 and 2
(d) Neither 1 nor 2
Ans: (a)
Explanation:
• The National Legal Services Authority (NALSA) has been constituted under the
Legal Services Authorities Act, 1987 to provide free Legal Services to the weaker
sections of the society and to organize Lok Adalats for amicable settlement of
disputes.
• The Chief Justice of India is patron-in-chief of NALSA while second senior most
judge of Supreme Court of India is the Executive-Chairman.
• In every State, State Legal Services Authority has been constituted to give effect
to the policies and directions of the NALSA and to give free legal services to the
people and conduct Lok Adalats in the State.
Refer: https://www.insightsonindia.com/2022/07/19/tele-law-service-is-being-made-free-
of-cost-for-citizens-from-this-year-minister-of-law-and-justice/

Telegram: https://t.me/insightsIAStips
79
Youtube: https://www.youtube.com/channel/UCpoccbCX9GEIwaiIe4HLjwA
Revision Through MCQs (RTM) Compilation (July - 2022)

155. Consider the following statements:


1. In India, Generic Drugs were defined under the Drugs & Cosmetics Act, 1940 and
Rules, 1945.
2. Drugs manufactured in India, irrespective of whether they are generic or branded,
are required to comply with the same standards as prescribed in the Drugs and
Cosmetics Act, 1940 and Rules, 1945 made thereunder for their quality.
Which of the given above statements is/are correct?
(a) 1 Only
(b) 2 Only
(c) Both 1 and 2
(d) Neither 1 nor 2
Ans: (b)
Explanation:
• Stat1: There is no definition of generic or branded medicines under the Drugs &
Cosmetics Act, 1940 and Rules, 1945 made thereunder. However, generic
medicines are generally those which contain same amount of same active
ingredient(s) in same dosage form and are intended to be administered by the
same route of administration as that of branded medicine.
• Stat2: Drugs manufactured in the country, irrespective of whether they are
generic or branded, are required to comply with the same standards as
prescribed in the Drugs and Cosmetics Act, 1940 and Rules, 1945 made
thereunder for their quality.
Refer: https://www.insightsonindia.com/2022/07/19/what-the-draft-medical-devices-bill-
says-and-what-it-does-not/
156. With respect to Sasakawa Award 2022, consider the following statements:
1. It is the UN's highest environmental honour.
2. It was established in 2005 by United Nations Environment Programme (UN
Environment).
Which of the statement(s) given above is/are correct?
(a) 1 only
(b) 2 only
(c) Both 1 and 2
(d) Neither 1 nor 2
Ans: (d)
Explanation:
• S1: The annual Champions of the Earth Award is the UN's highest
environmental honour. It recognizes outstanding leaders from government, civil
society, and the private sector
• S2: The United Nations Sasakawa Award for Disaster Risk Reduction is a global
award established in 1986 by the founding Chairman of the Nippon
Foundation, Mr Ryoichi Sasakawa for recognising excellence in reducing
disaster risk.
o SEEDS has been working with the communities across South Asia over
for the last 28 years.
Refer: https://www.insightsonindia.com/2022/07/19/risk-reduction-strategies-using-
traditional-knowledge/

Telegram: https://t.me/insightsIAStips
80
Youtube: https://www.youtube.com/channel/UCpoccbCX9GEIwaiIe4HLjwA
Revision Through MCQs (RTM) Compilation (July - 2022)

157. In which of the following relief sculpture inscriptions is ‘Ranyo Ashoka’ (King Ashoka)
mentioned along with the stone portrait of Ashoka?
(a) Kanganahalli
(b) Sanchi I
(c) Shahbazgarhi
(d) Sohgaura
Ans: (a)
Explanation:
• During the excavations (2000 to 2002) at Kanganahalli, the most important
finding of the excavation include a stone sculptured slab bearing the name Raya
Ashoka. This was the first inscribed portrait of Ashoka (surrounded by female
attendants and queens) that was unearthed from the ruined Buddhist stupa.
Refer: facts for prelims: https://www.insightsonindia.com/2022/07/19/mission-2023-
insights-daily-current-affairs-pib-summary-19-july-2022/
158. Consider the following statements about Bonalu Festival:
1. It is one of the biggest festivals of Telangana.
2. It is celebrated in the month of Ashada.
Which of the statement(s) given above is/are correct?
(a) 1 only
(b) 2 only
(c) Both 1 and 2
(d) Neither 1 nor 2
Ans: (c)
Explanation:
• The festival is dedicated to Goddess Mahakali, thanking her for fulfilling the
wishes of her devotees.
• Bonalu is a popular festival celebrated in Hyderabad and Secunderabad.
• Bonalu is celebrated in the month of Ashada which, according to the modern
calendar, falls around July-August.
Refer: facts for prelims: https://www.insightsonindia.com/2022/07/19/mission-2023-
insights-daily-current-affairs-pib-summary-19-july-2022/
159. Consider the following statements:
1. All crypto-currencies are based on Blockchain technology.
2. Altcoins refer to a class of alternative crypto-currencies that were launched after the
success of Bitcoin.
Which of the above statements is/are incorrect?
(a) 1 only
(b) 2 only
(c) Both 1 and 2
(d) Neither 1 nor 2
Ans: (a)
Explanation:
• Not all crypto-currencies are based on Blockchain technology. For instance,
‘IOTA’ is one such crypto-currency. Instead of a blockchain, IOTA uses a
mathematical ‘tangle’. Altcoins are the alternative cryptocurrencies launched
after the success of Bitcoin. Generally, they project themselves as better
substitutes to Bitcoin.

Telegram: https://t.me/insightsIAStips
81
Youtube: https://www.youtube.com/channel/UCpoccbCX9GEIwaiIe4HLjwA
Revision Through MCQs (RTM) Compilation (July - 2022)

Refer: facts for prelims: https://www.insightsonindia.com/2022/07/19/mission-2023-


insights-daily-current-affairs-pib-summary-19-july-2022/
160. With reference to the Blockchain Technology, consider the following statements:
1. It is a public ledger that everyone can inspect, which no single user controls
2. The structure and design of blockchain is such that all the data in it are about
cryptocurrency only
3. Applications that depend on the basic features of blockchain can be developed
without anybody’s permission
Which of the statements given above is/are correct?
(a) 1 only
(b) 1 and 2 only
(c) 2 only
(d) 1 and 3 only
Ans: (d)
Explanation:
• A blockchain is a decentralized, distributed, and oftentimes public, digital ledger
consisting of records called blocks that is used to record transactions across
many computers so that any involved block cannot be altered retroactively,
without the alteration of all subsequent blocks.
• This allows the participants to verify and audit transactions independently and
relatively inexpensively. A blockchain database is managed autonomously
using a peer-to-peer network and a distributed timestamping server. They
are authenticated by mass collaboration powered by collective self-
interests. Such a design facilitates robust workflow where participants’
uncertainty regarding data security is marginal.
• The great advantage to an open, permissionless, or public, blockchain network is
that guarding against bad actors is not required and no access control is
needed. This means that applications can be added to the network without
the approval or trust of others, using the blockchain as a transport layer.
• S2 is clearly wrong, Blockchain can be used for a variety of purposes of which
cryptocurrency is only one of them.
Refer: UPSC CSE 2019

RTM- REVISION THROUGH MCQS –20th-Jul-2022

161. Which of the following is/are the demerits of the Parliamentary System?
1. Government by Amateurs
2. Not conductive to administrative efficiency
3. Separation of Powers
Select the correct answer code:
(a) 1 only
(b) 2 and 3 only
(c) 1 and 2 only
(d) 1, 2 and 3
Ans: (c)
Explanation:
• Government by Amateurs: the parliamentary system is not conductive to
administrative efficiency as the ministers are not experts in their fields. The

Telegram: https://t.me/insightsIAStips
82
Youtube: https://www.youtube.com/channel/UCpoccbCX9GEIwaiIe4HLjwA
Revision Through MCQs (RTM) Compilation (July - 2022)

Prime Minister has a limited choice in the selection of ministers; his choice is
restricted to the members of Parliament alone and does not extend to external
talent.
• In the parliamentary system, the legislature and the executive are together and
inseparable. The cabinet acts as the leader of legislature as well as the executive.
Refer: https://www.insightsonindia.com/2022/07/20/inner-party-democracy/
162. Consider the following statements about the Competition Commission of India (CCI):
1. It acts as the competition regulator in India.
2. It is working under the Ministry of Commerce and Industry.
Which of the statements given above is/are correct?
(a) 1 only
(b) 2 only
(c) Both 1 and 2
(d) Neither 1 nor 2
Ans: (a)
Explanation:
• The Competition Commission of India (CCI) is a statutory and quasi-judicial
body working under the Ministry of Corporate Affairs. It was established under
the Competition Act, 2002 for the administration, implementation and
enforcement of the Act, and was duly constituted in March 2009.
Refer: https://www.insightsonindia.com/2022/07/20/empowering-cci-for-negotiated-case-
settlements/
163. In the context of India, which of the following factors is/are contributor/ contributors
to reducing the risk of a currency crisis?
1. The foreign currency earnings of India’s IT sector
2. Increasing the government expenditure
3. Remittances from Indians abroad
Select the correct answer using the code given below.
(a) 1 only
(b) 1 and 3 only
(c) 2 only
(d) 1, 2 and 3
Ans: (b)
Explanation:
• A currency crisis results in a sharp depreciation of domestic currency and may
force the authorities to sell foreign exchange reserves and take measures (for e.g.
raise domestic interest rates) to defend the downfall in currency.
• S1: If a country has a strong export performance, a currency crisis is difficulty to
occur. Since S1 leads to greater exports, it will reduce the risk.
• S2: Increase in government expenditure may actually increase imports and
adversely affect the currency crisis. Usually, austerity in public spending is one
of the measures prescribed to contain a foreign exchange or currency crisis.
• S3: Greater flow of remittances would help avert the currency crisis since greater
inflow of foreign currency (which is converted into rupees by the recipient of the
remittance in India) raises the demand for domestic currency.
• Decline in the value of a country’s currency is known as Currency Crisis that
negatively affects the economy by creating instabilities in currency exchange
rates. The foreign currency earnings of India’s Information Technology (IT) sector

Telegram: https://t.me/insightsIAStips
83
Youtube: https://www.youtube.com/channel/UCpoccbCX9GEIwaiIe4HLjwA
Revision Through MCQs (RTM) Compilation (July - 2022)

and remittances from Indians abroad together can reduce the risk of a currency
crisis in India.
Refer: https://www.insightsonindia.com/2022/07/20/explained-what-rs-80-to-a-dollar-
means/
164. Consider the following statements
As per the Environment Impact Assessment Notification 2020,
1. It shortens the period of public consultation hearings to a maximum of 40 days.
2. It allows the declaration of some areas as “economically sensitive areas” without a
public hearing.
Which of the given above statements is/are correct?
(a) 1 only
(b) 2 only
(c) Both 1 and 2
(d) Neither 1 nor 2
Ans: (c)
Explanation:
• S1: It shortens the period of public consultation hearings to a maximum of 40
days.
o It reduces from 30 to 20 days the time provided for the public to submit
their responses during a public hearing for any application seeking
environmental clearance.
• S2: It also allows the declaration of some areas as “economically sensitive areas”
without a public hearing or environmental clearance, and several “red” and
“orange”-classified toxic industries could now operate as close as 0-5 km from a
Protected Area in “callous disregard” for forests. • The increased validity of the
environment clearances for mining projects (50 years versus 30 years currently)
and river valley projects (15 years versus 10 years currently) raises the risk of
irreversible environmental, social and health consequences on account of the
project remaining unnoticed for long.
Refer: https://www.insightsonindia.com/2022/07/20/amendment-of-eia-rules/
165. Consider the following statements:
1. The guidelines for renunciation of Indian citizenship is issued by the Union Ministry
of Home Affairs (MHA).
2. Every minor child of the person who ceases to be a citizen of India shall also cease
to be a citizen of India.
Which of the above statements is/are correct?
(a) 1 only
(b) 2 only
(c) Both 1 and 2
(d) Neither 1 nor 2
Ans: (c)
Explanation:
• The citizenship act, of 1955 prescribes three ways of losing citizenship:
o By renunciation: Any citizen of India of full age and capacity can make a
declaration renouncing Indian citizenship. Such declaration may not be
accepted during the war. Even the minor children of the person who
renounces citizenship stand to lose their Indian citizenship. However,

Telegram: https://t.me/insightsIAStips
84
Youtube: https://www.youtube.com/channel/UCpoccbCX9GEIwaiIe4HLjwA
Revision Through MCQs (RTM) Compilation (July - 2022)

when their children attain the age of eighteen, they may resume Indian
citizenship
o By termination: If a citizen of India voluntarily acquires the citizenship of
another country, then he loses the citizenship of India
o By deprivation: Compulsory termination of Indian citizenship by the
Central government, if the person obtained the citizenship by fraud or
shown disloyalty to the Constitution of India or imprisoned for a term of
two years (in the last 5 years after naturalization) or citizen has been
ordinarily resident out of India for a period of 7 years
• The guidelines said that when a person ceases to be a citizen of India under
Section 8(1) of Citizenship Act, 1955, “every minor child of that person shall
thereupon ceases to be a citizen of India”. The minor child may, however, within
one year of attaining full age apply to resume Indian citizenship. The guidelines
are not clear if minors would also lose citizenship if only one of the parents gives
up her/his Indian citizenship.
Refer: facts for prelims: https://www.insightsonindia.com/2022/07/20/mission-2023-
insights-daily-current-affairs-pib-summary-20-july-2022/
166. As per RBI, which of the following has become the top remittance source after UAE (in
2020-21) for India?
(a) Australia
(b) Canada
(c) The European Union
(d) The United States of America
Ans: (d)
Explanation:
• As per RBI, the US has become the top remittance source after UAE (in
2020-21) for India.
• Reasons: Economic recovery in the US is one of the important drivers of
India’s remittances growth as it accounts for 23% of total remittances received
in India.
o India received about $87bn in remittances in FY21 (the world’s highest).
o Decline in remittances from GCC (Gulf): The share of remittances from
the GCC region in India’s inward remittances is estimated to have
declined from more than 50 per cent in 2016-17 (last surveyed period) to
about 30 per cent in 2020- 21.
o Remittances have exceeded foreign direct inflows in several countries,
including India.
Refer: facts for prelims: https://www.insightsonindia.com/2022/07/20/mission-2023-
insights-daily-current-affairs-pib-summary-20-july-2022/
167. Consider the following statements about National Statistical Office (NSO):
1. It is the statistical wing of the Ministry of Statistics and Programme Implementation
(MoSPI).
2. It releases the quarter gross value added (GVA) and gross domestic product (GDP)
numbers.
Which of the above statements is/are correct?
(a) 1 only
(b) 2 only
(c) Both 1 and 2

Telegram: https://t.me/insightsIAStips
85
Youtube: https://www.youtube.com/channel/UCpoccbCX9GEIwaiIe4HLjwA
Revision Through MCQs (RTM) Compilation (July - 2022)

(d) Neither 1 nor 2


Ans: (c)
Explanation:
• It is an overarching body formed by merging the National Sample Survey Office
(NSSO), Computer Centre and Central Statistical Office (CSO).
• NSO was first envisaged by Rangarajan Commission to implement and
maintain statistical standards and coordinate statistical activities of Central and
State agencies as laid down by the National Statistical Commission (NSC).
• Ministry: It is the statistical wing of the Ministry of Statistics and
Programme Implementation (MoSPI).
• The National Statistical Office (NSO) releases the quarter gross value added
(GVA) and gross domestic product (GDP) numbers.
Refer: facts for prelims: https://www.insightsonindia.com/2022/07/20/mission-2023-
insights-daily-current-affairs-pib-summary-20-july-2022/
168. In the context of India which one of the following is the characteristic appropriate for
Bureaucracy?
(a) An agency for widening the scope for parliamentary democracy
(b) An agency for strengthening the structure of federalism
(c) An agency for facilitating the political stability and economic growth
(d) An agency for the implementation of public policy
Ans: (d)
Explanation:
• Bureaucracy refers to an administrative policy-implementation group, which is a
body of non-elected government officials.
Refer: facts for prelims: https://www.insightsonindia.com/2022/07/20/mission-2023-
insights-daily-current-affairs-pib-summary-20-july-2022/
169. Mission Karmayogi was in news recently, is related to which of the following?
(a) Agriculture Reforms
(b) Women empowerment
(c) Socio-Economic empowerment of tribal people
(d) Reform in Indian Bureaucracy
Ans: (d)
Explanation: About Mission Karmayogi:
• The ‘National Programme for Civil Services Capacity Building – Mission
Karmayogi’ was launched to effect a transformational shift from rule based
training to role-based capacity development of all civil services in the country so
as to enhance citizen experience for government services and improve availability
of competent workforce.
Refer: facts for prelims: https://www.insightsonindia.com/2022/07/20/mission-2023-
insights-daily-current-affairs-pib-summary-20-july-2022/
170. The “SPRINT Challenges” was in news recently, is related to which of the following?
(a) Good governance
(b) Sports reforms
(c) Waste management
(d) None of the above
Ans: (d)
Explanation:

Telegram: https://t.me/insightsIAStips
86
Youtube: https://www.youtube.com/channel/UCpoccbCX9GEIwaiIe4HLjwA
Revision Through MCQs (RTM) Compilation (July - 2022)

• PM unveiled ‘SPRINT challenges’, an initiative aimed at boosting the usage of


indigenous technology in the Indian Navy.
Refer: facts for prelims: https://www.insightsonindia.com/2022/07/20/mission-2023-
insights-daily-current-affairs-pib-summary-20-july-2022/

RTM- REVISION THROUGH MCQS –21th-Jul-2022

171. Consider the following statements about the Jal Jeevan Mission:
1. It envisages a supply of 55 litres of water per person per day to every rural
household through Functional Household Tap Connections (FHTC) by 2030.
2. It is under the Ministry of Jal Shakti.
Which of the statements given above is/are correct?
(a) 1 only
(b) 2 only
(c) Both 1 and 2
(d) Neither 1 nor 2
Ans: (b)
Explanation:
• JJM envisages a supply of 55 litres of water per person per day to every rural
household through Functional Household Tap Connections (FHTC) by 2024.
• It is under the Ministry of Jal Shakti.
• It was launched in 2019.
• Funding is demand-driven.
• The mission is very strongly focused on Jan bhagidari, or community
engagement which is seen as a very important pillar of the project.
Refer: https://www.insightsonindia.com/2022/07/21/jal-jeevan-mission-jjm-has-hit-51-
mark/
172. Consider the following statements regarding the Central Bank Digital Currency
(CBDC):
1. A Central Bank Digital Currency (CBDC) is the legal tender issued by a central bank
in a digital form.
2. It is the same as a fiat currency and is exchangeable one-to-one with the fiat
currency.
Which of the statements given above is/are correct?
(a) 1 only
(b) 2 only
(c) Both 1 and 2
(d) Neither 1 nor 2
Ans: (c)
Explanation:
• Context: RBI is in the process of implementing the CBDC in a phased manner
for wholesale and retail segments.
• A CBDC is the legal tender issued by a central bank in a digital form. It is the
same as a fiat currency and is exchangeable one-to-one with the fiat currency.
Only its form is different.
Refer: https://www.insightsonindia.com/2022/07/21/central-bank-digital-currency/

Telegram: https://t.me/insightsIAStips
87
Youtube: https://www.youtube.com/channel/UCpoccbCX9GEIwaiIe4HLjwA
Revision Through MCQs (RTM) Compilation (July - 2022)

173. With reference to Ibrahim Adil Shah II, which one of the following statements is not
correct?
(a) He belonged to the Sunni Islam sect but was broadminded in his religious views and
practices.
(b) He issued a coin, the Nun-i-Nauras.
(c) He laid the foundation for a new capital, Navraspur, near Bijapur.
(d) He was killed at a truce negotiation meeting with Chhatrapati Shivaji Maharaj.
Ans : (d)
Explanation:
• Ibrahim Adil Shah II:
o Among the Bijapur Sultans of the Adil Shahi dynasty (1490–
1686), Ibrahim Adil Shah II (r. 1571–1627) stands out as a poet,
musician and painter, besides being a talented ruler and patron of
art.
o Ibrahim belonged to the Sunni Islam sect but was broadminded in
his religious views and practices. He was devoted to Prophet
Muhammad, Hindu deities Saraswati and Ganapati, and Sufi saint
Sayyid Muhammad Gisu Daraz (1321–1422) of Gulbarga.
o Ibrahim was well aware of the Indian aesthetic concept of
the Rasa (essence), In 1599, he laid the foundation for a new
capital, Navraspur, near Bijapur.
o He also issued a coin, the Nun-i-Nauras.
o Ibrahim penned 59 songs and 17 couplets—in Kitab-i-Nauras (Nauras
Nama)—in Deccani, which later developed into Urdu.
• Afzal Khan (general) was killed at a truce negotiation meeting with Shivaji, and
his army was defeated at the Battle of Pratapgad.
Refer: Facts For Prelims: https://www.insightsonindia.com/2022/07/21/mission-2023-
insights-daily-current-affairs-pib-summary-21-july-2022/
174. International Labour Organisation, Conventions 138 and 182 are related to
(a) Child labour
(b) Adaptation of agricultural practices to global climate change
(c) Regulation of food prices and food security
(d) Gender parity at the workplace
Ans: (a)
Explanation:
• International commitment: India has ratified the International Labour
Organizations Convention (ILO) no 138 (minimum age for employment)
and Convention no 182(worst forms of child labour).
Refer: Facts For Prelims: https://www.insightsonindia.com/2022/07/21/mission-2023-
insights-daily-current-affairs-pib-summary-21-july-2022/
175. Consider the following statements regarding landlord port model.
1. In the landlord port model, the publicly governed port authority acts as a regulatory
body that carry out port operations—mainly cargo handling activities.
2. Here, the port authority maintains ownership of the port.
Which of the above statements is/are correct?
(a) 1 only
(b) 2 only
(c) Both 1 and 2

Telegram: https://t.me/insightsIAStips
88
Youtube: https://www.youtube.com/channel/UCpoccbCX9GEIwaiIe4HLjwA
Revision Through MCQs (RTM) Compilation (July - 2022)

(d) Neither 1 nor 2


Ans: (b)
Explanation:
• In the landlord port model, the publicly governed port authority acts as a
regulatory body and as landlord while private companies carry out port
operations—mainly cargo-handling activities.
• Here, the port authority maintains ownership of the port while the infrastructure
is leased to private firms that provide and maintain their own superstructure
and install own equipment to handle cargo.
• In return, the landlord port gets a share of the revenue from the private entity.
• The role of the landlord port authority would be to carry out all public sector
services and operations such as the award of bids for cargo terminals and
dredging
Refer: Facts For Prelims: https://www.insightsonindia.com/2022/07/21/mission-2023-
insights-daily-current-affairs-pib-summary-21-july-2022/
176. Consider the following statements:
1. In the case of all cereals, pulses and oil-seeds, the procurement at Minimum
Support Prices (MSP) is unlimited in any State/UT of India.
2. In the case of cereals and pulses, the MSP is fixed in any State/UT at a level to
which the market price will never rise.
Which of the statements given above is/are correct?
(a) 1 only
(b) 2 only
(c) Both 1 and 2
(d) Neither 1 nor 2
Ans: (d)
Explanation:
• S1: While procurement is open-ended, it is not unlimited since our buffer stock
is limited.
• S2: This is absurd. MSP is only an indicative price, and Market price can rise
way above or way below MSP given the situation.
Refer: Facts For Prelims: https://www.insightsonindia.com/2022/07/21/mission-2023-
insights-daily-current-affairs-pib-summary-21-july-2022/
177. With reference to India’s Kuno National Park, consider the following statements:
1. It was selected as a possible site to implement the Asiatic Lion Reintroduction
Project.
2. It is part of the Khathiar-Gir dry deciduous forests ecoregion.
Which of the statements given above is/are correct?
(a) 1 only
(b) 2 only
(c) Both 1 and 2
(d) Neither 1 nor 2
Ans: (c)
Explanation:
• Kuno is a national park in Madhya Pradesh, India.
• It is part of the Khathiar-Gir dry deciduous forests ecoregion.

Telegram: https://t.me/insightsIAStips
89
Youtube: https://www.youtube.com/channel/UCpoccbCX9GEIwaiIe4HLjwA
Revision Through MCQs (RTM) Compilation (July - 2022)

• In the 1990s, it was selected as a possible site to implement the Asiatic Lion
Reintroduction Project, which aimed at establishing a second lion population in
India.
• India has plans to reintroduce cheetahs at the Kuno National Park in Sheopur
and Morena districts of Madhya Pradesh’s Gwalior-Chambal region.
Refer: Facts For Prelims: https://www.insightsonindia.com/2022/07/21/mission-2023-
insights-daily-current-affairs-pib-summary-21-july-2022/
178. In which one of the following States is Kuno-Palpur National Park is located?
(a) Gujarat
(b) Rajasthan
(c) Madhya Pradesh
(d) Maharashtra
Ans: (c)
Explanation:
• Kuno is a national park in Madhya Pradesh, India.
• It is part of the Khathiar-Gir dry deciduous forests ecoregion
Refer: Facts For Prelims: https://www.insightsonindia.com/2022/07/21/mission-2023-
insights-daily-current-affairs-pib-summary-21-july-2022/
179. Consider the following statements:
1. The cheetah is the world’s fastest land mammal.
2. It is listed as endangered in IUCN red listed species.
Which of the statements given above is/are correct?
(a) 1 only
(b) 2 only
(c) Both 1 and 2
(d) Neither 1 nor 2
Ans: (a)
Explanation:
• The cheetah is also the world’s fastest land mammal.
• It is listed as vulnerable in IUCN red listed species.
• The country’s last spotted feline died in Chhattisgarh in 1947. Later, the cheetah
— which is the fastest land animal — was declared extinct in India in 1952.
• The Asiatic cheetah is classified as a “critically endangered” species by the
IUCN Red List,and is believed to survive only in Iran.
Refer: Facts For Prelims: https://www.insightsonindia.com/2022/07/21/mission-2023-
insights-daily-current-affairs-pib-summary-21-july-2022/
180. Due to some reasons, if photosynthesis in plants suddenly stopped happening, what
could be its likely consequence/consequences?
1. most plants would die within short order
2. green algal blooms would cover the entire face of the planet
3. global emissions of CO2 would be greatly reduced
Select the correct answer using the code given below:
(a) 1 only
(b) 2 and 3 only
(c) 1 and 2 only
(d) 1, 2 and 3
Ans: (a)
Explanation:

Telegram: https://t.me/insightsIAStips
90
Youtube: https://www.youtube.com/channel/UCpoccbCX9GEIwaiIe4HLjwA
Revision Through MCQs (RTM) Compilation (July - 2022)

• If photosynthesis came to an abrupt end, most plants would die within short
order. Although they could hold out for a few days — or in some cases, a few
weeks — how long they lived would largely be a factor of how much sugar they
had stored within their cells.
• The only organisms able to exist under such conditions would be the
chemosynthetic bacteria, which can utilize the chemical energy of certain
inorganic compounds and thus are not dependent on the conversion of light
energy.
• S2 and S3 are absurd statements.
Refer: https://www.insightsonindia.com/2021/03/19/artificial-photosynthesis-to-provide-
solutions-for-carbon-capture-and-conversion/

RTM- REVISION THROUGH MCQS –22nd-Jul-2022

181. Consider the following statements regarding National Commission for Backward
Classes (NCBC).
1. National Commission for Backward Classes is a non-constitutional body under the
Ministry of Social Justice and Empowerment.
2. The commission was the outcome of Indra Sawhney & Ors. Vs. Union of India.
3. The commission have the same powers as a Civil Court.
Which of the above statements is/are correct?
(a) 1 and 2 only
(b) 2 and 3 only
(c) 1 and 3 only
(d) 1, 2 and 3
Ans: (b)
Explanation:
• National Commission for Backward Classes is a constitutional body (123rd
constitutional amendment bill 2017 and 102nd amendment 2018 in
constitution to make it constitutional body) (Article 338B of the Indian
Constitution) under Ministry of Social Justice and Empowerment established on
14 August 1993.
• It was constituted pursuant to the provisions of the National Commission for
Backward Classes Act, 1993. The commission was the outcome of Indra
Sawhney & Ors. Vs. Union of India.
• The National Commission for Backward Classes, National Commission for
Scheduled Castes as well as National Commission for Scheduled Tribes have the
same powers as a Civil Court.
Refer: https://www.insightsonindia.com/2022/07/22/reservation-in-ulb/
182. The 102nd Constitutional Amendment Act, sometimes seen in news is related to
(a) 10% reservation for Economically weaker sections
(b) Constitutional status to National Commission for Backward Classes
(c) Extend reservation for Scheduled Castes (SCs) and Scheduled Tribes (STs) to Lok
Sabha and legislative bodies
(d) Abolition of Legislative Councils in certain States
Ans: (b)
Explanation:

Telegram: https://t.me/insightsIAStips
91
Youtube: https://www.youtube.com/channel/UCpoccbCX9GEIwaiIe4HLjwA
Revision Through MCQs (RTM) Compilation (July - 2022)

• The One Hundred and Second Amendment of the Constitution of India, officially
known as the Constitution (One Hundred and Second Amendment) Act, 2018,
granted constitutional status to the National Commission for Backward Classes
(NCBC).
Refer: https://www.insightsonindia.com/2022/07/22/reservation-in-ulb/
183. Which of the following statements is / are correct regarding the Medical Termination
of Pregnancy (MTP) Amendment Act, 2021?
1. It extends the upper limit for medical termination of pregnancy to 24 weeks from 20
weeks, for certain categories of women.
2. The opinion of a registered medical practitioner, eligible as per the rules is required
for the termination of pregnancy at different gestation ages up to 20 weeks.
3. The opinion for medical termination of pregnancy beyond twenty weeks gestation
period will be given by a medical board duly constituted by the respective state
government or UT administration at approved facilities.
Select the correct answer using the code given below.
(a) 1 and 2 only
(b) 2 only
(c) 3 only
(d) 1, 2 and 3
Ans: (a)
Explanation: The Medical Termination of Pregnancy (Amendment) Act, 2021
• The Act amends the 1971 MTP law which regulates the conditions under which
medical termination of pregnancy is to be pursued. The amended MTP Act extends
the upper limit for medical termination of pregnancy to 24 weeks from 20
weeks, for certain categories of women.
• S2: The opinion of a registered medical practitioner, eligible as per the rules is
required for the termination of pregnancy at different gestation ages up to 20
weeks. Opinion of two registered medical practitioners is required for
termination of pregnancy beyond twenty weeks till twenty-four weeks of
gestation period.
• S3: The opinion for medical termination of pregnancy beyond twenty-four
weeks gestation period will be given by a medical board duly constituted by the
respective state government or UT administration at approved facilities. Two
registered medical practitioners eligible as per the MTP rules will perform the
termination of pregnancy based on the decision of the medical board.
• The rules stipulate that the medical board will have the power to allow or deny
termination of pregnancy beyond twenty-four weeks of gestation period only after
due consideration and ensuring that the procedure would be safe for the woman at
that gestation age and whether the foetal malformation has substantial risk of it
being incompatible with life or if the child is born it may suffer from serious physical
or mental abnormalities.
Refer: https://www.insightsonindia.com/2022/07/22/recent-issues-related-to-indias-law-
on-abortion/
184. Who is known as the 'father of genetics'?
(a) Charles Darwin
(b) Friedrich Miescher
(c) James Watson
(d) Gregor Mendel

Telegram: https://t.me/insightsIAStips
92
Youtube: https://www.youtube.com/channel/UCpoccbCX9GEIwaiIe4HLjwA
Revision Through MCQs (RTM) Compilation (July - 2022)

Ans: (d)
Explanation:
• Genetics: Genetics is the study of genes, genetic variation, and heredity in
organisms.
• It was first experimentally established by Gregor Mendel (a monk) (1822 to
1884)
• Johann Gregor Mendel is considered the father of genetics.
Refer: https://www.insightsonindia.com/2022/07/22/science-of-genetics/
185. Consider the following statements regarding Vesara style of temple architecture.
1. The Vesara style is described as the ‘Central Indian temple architecture style’.
2. The trend was started by the Chalukyas of Badami who built temples in this style.
3. It lost prominence during Rashtrakutas and Hoysalas, who built temples mainly in
Dravidian style.
Which of the above statements is/are correct?
(a) 1 only
(b) 1 and 2 only
(c) 2 and 3 only
(d) 1, 2 and 3
Ans: (b)
Explanation:
• The Vesara style contain elements of both Dravida and Nagara styles.
• The Vesara style is also described in some texts as the ‘Central Indian
temple architecture style’ or ‘Deccan architecture’.
• The trend was started by the Chalukyas of Badami (500-753AD) who built
temples in a style that was essentially a mixture of the nagara and the dravida
styles, further refined by the Rashtrakutas of Manyakheta (750-983AD) in
Ellora, Chalukyas of Kalyani (983-1195 AD) in Lakkundi, Dambal, Gadag etc.
and epitomized by the Hoysala empire.


Refer: facts for prelims: https://www.insightsonindia.com/2022/07/22/mission-2023-
insights-daily-current-affairs-pib-summary-22-july-2022/
186. Consider the following statements with reference to Narcotic Drugs and Psychotropic
Substances Act, 1985:
1. Under the NDPS Act, it is illegal for a person to produce/manufacture/cultivate,
possess, sell, purchase, transport, store, and/or consume any narcotic drug or
psychotropic substance.
2. Under one of the provisions of the act, the State Governments are empowered to
regulate the cultivation, production, manufacture, import, export, sale,
consumption, use of narcotic drugs and psychotropic substances.
Which of the given above statements is/are correct?
(a) 1 only

Telegram: https://t.me/insightsIAStips
93
Youtube: https://www.youtube.com/channel/UCpoccbCX9GEIwaiIe4HLjwA
Revision Through MCQs (RTM) Compilation (July - 2022)

(b) 2 only
(c) Both 1 and 2
(d) Neither 1 nor 2
Ans: (a)
Explanation: Narcotic Drugs and Psychotropic Substances Act, 1985:
• The Act is designed to fulfill India’s treaty obligations under the Single
Convention on Narcotic Drugs, Convention on Psychotropic Substances,
and United Nations Convention Against Illicit Traffic in Narcotic Drugs and
Psychotropic Substances.
• The Act 1985 sets out the statutory framework for drug law enforcement in
India.
• Under the Act, the cultivation, production, manufacture, possession, sale,
purchase, transportation, warehousing, consumption, inter-State movement,
transshipment and import and export of narcotic drugs and psychotropic
substances is prohibited, except for medical or scientific purposes and in
accordance with the terms and conditions of any license, permit or authorization
given by the Government.
• The Central Government is empowered to regulate the cultivation,
production, manufacture, import, export, sale, consumption, use etc of
narcotic drugs and psychotropic substances.
• State Governments are empowered to permit and regulate possession and inter-
State movement of opium, poppy straw, the manufacture of medicinal opium
and the cultivation of cannabis excluding hashish.
• The Central Government is empowered to declare any substance, based on an
assessment of its likely use in the manufacture of narcotics drugs and
psychotropic substances as a controlled substance.
• Both the Central Government and State Governments are empowered to appoint
officers for the purposes of the Act.
Refer: facts for prelims: https://www.insightsonindia.com/2022/07/22/mission-2023-
insights-daily-current-affairs-pib-summary-22-july-2022/
187. India Innovation Index 2022 is released by which of the following?
(a) National Innovation Foundation
(b) NITI Aayog
(c) Reserve Bank of India
(d) Ministry of Education
Ans: (b)
Explanation:
• NITI Aayog’s India Innovation Index, 2022
• The index determines innovation capacities and ecosystems at the sub-national
level.
• Key outcomes:
o Karnataka topped the ranking, followed by Telangana, Haryana,
Maharashtra and Tamil Nadu.
o Manipur secured the lead in the Northeast and Hill States category
o Chandigarh was the top performer in the Union Territories and the City
States category.
o Bottom rank: Chhattisgarh, Odisha, Bihar and Gujarat were at the
bottom of the index.

Telegram: https://t.me/insightsIAStips
94
Youtube: https://www.youtube.com/channel/UCpoccbCX9GEIwaiIe4HLjwA
Revision Through MCQs (RTM) Compilation (July - 2022)

Refer: facts for prelims: https://www.insightsonindia.com/2022/07/22/mission-2023-


insights-daily-current-affairs-pib-summary-22-july-2022/
188. The World Press Freedom Index is published by
(a) International Press Institute
(b) Reporters Without Borders
(c) International Center for Journalists
(d) International Federation of Journalists
Ans: (b)
Explanation:
• The 2022 World Press Freedom Index (20th edition)was recently published
by Reporters Without Borders.
• It was released on the World Press Freedom Day (WPFD) (3rd May).
• The report points to an overall two-fold increase in media polarisationcreating
divisions within countries, and between countries at the international level.
Refer: facts for prelims: https://www.insightsonindia.com/2022/07/22/mission-2023-
insights-daily-current-affairs-pib-summary-22-july-2022/
189. Consider the following Rivers:
1. Dibang
2. Lohit
3. Kameng
4. Manas
Which of the above are right bank tributaries of Brahmaputra?
(a) 1, 2 and 3 only
(b) 2, 3 and 4 only
(c) 1 and 2 only
(d) 3 and 4 only
Ans: (d)
Explanation: Tributaries of Brahmaputra River
• Left: Dibang River, called by Dimasa tribe , Lohit River, Dhansiri River, Kolong
River
• Right: Kameng River, Manas River, Beki River, Raidak River, Jaldhaka River,
Teesta River, Subansiri River


Refer: Major Rivers and Tributaries

Telegram: https://t.me/insightsIAStips
95
Youtube: https://www.youtube.com/channel/UCpoccbCX9GEIwaiIe4HLjwA
Revision Through MCQs (RTM) Compilation (July - 2022)

190. Among the following cities, which one lies on a longitude closest to that of Dehradun?
(a) Jaipur
(b) Bhopal
(c) Raipur
(d) Pune
Ans: (b)
Explanation:


Refer: INDIA MAP

RTM- REVISION THROUGH MCQS –23rd-Jul-2022

191. Consider the following statements about the law commission of India:
1. It works as an advisory body to the Supreme Court of India.
2. It is established for a fixed tenure.
3. It comprises legal experts appointed by the Central Government.
Which of the statements given above is/are correct?
(a) 1 and 2 only
(b) 1 and 3 only
(c) 2 and 3 only
(d) 1, 2 and 3
Ans: (c)
Explanation:

Telegram: https://t.me/insightsIAStips
96
Youtube: https://www.youtube.com/channel/UCpoccbCX9GEIwaiIe4HLjwA
Revision Through MCQs (RTM) Compilation (July - 2022)

• S1&S2: The Law Commission of India is a non-statutory body constituted by the


Indian Government every three years.
• S3: A law commission comprises legal experts appointed by the Central
Government to promote justice in society.
Refer: https://www.insightsonindia.com/2022/07/23/issue-of-simultaneous-polls-with-
law-panel-govt/
192. Consider the following statements regarding United Nations Convention on the Law of
the Sea (UNCLOS).
1. UNCLOS is the only international convention which stipulates a framework for state
jurisdiction in maritime spaces.
2. International Seabed Authority (ISA) and International Tribunal for the Law of the
Sea (ITLOS) were established by the UNCLOS.
Which of the above statements is/are correct?
(a) 1 only
(b) 2 only
(c) Both 1 and 2
(d) Neither 1 nor 2
Ans: (c)
Explanation:
• The United Nations Convention on the Law of the Sea (UNCLOS) defines the
rights and responsibilities of nations with respect to their use of the world’s
oceans, establishing guidelines for businesses, the environment, and the
management of marine natural resources.
• International Seabed Authority (ISA) was established to organize, regulate and
control all mineral-related activities in the international seabed area beyond the
limits of national jurisdiction (referred to as “the Area”), an area underlying most
of the world’s oceans. It is an organization established by the United Nations
Convention on the Law of the Sea.
• The International Tribunal for the Law of the Sea (ITLOS) is
an intergovernmental organization created by the mandate of the Third United
Nations Conference on the Law of the Sea. It was established by the United
Nations Convention on the Law of the Sea.
• UNCLOS is the only international convention which stipulates a framework for
state jurisdiction in maritime spaces. It provides a different legal status to
different maritime zones. It provides the backbone for offshore governance by
coastal states and those navigating the oceans
Refer: https://www.insightsonindia.com/2022/07/23/anti-maritime-piracy-bill/
193. Which of the following are the advantages of Direct seeded rice (DSR) technique?
1. Requires less water
2. early crop maturity
3. less methane emission
Select the correct answer code:
(a) 1 and 3 only
(b) 2 and 3 only
(c) 1 and 2 only
(d) 1, 2 and 3
Ans: (d)
Explanation:
• Direct Seeding of Rice:

Telegram: https://t.me/insightsIAStips
97
Youtube: https://www.youtube.com/channel/UCpoccbCX9GEIwaiIe4HLjwA
Revision Through MCQs (RTM) Compilation (July - 2022)

• Direct seeded rice (DSR), probably the oldest method of crop establishment, is
gaining popularity because of its low-input demand.
• It offers certain advantages viz., it saves labour, requires less water, less
drudgery, early crop maturity, low production cost, better soil physical
conditions for following crops and less methane emission, provides better option
to be the best fit in different cropping systems.
Refer: https://www.insightsonindia.com/2022/07/23/dsr-technique-not-gaining-ground-
in-punjab/
194. Consider the following statements:
1. Russia and Ukraine are the world’s largest palm oil producers.
2. India is the world's biggest importer of palm oil.
Which of the statements given above is/are correct?
(a) 1 only
(b) 2 only
(c) Both 1 and 2
(d) Neither 1 nor 2
Ans: (b)
Explanation: About Palm Oil:
• It is an edible vegetable oil derived from the mesocarp (reddish pulp) of the fruit
of the oil palms.
• Uses: As a cooking oil, in cosmetics, processed foods, cakes, chocolates, soaps,
spreads, shampoo, and biofuel ( the use of crude palm oil in making biodiesel is
being branded as ‘green diesel’)
• It is inexpensive and produces more oil per hectare as compared to other
alternatives such as soybean.
• India is the biggest importer of palm oil (almost 40% of its need)
• Government initiatives: National Mission on Edible Oil-Oil Palm(to boost
India’s domestic palm oil production by 3 times ( till 2025-26)); Kharif Strategy
2021 for oilseeds; yellow revolution.
• Indonesia and Malaysia, are the world’s two largest palm oil producers,
accounting for 90% of the global production. However, most of it is produced
on the plantation by clearing rainforests.
Refer: Facts for Prelims: https://www.insightsonindia.com/2022/07/23/mission-2023-
insights-daily-current-affairs-pib-summary-23-july-2022/
195. Consider the following statements regarding Supply chain resilience initiative.
1. It is an initiative of India, Japan and Australia to achieve supply-chain resilience in
the Indo-Pacific region.
2. The idea is to reduce the reliance on China for supply of goods and services.
Which of the above statements is/are correct?
(a) 1 only
(b) 2 only
(c) Both 1 and 2
(d) Neither 1 nor 2
Ans: (c)
Explanation:
• Supply chain resilience initiative (SCRI) (by India, Japan and Australia) to
counter China’s dominance in Indo-pacific.

Telegram: https://t.me/insightsIAStips
98
Youtube: https://www.youtube.com/channel/UCpoccbCX9GEIwaiIe4HLjwA
Revision Through MCQs (RTM) Compilation (July - 2022)

• help in promoting trade facilitation by identifying supply chain risks and


mapping out the complementarities
Refer: Facts for Prelims: https://www.insightsonindia.com/2022/07/23/mission-2023-
insights-daily-current-affairs-pib-summary-23-july-2022/
196. The term ‘Glasgow Breakthrough’ was mentioned in news recently, is related to
(a) Women empowerment
(b) Global population control
(c) Global vaccination plan
(d) None of the above
Ans: (d)
Explanation:
• Glasgow Breakthroughs ( launched at the COP26 climate summit): a series
of actions across five key economic sectors that together represent more than
50% of global emissions.
• India is among 42 leaders to back and sign up for the UK’s Glasgow
Breakthroughs.
• India is also a co-convener of the Glasgow Breakthrough on Road Transport,
together with the UK and the US.
• It aims to make ZEVs (Zero Emission Vehicle systems) affordable, accessible and
sustainable in all regions by 2030.
• India is the fifth largest and fastest growing vehicle market in the world,
providing massive potential for electric vehicle uptake.
Refer: Facts for Prelims: https://www.insightsonindia.com/2022/07/23/mission-2023-
insights-daily-current-affairs-pib-summary-23-july-2022/
197. Consider the following statements about Bureau of Indian Standards (BIS):
1. It is the National Standard Body of India.
2. Its primary functions are Standardization, Quality Certification of Products and
Services.
3. It works under the aegis of Ministry of Commerce and Industry.
Which of the statements given above is/are correct?
(a) 1 and 2 only
(b) 2 and 3 only
(c) 1 and 3 only
(d) 1, 2 and 3
Ans: (a)
Explanation:
• BIS is the National Standard Body of India for the harmonious development of
the activities of standardization, marking and quality certification of goods.
• Bureau of Indian Standards is an autonomous body functioning under the
Ministry of Consumer Affairs, Food and Public Distribution, Govt. of India.
Its primary functions, as given in the Bureau of Indian Standards Act 1986 are
Standardization, Quality Certification of Products and Services.
Refer: Facts for Prelims: https://www.insightsonindia.com/2022/07/23/mission-2023-
insights-daily-current-affairs-pib-summary-23-july-2022/
198. Consider the following statements:
1. The Standard Mark of Bureau of Indian Standards (BIS) is mandatory for
automotive tyres and tubes.

Telegram: https://t.me/insightsIAStips
99
Youtube: https://www.youtube.com/channel/UCpoccbCX9GEIwaiIe4HLjwA
Revision Through MCQs (RTM) Compilation (July - 2022)

2. AGMARK is a quality Certification Mark issued by the Food and Agriculture


Organisation (FAO).
Which of the statements given above is/are correct?
(a) 1 only
(b) 2 only
(c) Both 1 and 2
(d) Neither 1 nor 2
Ans: (a)
Explanation:
• Statement 1: You can find it here
http://www.bis.org.in/cert/ProdUnManCert.asp
• Statement 2: AGMARK is a certification mark employed on agricultural products
in India, assuring that they conform to a set of standards approved by the
Directorate of Marketing and Inspection, an agency of the Government of India.
• www.agritech.tnau.ac.in/amis/food_agmark.html
Refer: Facts for Prelims: https://www.insightsonindia.com/2022/07/23/mission-2023-
insights-daily-current-affairs-pib-summary-23-july-2022/
199. According to the Wildlife (Protection) Act, 1972, which of the following animals cannot
be hunted by any person except under some provisions provided by law?
1. Gharial
2. Indian wild ass
3. Wild buffalo
Select the correct answer using the code given below:
(a) 1 only
(b) 2 and 3 only
(c) 1 and 3 only
(d) 1, 2 and 3
Ans: (d)
Explanation:
• All of these are protected scheduled animals under the Wildlife Act.
• http://envfor.nic.in/legis/wildlife/wildlife2s1.pdf
Refer: UPSC CSE 2017

200. Consider the following pairs:


1. Radhakanta Deb — First President of the British Indian Association
2. Gazulu Lakshminarasu Chetty —Founder of the Madras Mahajana Sabha
3. Surendranath Banerjee — Founder of the Indian Association
Which of the above pairs is/are correctly matched?
(a) 1 only
(b) 1 and 3 only
(c) 2 and 3 only
(d) 1, 2 and 3
Ans: (b)
Explanation:
• Statement 1: The first committee of the association was composed of: Raja
Radhakanta Deb – President, Raja Kalikrishna Deb – Vice-President,
Debendranath Tagore – secretary, et al.

Telegram: https://t.me/insightsIAStips
100
Youtube: https://www.youtube.com/channel/UCpoccbCX9GEIwaiIe4HLjwA
Revision Through MCQs (RTM) Compilation (July - 2022)

• Statement 2: In 1884, M. Veeraraghavachariar, G. Subramania Iyer and P.


Anandacharlu established the Madras Mahajana Sabha.
• Statement 3: IA was the first avowed nationalist organization founded in British
India by Surendranath Banerjee and Ananda Mohan Bose in 1876.
• https://en.wikipedia.org/wiki/Indian_National_Association
• https://en.wikipedia.org/wiki/British_Indian_Association
• https://en.wikipedia.org/wiki/Madras_Mahajana_Sabha
Refer: UPSC CSE 2017

RTM- REVISION THROUGH MCQS –25th-Jul-2022

201. Consider the following statements with reference to Antarctic Treaty:


1. Antarctica is defined as all of the land and ice shelves south of 60°S latitude.
2. Currently it has more than 94 parties.
3. India became a member of this treaty in 1983.
Which of the statements given above is/are correct?
(a) 1 and 3 only
(b) 1 only
(c) 2 and 3 only
(d) 1, 2 and 3
Ans: (a)
Explanation: About the Antarctic Treaty:
• Signed between 12 countries in Washington on 1st December 1959 for making
the Antarctic Continent a demilitarized zone to be preserved for scientific
research only.
• The twelve original signatories are Argentina, Australia, Belgium, Chile, France,
Japan, New Zealand, Norway, South Africa, the Union of Soviet Socialist
Republics, the UK and the US.
• The treaty entered into force in 1961 and currently has 54 parties.
• Headquarters: Buenos Aires, Argentina.
• For the purposes of the treaty system, Antarctica is defined as all of the land and
ice shelves south of 60°S latitude.
Refer: https://www.insightsonindia.com/2022/07/25/indian-antarctic-bill-2022-passed/
202. Which one of the following was the first Indian vessel to navigate Antarctic waters?
(a) Sagar Kanya
(b) Sagar Manjusha
(c) Sagar Nidhi
(d) Sagar Swana
Ans: (c)
Explanation:
• Sagar Nidhi (2008): It is first Indian vessel to navigate Antarctic waters.
Refer: https://www.insightsonindia.com/2022/07/25/indian-antarctic-bill-2022-passed/
203. Consider the following statements about International Court of Justice (ICJ):
1. It was established in 1945 by the United Nations charter.
2. It is situated at the Peace Palace in The Hague.
3. Its official languages are English and French.
Which of the given above statements is/are correct?

Telegram: https://t.me/insightsIAStips
101
Youtube: https://www.youtube.com/channel/UCpoccbCX9GEIwaiIe4HLjwA
Revision Through MCQs (RTM) Compilation (July - 2022)

(a) 1 and 2 only


(b) 2 and 3 only
(c) 2 and 3 only
(d) 1, 2 and 3
Ans: (d)
Explanation:
• S1: ICJ was established in 1945 by the United Nations charter and started
working in April 1946.
• S2: It is the principal judicial organ of the United Nations, situated at the Peace
Palace in The Hague (Netherlands).
• S3: ICJ is assisted by a Registry, its administrative organ. Its official languages
are English and French.
Refer: https://www.insightsonindia.com/2022/07/25/the-icjs-latest-judgment-in-the-
case-of-genocide-against-myanmar/
204. Consider the following statements regarding International Criminal Court (ICC).
1. It is the court of last resort for prosecution of genocide, war crimes, and crimes
against humanity.
2. It headquarters at The Hague, the Netherlands.
3. India and U.S. are the members of the court.
Which of the above statements is/are correct?
(a) 1 only
(b) 1 and 2 only
(c) 2 and 3 only
(d) 1, 2 and 3
Ans: (b)
Explanation:
• The International Criminal Court (ICC), located in The Hague, is the court of
last resort for prosecution of genocide, war crimes, and crimes against
humanity.
• It is the first permanent, treaty based, international criminal court established to
help end impunity for the perpetrators of the most serious crimes of concern to
the international community.
• Its founding treaty, the Rome Statute, entered into force on July 1, 2002.
• India is not a party to Rome Statute along with US and China.
Refer: https://www.insightsonindia.com/2022/07/25/the-icjs-latest-judgment-in-the-
case-of-genocide-against-myanmar/
205. Indian Bio-economy Report (IBER) 2022 is released by
(a) NITI Aayog
(b) BIRAC
(c) RBI
(d) SEBI
Ans: (b)
Explanation:
• BIRAC (Biotechnology Industry Research Assistance Council) releases the Indian
Bio-economy Report (IBER) 2022
Refer: Facts for Prelims: https://www.insightsonindia.com/2022/07/25/mission-2023-
insights-daily-current-affairs-pib-summary-25-july-2022/

Telegram: https://t.me/insightsIAStips
102
Youtube: https://www.youtube.com/channel/UCpoccbCX9GEIwaiIe4HLjwA
Revision Through MCQs (RTM) Compilation (July - 2022)

206. Consider the following statements:


1. Monkeypox is a viral zoonotic disease that occurs primarily in tropical rainforest
areas of Central and West Africa.
2. Monkeypox does not spread nearly as easily as COVID and there are vaccines and
treatments available, unlike the coronavirus when it emerged.
Which of the statements given above is/are correct?
(a) 1 only
(b) 2 only
(c) Both 1 and 2
(d) Neither 1 nor 2
Ans: (c)
Explanation: WHO declares monkey pox public health emergency of international
concern
• A “public health emergency of international concern” is WHO’s highest level of
alert.
• Monkeypoxis a viral zoonosis (a virus transmitted to humans from animals) with
symptoms similar to those seen in the past in smallpox patients, although it is
clinically less severe.
• Monkeypox does not spread nearly as easily as COVIDand there are vaccines
and treatments available, unlike the coronavirus when it emerged. But it has
still raised alarm.
• The viral disease, which causes flu-like symptoms and skin lesions, is endemicin
parts of Africa.
Refer: Facts for Prelims: https://www.insightsonindia.com/2022/07/25/mission-2023-
insights-daily-current-affairs-pib-summary-25-july-2022/
207. Due to some reasons, if there is a huge fall in the population of species of butterflies,
what could be its likely consequence/consequences?
1. Pollination of some plants could be adversely affected.
2. There could be a drastic increase in the fungal infections of some cultivated plants.
3. It could lead to a fall in the population of some species of wasps, spiders and birds.
Select the correct answer using the code given below:
(a) 1 only
(b) 2 and 3 only
(c) 1 and 3 only
(d) 1, 2 and 3
Ans: (c)
Explanation:
• Statement 1: Bees and butterflies play a vital role in the pollination of plants and
the production of crops by transporting pollen grains from one place to another.
• Statement 2: This is an absurd statement.
• Statement 3: These are some of the common predators of butterflies: wasps,
ants, parasitic flies, birds, snakes, toads, rats etc. The decline in butterfly
population would therefore adversely affect the food chain.
http://www.thebutterflysite.com/what-eats-
butterflies.shtml#sthash.PPoslJj4.dpuf
Refer: Facts for Prelims: https://www.insightsonindia.com/2022/07/25/mission-2023-
insights-daily-current-affairs-pib-summary-25-july-2022/

Telegram: https://t.me/insightsIAStips
103
Youtube: https://www.youtube.com/channel/UCpoccbCX9GEIwaiIe4HLjwA
Revision Through MCQs (RTM) Compilation (July - 2022)

208. Consider the following:


1. Birds
2. Dust blowing
3. Rain
4. Wind blowing
Which of the above spread plant diseases?
(a) 1 and 3 only
(b) 3 and 4 only
(c) 1, 2 and 4 only
(d) 1, 2, 3 and 4
Ans: (d)
Explanation:
• Nematodes, snails, birds, and wild and domestic animals often help
dissemination of plant diseases.

• The spores of many parasitic fungi are disseminated by air currents from
diseased to disease-free host Pathogens like, bacteria are often disseminated by
splashing of raindrops, as in case of Citrus canker disease.

• Soil and field operation also disseminate the diseases as they result in dust
blowing.
Refer: Facts for Prelims: https://www.insightsonindia.com/2022/07/25/mission-2023-
insights-daily-current-affairs-pib-summary-25-july-2022/

209. Consider the following statements:


1. No criminal proceedings shall be instituted against the governor of a state in any
court during his term of office
2. Emoluments and allowances of the governor of a state shall not be diminished
during his term of office.
Which of the statements given above is/are correct:
(a) 1 only
(b) 2 only
(c) Both 1 and 2
(d) Neither 1 nor 2
Ans: (c)
Explanation:
• Article 361. Protection of President and Governors
• No criminal proceedings whatsoever shall be instituted or continued against the
President, or the Governor of a State, in any court during his term of office,
• Article 158: Conditions of Governor Office
• The emoluments and allowances of the Governor shall not be diminished during
his term of office
Refer: UPSC CSE 2018
210. The well-known painting “BaniThani” belongs to the
(a) Bundi school
(b) Jaipur school
(c) Kangra school
(d) Kishangarh school

Telegram: https://t.me/insightsIAStips
104
Youtube: https://www.youtube.com/channel/UCpoccbCX9GEIwaiIe4HLjwA
Revision Through MCQs (RTM) Compilation (July - 2022)

Ans: (d)
Explanation:
• BaniThani is an Indian miniature painting painted by Nihâl Chand from the
Marwar school of Kishangarh. It portrays a woman who is elegant and graceful.
The painting’s subject, BaniThani, was a singer and poet in Kishangarh in the
time of kingSawant Singh (1748–1764).
• She has been compared to the Mona Lisa. Inspired by Radha, BaniThani is
characterized by idealized distinct features such as arched eyebrows, lotus-like
elongated eyes and pointed chin. This painting was featured in an Indian stamp
issued in 1973.
• Source: https://en.wikipedia.org/wiki/Bani_Thani
Refer: UPSC CSE 2018

RTM- REVISION THROUGH MCQS –26th-Jul-2022

211. With reference to Sun Temple, Modhera, consider the following statements:
1. It is situated on the bank of the river Sabarmati.
2. It was built during the reign of Bhima I of the Chaulukya dynasty.
Which of the statements given above is/are correct?
(a) 1 only
(b) 2 only
(c) Both 1 and 2
(d) Neither 1 nor 2
Ans: (b)
Explanation:
• The Sun Temple is a Hindu temple dedicated to the solar deity Surya located
at Modhera village of Mehsana district, Gujarat, India.
• It is situated on the bank of the river Pushpavati.
• It was built after 1026-27 CE during the reign of Bhima I of the Chalukyas
dynasty.
Refer: Facts for Prelims: https://www.insightsonindia.com/2022/07/26/mission-2023-
insights-daily-current-affairs-pib-summary-26-july-2022/
212. After the Santhal Uprising subsided, what was/were the measure/measures taken by
the colonial government?
1. The territories called ‘Santhal Paraganas’ were created.
2. It became illegal for a Santhal to transfer land to a non-Santhal.
Select the correct answer using the codes below.
(a) 1 only
(b) 2 only
(c) Both 1 and 2
(d) Neither 1 nor 2
Ans: (c)
Explanation:
• S1: Due to the immense force of the uprising, the British government became
more cautious about the needs of the tribals the Santhal inhabited areas were

Telegram: https://t.me/insightsIAStips
105
Youtube: https://www.youtube.com/channel/UCpoccbCX9GEIwaiIe4HLjwA
Revision Through MCQs (RTM) Compilation (July - 2022)

constituted into a separate administrative unit, called the Santhal Parganas,


which recognised the distinctiveness of their tribal culture and identity.
• S2: Also, it led the British government to enact laws so that the land of tribals
could not be taken by outsiders (dikus).
Refer: Facts for Prelims: https://www.insightsonindia.com/2022/07/26/mission-2023-
insights-daily-current-affairs-pib-summary-26-july-2022/
213. Consider the following statements regarding Paika Rebellion.
1. It was an armed rebellion against the British East India Company’s rule in Gujarat.
2. Rise in prices of salt due to taxes imposed on it was one of the causes of the
rebellion.
Which of the above statements is/are correct?
(a) 1 only
(b) 2 only
(c) Both 1 and 2
(d) Neither 1 nor 2
Ans: (b)
Explanation:
• The Paika Rebellion, also called the Paika Bidroha, was an armed rebellion
against the British East India Company’s rule in Odisha in 1817.
• The Paikas rose in rebellion under their leader Bakshi Jagabandhu and,
projecting Jagannath as the symbol of Odia unity, the rebellion quickly spread
across most of Odisha before being ruthlessly put down by the company’s forces.
• They attacked British symbols of power, setting ablaze police stations,
administrative offices and the treasury during their march towards Khurda, from
where the British fled.
• The extortionist land revenue policy of the company affected the peasants and
the zamindars alike. A source of much consternation for the common people was
the rise in prices of salt due to taxes imposed on it by the new
government. The British authority deprived the zamindars and the local people
of coastal region of their traditional rights to manufacture salt.
• They were supported by the zamindars, village heads and ordinary
peasants.
• It is the first war of independence that started in 1817 much before the Sepoy
mutiny of 1857.
Refer: Facts for Prelims: https://www.insightsonindia.com/2022/07/26/mission-2023-
insights-daily-current-affairs-pib-summary-26-july-2022/
214. With reference to tribal rebellion in India, ‘Buddhu Bhagat, Joa Bhagat and Madara
Mahato’ were associated with
(a) Paika rebellion
(b) Kol revolt
(c) Bhil uprising
(d) Santhal revolution
Ans: (b)
Explanation:
• The Kols, tribal people from the Chhota Nagpur area, rose in revolt against
the British in 1831. The trigger here too was the gradual takeover of tribal land
and property by non-tribal settlers who were aided by new land laws.

Telegram: https://t.me/insightsIAStips
106
Youtube: https://www.youtube.com/channel/UCpoccbCX9GEIwaiIe4HLjwA
Revision Through MCQs (RTM) Compilation (July - 2022)

• It was led to an uprising led by Buddhu Bhagat, Joa Bhagat and Madara
Mahato among others.
Refer: Facts for Prelims: https://www.insightsonindia.com/2022/07/26/mission-2023-
insights-daily-current-affairs-pib-summary-26-july-2022/
215. “To abide by the Constitution and respect its ideals and institutions, the National Flag
and the National Anthem ” is a provision made in the
(a) Preamble of the Constitution
(b) Directive Principles of State Policy
(c) Fundamental Rights
(d) Fundamental Duties
Ans: (d)
Explanation:
• Art 51A(a) – To abide by the Constitution and respect its ideals and institutions,
the National Flag and the National Anthem.
Refer: Facts for Prelims: https://www.insightsonindia.com/2022/07/26/mission-2023-
insights-daily-current-affairs-pib-summary-26-july-2022/
216. Consider the following statements about the Swadesh Darshan Scheme:
1. It is a Central Sector scheme.
2. It was launched by the Ministry of Tourism, to develop theme-based tourist circuits
in the country.
Which of the above statements is/are correct?
(a) 1 only
(b) 2 only
(c) Both 1 and 2
(d) Neither 1 nor 2
Ans: (c)
Explanation: Swadesh Darshan Scheme:
• It was launched by the Ministry of Tourism in 2014-15 to develop theme-based
tourist circuits in the country.
• It is a 100% centrally funded scheme for project components undertaken for
public funding.
• It also has provision for leveraging voluntary funding under Corporate Social
Responsibility (CSR) initiatives of Central Public Sector Undertakings and the
corporate sector.
Refer: Facts for Prelims: https://www.insightsonindia.com/2022/07/26/mission-2023-
insights-daily-current-affairs-pib-summary-26-july-2022/
217. Snow Leopard is naturally found in which of the following states?
1. Jammu and Kashmir
2. Arunachal Pradesh
3. Sikkim
4. Uttar Pradesh
Select the correct answer code:
(a) 1 and 3 only
(b) 2 and 3 only
(c) 1, 2, 3 and 4
(d) 1, 2 and 3 only
Ans: (d)
Explanation:

Telegram: https://t.me/insightsIAStips
107
Youtube: https://www.youtube.com/channel/UCpoccbCX9GEIwaiIe4HLjwA
Revision Through MCQs (RTM) Compilation (July - 2022)

• The snow leopard inhabits the higher Himalayan and trans-Himalayan


landscape in Jammu and Kashmir, Himachal Pradesh, Uttarakhand, Sikkim,
and Arunachal Pradesh.
• Snow Leopard is not found naturally in Uttar Pradesh.
Refer: Facts for Prelims: https://www.insightsonindia.com/2022/07/26/mission-2023-
insights-daily-current-affairs-pib-summary-26-july-2022/
218. ‘Virunga National Park’, a UNESCO World Heritage Site, was in news recently, is
located in the
(a) USA
(b) Democratic Republic of the Congo
(c) Bangladesh
(d) Sri Lanka
Ans: (b)
Explanation:


Refer: Facts for Prelims: https://www.insightsonindia.com/2022/07/26/mission-2023-
insights-daily-current-affairs-pib-summary-26-july-2022/
219. “System of Rice Intensification” of cultivation, in which alternate wetting and drying of
rice fields is practised, results in:
1. Reduced seed requirement
2. Reduced methane production
3. Reduced electricity consumption
Select the correct answer using the code given below:
(a) 1 and 2 only
(b) 2 and 3 only
(c) 1 and 3 only
(d) 1, 2 and 3
Ans: (d)
Explanation:
• Option d is the correct answer
• System of Rice Intensification (SRI)
o It is also called the Madagascar method of rice cultivation as it was first
developed there. It promises to save 15 to 20 per cent of ground water,
and improve rice productivity, which is almost at a stagnant point now.
o

Telegram: https://t.me/insightsIAStips
108
Youtube: https://www.youtube.com/channel/UCpoccbCX9GEIwaiIe4HLjwA
Revision Through MCQs (RTM) Compilation (July - 2022)

o Experts said that it gives equal to or more productive than conventional


rice cultivation, with less water, less seed and less chemicals. The net
effect is a substantial reduction in the investments in external inputs.
o
o SRI is suitable in all types of soil including less fertile soil as in such soil
the number of seedlings can be increased to double.
• Refer: https://drawdown.org/solutions/system-of-rice-intensification
Refer: Facts for Prelims: https://www.insightsonindia.com/2022/07/26/mission-2023-
insights-daily-current-affairs-pib-summary-26-july-2022/
220. N-Treat technology is related to
(a) cancer treatment
(b) coal gas separation
(c) disaster management
(d) sewage management
ANs: (d)
Explanation:
• IIT-Bombay to help treat Mumbai’s sewage with new technology.
• N-Treat is a seven-stage process for waste treatment that uses screens, gates,
silt traps, curtains of coconut fibres for filtration, and disinfection using
sodium hypochlorite.
• The first stage involves screening to prevent the entry of floating
objects such as plastic cups, paper dishes, polythene bags, sanitary napkins, or
wood.
• The second stage has proposed the construction of a silt trap, which creates
an inclination and ‘parking spot’ on the bed of the nullah for sedimentation.
• The next three stages are the installation of ‘bio zones’ in the form of
coconut fibre curtains that will act as filters and promote the growth of biofilm
to help in the decomposition of organic matter.
• The final stage for sewage treatment will include disinfection using sodium
hypochlorite, to kill the bacteria in the water.
Refer: Facts for Prelims: https://www.insightsonindia.com/2022/07/26/mission-2023-
insights-daily-current-affairs-pib-summary-26-july-2022/

RTM- REVISION THROUGH MCQS –27th-Jul-2022

221. Consider the following statements regarding Speaker in India


1. He is provided with a security of tenure.
2. His salaries and allowances are fixed by Parliament.
3. There is a convention that the Speaker has to resign from his party and remain
politically neutral.
Which of the statements given above are correct?
(a) 1 and 2 only
(b) 2 and 3 only
(c) 1 and 3 only
(d) 1, 2 and 3
Ans: (a)

Telegram: https://t.me/insightsIAStips
109
Youtube: https://www.youtube.com/channel/UCpoccbCX9GEIwaiIe4HLjwA
Revision Through MCQs (RTM) Compilation (July - 2022)

Explanation:
• Independence and Impartiality: As the office of the Speaker is vested with
great prestige, position and authority, independence and impartiality become
its sine qua non. The following provisions ensure the independence and
impartiality of the office of the Speaker:
• He is provided with a security of tenure. He can be removed only by a
resolution passed by the Lok Sabha by a special majority (ie, a majority of all the
then members of the House) and not by an ordinary majority (ie, a majority of
the members present and voting in the House). This motion of removal can be
considered and discussed only when it has the support of at least 50 members.
• His salaries and allowances are fixed by Parliament. They are charged on the
Consolidated Fund of India and thus are not subject to the annual vote of
Parliament.
o His work and conduct cannot be discussed and criticized in the Lok
Sabha except on a substantive motion.
o His powers of regulating procedure or conducting business or
maintaining order in the House are not subject to the jurisdiction of any
Court.
o He cannot vote in the first instance. He can only exercise a casting vote
in the event of a tie. This makes the position of Speaker impartial.
o He is given a very high position in the order of precedence. He is placed
at seventh rank, along with the Chief Justice of India. This means, he
has a higher rank than all cabinet ministers, except the Prime Minister or
Deputy Prime Minister.
• In Britain, the Speaker is strictly a nonparty man. There is a convention
that the Speaker has to resign from his party and remain politically
neutral.
• This healthy convention is not fully established in India where the Speaker does
not resign from the membership of his party on his election to the exalted office.
Refer: https://www.insightsonindia.com/2022/07/27/suspension-of-mps-from-
parliament/
222. Consider the following statements:
1. Like the Speaker in Lok Sabha, the Chairman of Rajya Sabha is empowered to
“direct any Member whose conduct is in his opinion grossly disorderly to withdraw
immediately” from the House.
2. Unlike the Speaker, the Rajya Sabha Chairman does not have the power to suspend
a Member.
Which of the statements given above is/are correct?
(a) 1 only
(b) 2 only
(c) Both 1 and 2
(d) Neither 1 nor 2
Ans: (c)
Explanation:
• Like the Speaker in Lok Sabha, the Chairman of Rajya Sabha is empowered
under Rule Number 255 of its Rule Book to “direct any Member whose conduct
is in his opinion grossly disorderly to withdraw immediately” from the House.
• Unlike the Speaker, however, the Rajya Sabha Chairman does not have the
power to suspend a Member.

Telegram: https://t.me/insightsIAStips
110
Youtube: https://www.youtube.com/channel/UCpoccbCX9GEIwaiIe4HLjwA
Revision Through MCQs (RTM) Compilation (July - 2022)

Refer: : https://www.insightsonindia.com/2022/07/27/suspension-of-mps-from-
parliament/
223. Consider the following statements about the Bharat Stage Emission Standards
(BSES):
1. They are set by the Central Pollution Control Board under the Ministry of
Environment and Climate Change.
2. They are based on European (EURO) emission standards.
3. Bharat Stage (BS) emission norms were first brought into effect in 2014 under the
head “India 2020”.
Which of the statements given above is/are correct?
(a) 1 and 2 only
(b) 2 and 3 only
(c) 1 and 3 only
(d) 1, 2 and 3
Ans: (a)
Explanation: Bharat Stage Emission Standards (BSES
• Introduced in the year 2000.
• They are set by the Central Pollution Control Board under the Ministry of
Environment and Climate Change.
• Objective: To keep air pollutants emitted by the internal combustion engine of
vehicles under control.
• They are based on European (EURO) emission standards.
• Bharat Stage (BS) emission norms were first brought into effect in 2000 under
the head “India 2000”. This was followed by BS2 in 2001 and BS3 in 2005.
• However, the emission norms were made more stringent only with the
enforcement of Bharat Stage IV (BS4). Thereafter, the Government of India
skipped the implementation of BS5 in 2016 and decided to introduce
Bharat Stage VI (BS6) in 2020 instead.
Refer: https://www.insightsonindia.com/2022/07/27/use-of-technology-to-make-efficient-
auto-vehicles/
224. Consider the following pairs:
1. Karikili Bird Sanctuary: Kerala
2. Pala wetland: West Bengal
3. Pichavaram Mangrove: Tamil Nadu
4. Sakhya Sagar: Madhya Pradesh
Which of the given above pairs is/are correctly matched?
(a) 1 and 2 only
(b) 1, 2 and 3 only
(c) 2, 3 and 4 only
(d) 3 and 4 only
Ans: (d)
Explanation:
• Pallikaranai Marsh Reserve Forest (TN): It is a freshwater marsh in Chennai.
It is the only surviving wetland ecosystem in Chennai and the last remaining
natural wetlands of South India.
• Karikili Bird Sanctuary (TN): Located in the Kancheepuram District of Tamil
Nadu

Telegram: https://t.me/insightsIAStips
111
Youtube: https://www.youtube.com/channel/UCpoccbCX9GEIwaiIe4HLjwA
Revision Through MCQs (RTM) Compilation (July - 2022)

• Pichavaram Mangrove (TN): It is located near Chidambaram in the Cuddalore


District of Tamil Nadu. It could be counted among the largest mangrove forests
in the country (more than 1100 hectares).
• Pala wetland (Mizoram): It is the largest natural wetland in Mizoram. The
wetland is surrounded by green woodlands.
• Sakhya Sagar (MP): This lake is an integral part of Madhav National Park in
Shivpuri, Madhya Pradesh.
Refer: facts for prelims: https://www.insightsonindia.com/2022/07/27/mission-2023-
insights-daily-current-affairs-pib-summary-27-july-2022/
225. Consider the following statements about the Convention on Wetlands:
1. It is the intergovernmental treaty that provides the framework for the conservation
and wise use of wetlands and their resources.
2. It was adopted in the Iraqi city of Ramsar in 1971.
3. The United Nations Environment Programme (UNEP) provides secretariat services to
the Convention on Wetlands.
Which of the given above statements is/are not correct?
(a) 1 only
(b) 2 and 3 only
(c) 2 only
(d) 1, 2 and 3
Ans: (b)
Explanation: here the directive word is not correct!!
• S1: The Convention on Wetlands is the intergovernmental treaty that
provides the framework for the conservation and wise use of wetlands and their
resources.
• S2: The Convention was adopted in the Iranian city of Ramsar in 1971 and
came into force in 1975. Since then, almost 90% of UN member states, from all
the world’s geographic regions, have acceded to become “Contracting Parties”.
• S3: The International Union for Conservation of Nature (IUCN) provides
administrative services to the Convention on Wetlands. The Convention
Secretariat follows IUCN procedures and policies as applicable.
Refer: facts for prelims: https://www.insightsonindia.com/2022/07/27/mission-2023-
insights-daily-current-affairs-pib-summary-27-july-2022/
226. Consider the following statements about the Montreux Record:
1. It was established by Recommendation of the Conference of the Contracting Parties
to the United Nations Environment Programme (UNEP).
2. It was designed to stop the production and import of wetland depleting substances
and reduce their concentration in the atmosphere.
Which of the given above statements is/are correct?
(a) 1 only
(b) 2 only
(c) Both 1 and 2
(d) Neither 1 nor 2
Ans: (d)
Explanation:
• Montreux Record under the Convention is a register of wetland sites on the
List of Wetlands of International Importance where changes in ecological

Telegram: https://t.me/insightsIAStips
112
Youtube: https://www.youtube.com/channel/UCpoccbCX9GEIwaiIe4HLjwA
Revision Through MCQs (RTM) Compilation (July - 2022)

character have occurred, are occurring, or are likely to occur as a result of


technological developments, pollution or other human interference.
o It is maintained as part of the Ramsar List.
• The Montreux Record was established by Recommendation of the Conference
of the Contracting Parties (1990).
• Sites may be added to and removed from the Record only with the approval of
the Contracting Parties in which they lie.
• Currently, two wetlands of India are in Montreux record: Keoladeo National
Park (Rajasthan) and Loktak Lake (Manipur).
• Chilka lake (Odisha) was placed in the record but was later removed from it
Refer: facts for prelims: https://www.insightsonindia.com/2022/07/27/mission-2023-
insights-daily-current-affairs-pib-summary-27-july-2022/
227. Arrange the following Ramsar Sites of India in the direction of East to West:
1. Kanwar Taal
2. Chilika Lake
3. Lonar Lake
4. Bhoj Wetland
Select the correct answer using the code below:
(a) 1-2-3-4
(b) 1-2-4-3
(c) 2-1-3-4
(d) 2-1-4-3
Ans: (b)
Explanation:’


Refer: facts for prelims: https://www.insightsonindia.com/2022/07/27/mission-2023-
insights-daily-current-affairs-pib-summary-27-july-2022/
228. Consider the following statements:
1. Alibaug is known as Goa of Maharashtra State.
2. Alibagh is known for growing white onion using the traditional method.
3. The famed white onions of Alibagh have low pungency and ‘no tear’ factor.
Which of the statements given above is/are correct?
(a) 1 and 2 only
(b) 2 only
(c) 1 and 3 only
(d) 1, 2 and 3

Telegram: https://t.me/insightsIAStips
113
Youtube: https://www.youtube.com/channel/UCpoccbCX9GEIwaiIe4HLjwA
Revision Through MCQs (RTM) Compilation (July - 2022)

Ans: (d)
Explanation:
• S1: Alibaug is a coastal town also known as 'Mini Goa' located on the western
coast of India in the Raigad district of Maharashtra.
• S2: Alibagh (Maharastra) is known for growing white onion using the traditional
method and utilizing geo-climatic conditions for a unique taste, flavour and
shape.
o The soil of Alibaug taluka has low sulphur content.
• S3: The onions have low pungency, sweet taste, ‘no tear’ factor, low pyruvic
acid, high protein, fat and fibre content, besides high antioxidant
compounds (quercetin).
• It boosts immunity, helps with insomnia, blood cleaning, blood pressure and
heat-related ailments.
Refer: facts for prelims: https://www.insightsonindia.com/2022/07/27/mission-2023-
insights-daily-current-affairs-pib-summary-27-july-2022/
229. Consider the following statements:
1. Under Article 82 of the Constitution of India, the Parliament enacts a Delimitation
Act after every Census.
2. Under Article 170 of the Constitution of India, States also get divided into territorial
constituencies as per the Delimitation Act after every Census.
Which of the given above statements is/are correct?
(a) 1 only
(b) 2 only
(c) Both 1 and 2
(d) Neither 1 nor 2
Ans: (c)
Explanation:
• What is Delimitation:- Delimitation literally means the process of fixing limits
or boundaries of territorial constituencies in a state that has a legislative body.
• Delimitation commission orders have the force of law and they cannot be
challenged before any court.
• Composition of the Commission: According to the Delimitation Commission
Act, 2002, the Delimitation Commission will have three members: a serving
or retired judge of the Supreme Court as the chairperson, and the Chief Election
Commissioner or Election Commissioner nominated by the CEC and the State
Election Commissioner as ex-officio members.
• Constitutional Provisions:
o Under Article 82, the Parliament enacts a Delimitation Act after every
Census.
o Under Article 170, States also get divided into territorial constituencies
as per the Delimitation Act after every Census.
Refer: facts for prelims: https://www.insightsonindia.com/2022/07/27/mission-2023-
insights-daily-current-affairs-pib-summary-27-july-2022/
230. Along with the budget, the Finance Minister also places other documents before the
parliament which include “The Macro Economic Framework Statement” The aforesaid
document is presented because this mandated by
(a) Long standing parliamentary convention
(b) Article 112 and article 110(1) of the constitution of India

Telegram: https://t.me/insightsIAStips
114
Youtube: https://www.youtube.com/channel/UCpoccbCX9GEIwaiIe4HLjwA
Revision Through MCQs (RTM) Compilation (July - 2022)

(c) Article 114 of the Constitution of India


(d) Provisions of the Fiscal Responsibility and Budget Management act, 2003
Ans: (d)
Explanation:
• The Macro-economic Framework Statement is a statement presented to the
Parliament at the time of Union Budget under Section 3(5) of the Fiscal
Responsibility and Budget Management Act, 2003.]
• Read more>> https://www.business-standard.com/about/what-is-frbm-act
Refer: facts for prelims: https://www.insightsonindia.com/2022/07/27/mission-2023-
insights-daily-current-affairs-pib-summary-27-july-2022/

RTM- REVISION THROUGH MCQS –28th-Jul-2022

231. With reference to Foreign Contribution Regulation (Amendment), Act 2020, which of
the following statements is/are correct?
1. Under the Act, foreign contribution cannot be transferred to any other person unless
such person is also registered for that purpose.
2. The act states that foreign contributions must be received only in an FCRA account
opened in the State Bank of India, New Delhi Branch.
3. The act increased administrative expenses through foreign funds by an organization
compared to Foreign Contribution (Regulation) Act, 2010.
Select the correct answer using the code below:
(a) 1 and 2 only
(b) 2 and 3 only
(c) 1 and 3 only
(d) 1, 2 and 3
Ans: (a)
Explanation; Foreign Contribution Regulation (Amendment), Act 2020:
• Under the Act, foreign contribution cannot be transferred to any other
person unless such person is also registered for that purpose.
• The amendment also forbids sub-granting by NGOs to smaller NGOs who work
at the grassroots.
• The act states that foreign contributions must be received only in an FCRA
account opened in the State Bank of India, New Delhi Branch. No funds
other than the foreign contribution should be received or deposited in this
account.
• Aadhar usage: The act makes it compulsory for all trustees to register their
Aadhaar card with the FCRA account.
• The Act also makes Aadhaar a mandatory identification document. It is for all
the office bearers, directors, and other key functionaries of an NGO.
• Restriction in utilisation of foreign contribution: The act gives government
powers to stop utilization of foreign funds by an organization through a
“summary enquiry”.
• Reduction in use of foreign contribution for administrative purposes: The
act decreases administrative expenses through foreign funds by an
organization to 20% from 50% earlier.

Telegram: https://t.me/insightsIAStips
115
Youtube: https://www.youtube.com/channel/UCpoccbCX9GEIwaiIe4HLjwA
Revision Through MCQs (RTM) Compilation (July - 2022)

• Surrender of certificate: The act allows the central government to permit a


person to surrender their registration certificate.
Refer: https://www.insightsonindia.com/2022/07/28/seeking-to-destroy-indias-civil-
society/

232. Consider the following statements about the Warehousing Development and
Regulatory Authority (WDRA):
1. It was constituted in 2010 under the Warehousing (Development and Regulation)
Act, 2007.
2. It works under the Department of Food and Public Distribution to ensure scientific
storage by prescribing infrastructural and procedural standards.
Which of the statements given above is/are correct?
(a) 1 only
(b) 2 only
(c) Both 1 and 2
(d) Neither 1 nor 2
Ans: (c)
Explanation:
• It was constituted in 2010 under the Warehousing (Development and
Regulation) Act, 2007, under the Department of Food and Public Distribution
to ensure scientific storage by prescribing infrastructural and procedural
standards.
• Negotiable Warehouse Receipt System
• Launched in 2011, through it farmers can seek loans from banks against the
warehouse receipts issued to them against their storage. The Electronic
Negotiable Warehouse Receipt (e-NWR) System was launched in 2017.
Refer: https://www.insightsonindia.com/2022/07/28/centre-to-amend-warehousing-act/
233. “In India, it is traditionally grown in the Western Ghats spread over Karnataka, Kerala
and Tamil Nadu. It requires hot and humid climate with temperature varying between
15°C and 28 °C and rainfall from 150 to 250 cm. It does not tolerate frost, snowfall,
high temperature above 30°C and strong sun shine and is generally grown under shady
trees.” Which one of the following is that crop?
(a) Jute
(b) Sugarcane
(c) Tea
(d) Coffee
Ans: (d)
Explanation:

Telegram: https://t.me/insightsIAStips
116
Youtube: https://www.youtube.com/channel/UCpoccbCX9GEIwaiIe4HLjwA
Revision Through MCQs (RTM) Compilation (July - 2022)


Refer: facts for prelims: https://www.insightsonindia.com/2022/07/28/mission-2023-
insights-daily-current-affairs-pib-summary-28-july-2022/
234. Consider the following statements about the United Nations Relief and Works Agency
(UNRWA):
1. It is a United Nations agency established by the General Assembly in 1999.
2. It supports the relief and human development of Afghanistan refugees.
3. It is funded almost entirely by voluntary contributions and financial support.
Which of the statements given above is/are not correct?
(a) 1 only
(b) 1 and 2 only
(c) 2 and 3 only
(d) 1, 2 and 3
Ans: (b)
Explanation: United Nations Relief and Works Agency (UNRWA)
• Established in 1949 with the mandate to provide assistance and protection to
about 5.6 mn Palestinian refugees (in West Bank, Gaza Strip, Lebanon, Syria
and Jordan)
• Funding: funded almost entirely by voluntary contributions and financial
support
• India: As a part of the Link West Policy, India has de-hyphenated its
relationship with Israel and Palestine in 2018 to treat both the countries as
mutually independent and exclusive.
Refer: facts for prelims: https://www.insightsonindia.com/2022/07/28/mission-2023-
insights-daily-current-affairs-pib-summary-28-july-2022/
235. Consider the following statements:
1. Indian Council of Medical Research (ICMR) is India's national registry for clinical
trials.
2. It is funded by the Government of India through the Department of Health Research,
Ministry of Health & Family Welfare.
Which of the statements given above is/are correct?
(a) 1 only
(b) 2 only

Telegram: https://t.me/insightsIAStips
117
Youtube: https://www.youtube.com/channel/UCpoccbCX9GEIwaiIe4HLjwA
Revision Through MCQs (RTM) Compilation (July - 2022)

(c) Both 1 and 2


(d) Neither 1 nor 2
Ans: (b)
Explanation:
• Indian Council of Medical Research (ICMR) is the apex body in India for the
formulation, coordination and promotion of biomedical research.
• S1: The ICMR is funded by the Government of India through the Department of
Health Research, Ministry of Health and Family Welfare. In 2007 the
organization established the Clinical Trials Registry - India, which is India's
national registry for clinical trials.
• ICMR’s EOI document states that it reserves all the Intellectual Property
Rights and Commercialisation Rights on the Monkeypox virus isolates and
its method/protocols for purification, propagation and characterisation.
• ICMR is lawfully entitled to enter into any form of non-exclusive agreements
with experienced Drug/Pharma/Vaccine/IVD manufacturers through
defined agreement for undertaking R&D as well as manufacturing activities.
Refer: facts for prelims: https://www.insightsonindia.com/2022/07/28/mission-2023-
insights-daily-current-affairs-pib-summary-28-july-2022/
236. Consider the following statements about Light-malted Albatross:
1. It is a seabird native to Bay of Bengal.
2. It is listed as 'Vulnerable' on the IUCN Red List of Threatened Species.
Which of the statements given above is/are correct?
(a) 1 only
(b) 2 only
(c) Both 1 and 2
(d) Neither 1 nor 2
Ans: (d)
Explanation:
• Context: It is a seabird native to Antarctic seas, but has been sighted for the
first time in Asia (Rameswaram and adjoining islets of the Gulf of Mannar
Marine National Park on the Adam’s Bridge (Ram Setu)
• Indication: It shows the pattern of bird migration away from the well-known and
established routes and sites. Changes in the wind pattern triggered by global
warming are bringing non-native birds to other areas.
• IUCN: Near Threatened
Refer: facts for prelims: https://www.insightsonindia.com/2022/07/28/mission-2023-
insights-daily-current-affairs-pib-summary-28-july-2022/
237. With reference to the religious history of India, consider the following statements:
1. Sautrantika and Sammitiya were the sects of Jainism.
2. Sarvastivadin held that the constituents of phenomena were not wholly momentary,
but existed forever in a latent form.
Which of the statements given above is/are correct?
(a) 1 only
(b) 2 only
(c) Both 1 and 2
(d) Neither 1 nor 2
Ans: (b)
Explanation:

Telegram: https://t.me/insightsIAStips
118
Youtube: https://www.youtube.com/channel/UCpoccbCX9GEIwaiIe4HLjwA
Revision Through MCQs (RTM) Compilation (July - 2022)

• Statement 1: These are sects of Buddhism.


• Statement 2: While, like all Buddhists, the Sarvastivadins consider everything
empirical to be impermanent, they maintain that the dharma factors are
eternally existing realities.
• The dharmas are thought to function momentarily, producing the empirical
phenomena of the world, which is illusory, but to exist outside the empirical
world.
• https://www.britannica.com/topic/Sarvastivada
Refer: UPSC CSE 2017
238. Mediterranean Sea is a border of which of the following countries?
1. Jordan
2. Iraq
3. Lebanon
4. Syria
Select the correct answer using the code given below:
(a) 1, 2 and 3 only
(b) 2 and 3 only
(c) 3 and 4 only
(d) 1, 3 and 4 only
Ans: (c)
Explanation:


Refer: UPSC CSE 2017
239. Consider the following statements:
1. In India, the Himalayas are spread over five States only.
2. Western Ghats are spread over five States only.
3. Pulicat Lake is spread over two States only.
Which of the statements given above is/are correct?
(a) 1 and 2 only
(b) 3 only
(c) 2 and 3 only

Telegram: https://t.me/insightsIAStips
119
Youtube: https://www.youtube.com/channel/UCpoccbCX9GEIwaiIe4HLjwA
Revision Through MCQs (RTM) Compilation (July - 2022)

(d) 1 and 3 only


Ans: (b)
Explanation:
• Statement 1: Himalayas are spread over almost all the Northern and north-
eastern Indian states. Statement 1 is wrong.
• Statement 2: Western Ghats traverse the States of Kerala, Tamil Nadu,
Karnataka, Goa, Maharashtra and Gujarat (6 states).
• Statement 3: Pulicat Lake (second largest brackish water lake in India) straddles
the border of Andhra Pradesh and Tamil Nadu states (2 states).
Refer: UPSC CSE 2017
240. With reference to ‘Indian Ocean Dipole (IOD)’ sometimes mentioned in the news while
forecasting Indian monsoon, which of the following statements is/are correct?
1. IOD phenomenon is characterised by a difference in sea surface temperature
between tropical Western Indian Ocean and tropical Eastern Pacific Ocean.
2. An IOD phenomenon can influence an El Nino’s impact on the monsoon.
Select the correct answer using the code given below:
(a) 1 only
(b) 2 only
(c) Both 1 and 2
(d) Neither 1 nor 2
Ans: (b)
Explanation:
• Statement 1: The IOD, also known as the Indian Niño, is an irregular oscillation
of sea-surface temperatures in which the western Indian Ocean becomes
alternately warmer and then colder than the eastern part of the Indian ocean
(not tropical eastern pacific ocean.
• Statement 2: IOD has a much more significant effect on the rainfall patterns in
south-east Australia than the El Niño-Southern Oscillation (ENSO) in the Pacific
Ocean as shown in several recent studies.
Refer: UPSC CSE 2017

RTM- REVISION THROUGH MCQS –29th-Jul-2022

241. Consider the following statements about The Energy and Resources Institute (TERI):
1. It is funded by the Ministry of Science and Technology.
2. It operates as an autonomous body through the Societies Registration Act, 1860.
3. It conducts research work in the fields of energy, environment and sustainable
development.
Which of the statements given above is/are correct?
(a) 1 and 2 only
(b) 3 only
(c) 2 and 3 only
(d) 1, 2 and 3
Ans: (b)
Explanation:

Telegram: https://t.me/insightsIAStips
120
Youtube: https://www.youtube.com/channel/UCpoccbCX9GEIwaiIe4HLjwA
Revision Through MCQs (RTM) Compilation (July - 2022)

• The Energy and Resources Institute (TERI) is an independent, multi-


dimensional research organization with capabilities in policy research,
technology development, and implementation.
Refer: https://www.insightsonindia.com/2022/07/29/teri-roadmap-for-achieving-indias-
decarbonization-targets/
242. Consider the following statements:
1. As per the World Wide Fund for Nature, the number of tigers increased by 95 per
cent over the past 150 years.
2. India is the land of royal tigers and current tiger population is 80 per cent of the
global tiger population.
3. In India, Madhya Pradesh has the highest number of tigers, closely followed by
Karnataka.
Which of the statements given above is/are correct?
(a) 1 and 2 only
(b) 3 only
(c) 2 and 3 only
(d) 1, 2 and 3
Ans: (b)
Explanation: Key facts related to tiger population:
• As per the World Wide Fund for Nature,the number of tigers dropped by 95 per
cent over the past 150 years.
• India is the land of royal tigers and current tiger population stands at 2967
which is 70 per cent of the global tiger population.
• Madhya Pradesh has the highest number of tigers at 526,closely followed by
Karnataka (524) and Uttarakhand (442).
• Kanha Tiger Reserve in Madhya Pradesh is the first tiger reserve in India to
officially introduce a mascot, Bhoorsingh the Barasingha.
Refer: https://www.insightsonindia.com/2022/07/29/what-numbers-do-not-reveal-about-
tiger-conservation/
243. Consider the following statements regarding National Tiger Conservation Authority.
1. It is a statutory body under the Ministry of Environment, Forests and Climate
Change.
2. It is headed by the Prime Minister of India.
3. It accords approval for declaring new Tiger Reserves.
Which of the statements given above is/are correct?
(a) 1 and 2 only
(b) 1 and 3 only
(c) 2 and 3 only
(d) 1, 2 and 3
Ans: (b)
Explanation:
• The National Tiger Conservation Authority is a statutory body under the
Ministry of Environment, Forests and Climate Change.
o The National Tiger Conservation Authority has been fulfilling its mandate
within the ambit of the Wildlife (Protection) Act, 1972 for strengthening
tiger conservation in the country by retaining an oversight through
advisories/normative guidelines, based on appraisal of tiger status,

Telegram: https://t.me/insightsIAStips
121
Youtube: https://www.youtube.com/channel/UCpoccbCX9GEIwaiIe4HLjwA
Revision Through MCQs (RTM) Compilation (July - 2022)

ongoing conservation initiatives and recommendations of specially


constituted Committees.
• Set up under the Chairmanship of the Minister for Environment and
Forests.
• Functions of NTCA are as follows:
o Ensuring normative standards in tiger reserve management.
o Preparation of reserve specific tiger conservation plan.
o Laying down annual/ audit report before Parliament.
o Instituting State level Steering Committees under the Chairmanship of
Chief Minister and establishment of Tiger Conservation Foundation.
o According approval for declaring new Tiger Reserves.
Refer: https://www.insightsonindia.com/2022/07/29/what-numbers-do-not-reveal-about-
tiger-conservation/
244. The term ‘M-STrIPES’ is sometimes seen in the news in the context of
(a) Captive breeding of Wild Fauna
(b) Maintenance of Tiger Reserves
(c) Indigenous Satellite Navigation System
(d) Security of National Highways
Ans: (b)
Explanation:
• M-STrIPES (Monitoring system for tigers – intensive protection and ecological
status) using GPS to geotag photo-evidences and survey information, made this
exercise more accurate.
• CaTRAT (Camera Trap data Repository and Analysis Tool) for automated
segregation of camera trap photographs to species.
Refer: https://www.insightsonindia.com/2022/07/29/what-numbers-do-not-reveal-about-
tiger-conservation/
245. Consider the following statements:
1. The Global Tiger Initiative (GTI) program of the World Bank brought global partners
together to strengthen the tiger agenda.
2. Global Tiger Forum is an Inter-Governmental international body working exclusively
for the conservation of Tigers.
Which of the statements given above is/are correct?
(a) 1 only
(b) 2 only
(c) Both 1 and 2
(d) Neither 1 nor 2
Ans: (c)
Explanation:
• Global Tiger Initiative (GTI): read here>>
https://www.worldbank.org/en/topic/environment/brief/the-global-tiger-
initiative
o The Global Tiger Initiative (GTI) was launched in 2008 as a global alliance
of governments, international organizations, civil society, the
conservation and scientific communities and the private sector, with the
aim of working together to save wild tigers from extinction. In 2013, the
scope was broadened to include Snow Leopards.

Telegram: https://t.me/insightsIAStips
122
Youtube: https://www.youtube.com/channel/UCpoccbCX9GEIwaiIe4HLjwA
Revision Through MCQs (RTM) Compilation (July - 2022)

o The GTI’s founding partners included the World Bank, the Global
Environment Facility (GEF), the Smithsonian Institution, Save the Tiger
Fund, and International Tiger Coalition (representing more than 40 non-
government organizations). The initiative is led by the 13 tiger range
countries (TRCs).
• Global Tiger Forum engagement along with partners in areas of Conservation
Assured Tiger Standards (CA|TS), Security Audit of Tiger Reserves,
Management Planning etc.
o Global Tiger Forum is an Inter-Governmental international body working
exclusively for the conservation of Tigers.
Refer: https://www.insightsonindia.com/2022/07/29/what-numbers-do-not-reveal-about-
tiger-conservation/
246. With reference to RBI’s Digital Payments Index (RBI-DPI), consider the following
statements:
1. The index is formulated based on six parameters.
2. The base year of the RBI Digital Payment Index is 2018.
3. The index is published in semi – annual basis.
Which of the statements given above is/are correct?
(a) 1 and 2 only
(b) 1 and 3 only
(c) 2 and 3 only
(d) 1, 2 and 3
Ans: (c)
Explanation: RBI’s Digital Payments Index (RBI-DPI)
• Launched in January 2021, it captures the extent of digitisation of payments
across the country (with 2018 as a base year- score set as 100) and is
published semi-annually.
• The index is formulated based on five parameters. They are payment enablers
(25%), consumer centricity (5%), payment performance (45%), supply side
factors (15%), and demand side factors (10%).
Refer: facts for prelims: https://www.insightsonindia.com/2022/07/29/mission-2023-
insights-daily-current-affairs-pib-summary-29-july-2022/
247. Which among the following countries was the first in the world to officially sponsor
family planning programmes to control population?
(a) India
(b) China
(c) USA
(d) Sweden
Ans: (a)
Explanation:
• India was 1st country in the world to launch a National Programme for
Family Planning (1952), the National Commission on Population (2000), and
member of Family planning (FP) 2030 (a global initiative for family planning)
Refer: facts for prelims: https://www.insightsonindia.com/2022/07/29/mission-2023-
insights-daily-current-affairs-pib-summary-29-july-2022/
248. Among the following Acts, an Enforcement Directorate (ED) in India will be most
concerned with
(a) Negotiable Instrument Act

Telegram: https://t.me/insightsIAStips
123
Youtube: https://www.youtube.com/channel/UCpoccbCX9GEIwaiIe4HLjwA
Revision Through MCQs (RTM) Compilation (July - 2022)

(b) Public Debt Act


(c) Prevention of Money Laundering Act
(d) Narcotic Drugs and Psychotropic Substances Act
Ans: (c)
Explanation:
• Enforcement Directorate (ED) is a law enforcement agency and economic
intelligence agency responsible for enforcing economic laws and fighting
economic crime in India. It is part of the Department of Revenue, Ministry of
Finance.
• The prime objective of the Enforcement Directorate is the enforcement of two key
Acts- the Foreign Exchange Management Act 1999 (FEMA) and the
Prevention of Money Laundering Act 2002 (PMLA).
Refer: facts for prelims: https://www.insightsonindia.com/2022/07/29/mission-2023-
insights-daily-current-affairs-pib-summary-29-july-2022/
249. Consider the following statements regarding United Nations Convention on the Law of
the Sea (UNCLOS).
1. UNCLOS is a UN specialised agency that defines the rights and responsibilities of
nations with respect to their use of the world's oceans and the management of
marine natural resources.
2. International Seabed Authority (ISA) and International Tribunal for the Law of the
Sea (ITLOS) were established by the UNCLOS.
Which of the above statements is/are correct?
(a) 1 only
(b) 2 only
(c) Both 1 and 2
(d) Neither 1 nor 2
Ans: (b)
Explanation:
• The United Nations Convention on the Law of the Sea (UNCLOS) defines the
rights and responsibilities of nations with respect to their use of the world's
oceans, establishing guidelines for businesses, the environment, and the
management of marine natural resources.
• International Seabed Authority (ISA) was established to organize, regulate and
control all mineral-related activities in the international seabed area beyond the
limits of national jurisdiction (referred to as "the Area"), an area underlying most
of the world's oceans. It is an organization established by the United Nations
Convention on the Law of the Sea.
• The International Tribunal for the Law of the Sea (ITLOS) is an
intergovernmental organization created by the mandate of the Third United
Nations Conference on the Law of the Sea. It was established by the United
Nations Convention on the Law of the Sea.
Refer; facts for prelims: https://www.insightsonindia.com/2022/07/29/mission-2023-
insights-daily-current-affairs-pib-summary-29-july-2022/
250. Regarding Wood’s Dispatch, which of the following statements are true?
1. Grants-in-Aid system was introduced.
2. Establishment of universities was recommended.
3. English as a medium of instruction at all levels of education was recommended.
Select the correct answer using the code given below:

Telegram: https://t.me/insightsIAStips
124
Youtube: https://www.youtube.com/channel/UCpoccbCX9GEIwaiIe4HLjwA
Revision Through MCQs (RTM) Compilation (July - 2022)

(a) 1 and 2 only


(b) 2 and 3 only
(c) 1 and 3 only
(d) 1, 2 and 3
Ans: (a)
Explanation:
• Wood’s Despatch is called Magnacarta of English Education in India. As per this
despatch:
• An education department was to be set in every province.
• Universities on the model of the London University be established in big cities
such as Bombay, Calcutta and Madras.
• At least one government school be opened in every district.
• Affiliated private schools should be given grant in aid.
• The Indian natives should be given training in their mother tongue also.
• In accordance with Wood’s despatch, Education Departments were established
in every province and universities were opened at Calcutta, Bombay and Madras
in 1857 on the model of the London University.
• Later more universities were opened in Punjab in 1882 and at Allahabad 1887.
Refer: UPSC CSE 2018

RTM- REVISION THROUGH MCQS –30th-Jul-2022

251. Consider the following statements:


1. AI4Bharat is an initiative by IIT Madras
2. It has been aligned with the objectives of the Digital India Bhashini Mission.
Which of the statements given above is/are correct?
(a) 1 only
(b) 2 only
(c) Both 1 and 2
(d) Neither 1 nor 2
Ans: (c)
Explanation:
• AI4Bharat is an initiative by IIT Madras. This initiative would contribute to
and accelerate Indic language AI work as a public good.
• It has been aligned with the objectives of the Digital India Bhashini Mission.
• It has been started to build open-source language AI for Indian Languages.
• The Digital India Bhashini mission was launched with the objective of making all
services and information available to citizens in their own language.
Refer: https://www.insightsonindia.com/2022/07/31/overcoming-the-aryan-dravidian-
divide/
252. Who is the author of the book 'Arctic Home of the Vedas'?
(a) Gopal Krishna Gokhale
(b) Lala Lajpat Rai
(c) Bipin Chandra Pal
(d) Bal Gangadhar Tilak
Ans: (d)
Explanation:

Telegram: https://t.me/insightsIAStips
125
Youtube: https://www.youtube.com/channel/UCpoccbCX9GEIwaiIe4HLjwA
Revision Through MCQs (RTM) Compilation (July - 2022)

• The Arctic Home in the Vedas is a 1903 book on the origin of the Indo-Aryan
peoples by Indian nationalist, teacher and independence activist Bal Gangadhar
Tilak.
Refer: https://www.insightsonindia.com/2022/07/31/overcoming-the-aryan-dravidian-
divide/
253. Consider the following statements regarding the Places of Worship (Special Provisions)
Act, 1991:
1. It was enacted to freeze the status of all places of worship in the country as it was in
1991.
2. It will not apply to ancient and historical monuments and archaeological sites and
remains that are covered by the AMASR Act 1958.
3. It manifests the secular values of the Constitution and strictly prohibits
retrogression.
Which of the given above statements is/are correct?
(a) 1 and 2 only
(b) 3 only
(c) 2 and 3 only
(d) 1, 2 and 3
Ans: (c)
Explanation:
• S1: The Act says that no person shall convert any place of worship of any
religious denomination into one of a different denomination or section. It
contains a declaration that a place of worship shall continue to be as it was
on August 15, 1947.
• S2: The 1991 Act will not apply in some cases. It will not apply to ancient and
historical monuments and archaeological sites and remains that are covered by
the Ancient Monuments and Archaeological Sites and Remains Act, 1958. It will
also not apply to any suit that has been finally settled or disposed of, any
dispute that has been settled by the parties before the 1991 Act came into force,
or to the conversion of any place that took place by acquiescence.
• The Act specifically exempted from its purview the place of worship commonly
referred to as Ram Janmabhoomi-Babri Masjid in Ayodhya. It was done to allow
the pending litigation to continue as well as to preserve the scope for a
negotiated settlement.
• S3: In the 2019 Ayodhya verdict, the Constitution Bench led by former Chief
Justice of India Ranjan Gogoi referred to the law and said it manifests the
secular values of the Constitution and strictly prohibits retrogression.
Refer: https://www.insightsonindia.com/2022/07/31/supreme-court-places-of-worship-
act-cant-be-enforced-within-the-same-religion/
254. Consider the following statements about Comptroller and Auditor General of India
(CAG):
1. The CAG is mentioned in the Constitution of India.
2. He is the head of the Indian Audit and Accounts Department.
3. His duty is to uphold the Constitution of India and the laws of Parliament in the
field of financial administration.
Which of the statements given above is/are correct?
(a) 1 and 2 only
(b) 2 and 3 only

Telegram: https://t.me/insightsIAStips
126
Youtube: https://www.youtube.com/channel/UCpoccbCX9GEIwaiIe4HLjwA
Revision Through MCQs (RTM) Compilation (July - 2022)

(c) 1 and 3 only


(d) 1, 2 and 3
Ans: (d)
Explanation:
• The Constitution of India provides for an independent office of the Comptroller
and Auditor General of India (CAG) in chapter V under Part V.
• The CAG is mentioned in the Constitution of India under Article 148 – 151.
• He is the head of the Indian Audit and Accounts Department.
• He is the guardian of the public purse and controls the entire financial system of
the country at both the levels- the centre and state.
• His duty is to uphold the Constitution of India and the laws of Parliament in the
field of financial administration.
Refer: https://www.insightsonindia.com/2022/07/31/need-for-a-new-legal-framework-
governing-telecommunication-in-india/
255. ‘Paryushan Parva’ is related to which community?
(a) Jain
(b) Buddha
(c) Sindhi
(d) Sikh
Ans: (a)
Explanation: Pryushan Parv (festival)
• It is usually celebrated in August or September (rainy season)
• During Paryushan, Jains increase their level of spiritual intensity often
using fasting and prayer/meditation to help.
• The five main vows are eSeekho aur Kamao (Learn and Earn) schememphasized
during this time- Ahiṃsā(Non-violence), Satya (Truth), Asteya (Non-stealing),
Brahmacharya (Chastity), Aparigraha (Non-possession)
• Pratikraman is also performed by many Jains during the festival. The word
Pratikraman is made from the combination of two words, Pra meaning return
and atikraman meaning violation.
Refer: facts for prelims: https://www.insightsonindia.com/2022/07/31/mission-2023-
insights-daily-current-affairs-pib-summary-30-july-2022/
256. With reference to ‘Seekho aur Kamao’ (Learn and Earn) scheme, consider the following
statements:
1. It is a Skill development scheme for the youth of the 19 – 30 years age group.
2. The scheme implemented by the Ministry of Minority Affairs.
Which of the statements given above is/are correct?
(a) 1 only
(b) 2 only
(c) Both 1 and 2
(d) Neither 1 nor 2
Ans: (b)
Explanation: About the Scheme
• Nodal ministry: Central Sector Scheme under the Ministry of Minority
Affairs (since 2013-14)
• Aim: Upgrading the skills of minority youth (14-35 years age group) and
ensure 75% placements, out of which 50% should be in the organized sector.

Telegram: https://t.me/insightsIAStips
127
Youtube: https://www.youtube.com/channel/UCpoccbCX9GEIwaiIe4HLjwA
Revision Through MCQs (RTM) Compilation (July - 2022)

Post placement support of Rs. 2000/- per month is provided to placed trainees
for two months as placement assistance.
• Implementation: Through selected Project Implementing Agencies (PIAs).
Refer: facts for prelims: https://www.insightsonindia.com/2022/07/31/mission-2023-
insights-daily-current-affairs-pib-summary-30-july-2022/
257. ‘Naya Savera Scheme’ is related/aims to
(a) Leadership development of women belonging to minority communities
(b) Upgrading the Skills and Training in Traditional Arts/Crafts for Development
(c) interest subsidy on educational loans for overseas higher studies
(d) provide free coaching to minority students for the various competitive exam
Ans: (d)
Explanation:
• Naya Savera Scheme (to provide free coaching to minority students for the
various competitive exam)
• Padho Pardesh Scheme (interest subsidy on educational loans for overseas
higher studies)
• Nai Udaan Scheme (supports students clearing Prelims of UPSC, SPSC or other
exams)
• Nai Roshni Scheme ( Leadership development of women belonging to minority
communities)
• USTTAD (Upgrading the Skills and Training in Traditional Arts/Crafts for
Development)
• Nai Manzil Scheme (for formal school education & skilling of school
dropouts)
• Hamari Dharohar (to preserve the rich heritage of minority communities of India)
Refer: facts for prelims: https://www.insightsonindia.com/2022/07/31/mission-2023-
insights-daily-current-affairs-pib-summary-30-july-2022/
258. With reference to United Nations Human Rights Council (UNHRC), consider the
following statements:
1. The United Nations Human Rights Council (UNHRC) is a United Nations body whose
mission is to promote and protect human rights around the world.
2. The Council has 47 members elected for staggered three-year terms on a regional
group basis.
3. The headquarters of the Council is in Geneva, Switzerland.
Which of the statements given above is/are correct?
(a) 1 and 2 only
(b) 2 and 3 only
(c) 1 and 3 only
(d) 1, 2 and 3
Ans: (d)
Explanation:
• The United Nations Human Rights Council (UNHRC) is a United Nations
body whose mission is to promote and protect human rights around the world.
The Council has 47 members elected for staggered three-year terms on a
regional group basis. The headquarters of the Council is in Geneva,
Switzerland.
• Functions:

Telegram: https://t.me/insightsIAStips
128
Youtube: https://www.youtube.com/channel/UCpoccbCX9GEIwaiIe4HLjwA
Revision Through MCQs (RTM) Compilation (July - 2022)

• The UNHRC passes non-binding resolutions on human rights issues through a


periodic review of all 193 UN member states called the Universal Periodic Review
(UPR).
• It oversees expert investigation of violations in specific countries (Special
Procedures).
Refer: facts for prelims: https://www.insightsonindia.com/2022/07/31/mission-2023-
insights-daily-current-affairs-pib-summary-30-july-2022/
259. Aridity Anomaly Outlook Index is released by which of the following?
(a) Indian Meteorological Department
(b) NITI Aayog
(c) Indian Space Research Organisation
(d) National Physical Laboratory of India
Ans: (a)
Explanation:
• Recently Indian Meteorological Department (IMD) released the Index for the
month of July.
• Key findings
o Nearly 660 of 756 districts (85%) were facing different degrees of aridity,
while only 63 are non-arid.
o At least 196 districts are in the grip of a ‘severe’ degree of dryness and 65
of these are in Uttar Pradesh
o Nearly 69% of India is dry land
o Applications: Impacts of drought in agriculture, especially in the tropics
where defined wet and dry seasons are part of the climate regime. Both
winter and summer cropping seasons can be assessed using this method.
Refer: facts for prelims: https://www.insightsonindia.com/2022/07/31/mission-2023-
insights-daily-current-affairs-pib-summary-30-july-2022/

260. Consider the following statements:


1. Agriculture Census Scheme is being undertaken by Ministry of Statistics and
Programme Implementation (MoSPI) since 1970-71.
2. Under this exercise, Centre government provides funds to states to do entire work.
Which of the statements given above is/are correct?
(a) 1 only
(b) 2 only
(c) Both 1 and 2
(d) Neither 1 nor 2
Ans: (b)
Explanation:
• Agriculture Census Scheme is being undertaken by Agriculture Ministry since
1970-71. For this, 1970-71 was taken as reference year.
• So far, ten Agriculture Censuses have been conducted. Last edition of
Agriculture census was conducted in 2015-16.
• Under this exercise, centre government provides funds to states to do
entire work.
Refer: facts for prelims: https://www.insightsonindia.com/2022/07/31/mission-2023-
insights-daily-current-affairs-pib-summary-30-july-2022/

Telegram: https://t.me/insightsIAStips
129
Youtube: https://www.youtube.com/channel/UCpoccbCX9GEIwaiIe4HLjwA
Revision Through MCQs (RTM) Compilation (July - 2022)

Telegram: https://t.me/insightsIAStips
130
Youtube: https://www.youtube.com/channel/UCpoccbCX9GEIwaiIe4HLjwA

You might also like